You are on page 1of 116

Mục lục

1 CÁC KHÁI NIỆM CƠ BẢN CỦA SỐ HỌC 3


1 Phép chia hết trong tập số nguyên . . . . . . . . . . . . . . . . . . . . . . . . . . 3
I. Các khái niệm . . . . . . . . . . . . . . . . . . . . . . . . . . . . . . . . . . 3
II. Tính chất . . . . . . . . . . . . . . . . . . . . . . . . . . . . . . . . . . . . 3
III. Ví dụ minh họa . . . . . . . . . . . . . . . . . . . . . . . . . . . . . . . . . 4
IV. Bài tập . . . . . . . . . . . . . . . . . . . . . . . . . . . . . . . . . . . . . 8

hu
2 Hợp số - số nguyên tố . . . . . . . . . . . . . . . . . . . . . . . . . . . . . . . . . . 15
I. Định nghĩa . . . . . . . . . . . . . . . . . . . . . . . . . . . . . . . . . . . 15
II. Đinh lí . . . . . . . . . . . . . . . . . . . . . . . . . . . . . . . . . . . . . . 15

T
III. Định lí cơ bản của số học . . . . . . . . . . . . . . . . . . . . . . . . . . . 15
IV. Ví dụ minh họa . . . . . . . . . . . . . . . . . . . . . . . . . . . . . . . . . 16
V. Bài tập . . . . . . . . . . . . . . . . . . . .
t . . . . . . . . . . . . . . . . . 20
3 Ước chung lớn nhất - Bội chung nhỏ nhất . . . . . . . . . . . . . . . . . . . . . . . 27
Tấ
I. Ước chung lớn nhất . . . . . . . . . . . . . . . . . . . . . . . . . . . . . . . 27
II. Bội chung nhỏ nhất . . . . . . . . . . . . . . . . . . . . . . . . . . . . . . . 27
III. Ví dụ minh họa . . . . . . . . . . . . . . . . . . . . . . . . . . . . . . . . . 28
IV. Bài tập . . . . . . . . . . . . . . . . . . . . . . . . . . . . . . . . . . . . . 31
4 Đồng dư . . . . . . . . . . . . . . . . . . . . . . . . . . . . . . . . . . . . . . . . . 34
n

I. Các khái niệm về đồng dư . . . . . . . . . . . . . . . . . . . . . . . . . . . 34


II. Đồng dự tuyến tính . . . . . . . . . . . . . . . . . . . . . . . . . . . . . . . 37
yễ

III. Hệ thặng dư đầy đủ - Hệ thặng dư thu gọn . . . . . . . . . . . . . . . . . . 38


IV. Các định lí về đồng dư . . . . . . . . . . . . . . . . . . . . . . . . . . . . . 39
1. Định lí Phecma nhỏ . . . . . . . . . . . . . . . . . . . . . . . . . 39
gu

2. Định lí Ơ le . . . . . . . . . . . . . . . . . . . . . . . . . . . . . . 41
3. Định lí Wilson . . . . . . . . . . . . . . . . . . . . . . . . . . . . 42
4. Định lí thặng dư trung hoa . . . . . . . . . . . . . . . . . . . . . 43
N

V. Bài tập . . . . . . . . . . . . . . . . . . . . . . . . . . . . . . . . . . . . . 46
5 Hàm số học . . . . . . . . . . . . . . . . . . . . . . . . . . . . . . . . . . . . . . . 58
I. Hàm phần nguyên . . . . . . . . . . . . . . . . . . . . . . . . . . . . . . . 58
II. Số các ước của số tự nhiên . . . . . . . . . . . . . . . . . . . . . . . . . . . 62
III. Hàm tổng các ước nguyên dương của n . . . . . . . . . . . . . . . . . . . . 64
1. Định nghĩa . . . . . . . . . . . . . . . . . . . . . . . . . . . . . . 64
IV. Bài tập . . . . . . . . . . . . . . . . . . . . . . . . . . . . . . . . . . . . . 65

2 PHƯƠNG TRÌNH DIOPHANTINE 79


1 Một số phương trình Diophantine cổ điển . . . . . . . . . . . . . . . . . . . . . . . 79
I. Phương trình bậc nhất . . . . . . . . . . . . . . . . . . . . . . . . . . . . . 79
1. Phương trình bậc nhất hai ẩn . . . . . . . . . . . . . . . . . . . . 79
2. Phương trình bậc nhất nhiều ẩn . . . . . . . . . . . . . . . . . . . 80
II. Phương trình bậc hai . . . . . . . . . . . . . . . . . . . . . . . . . . . . . . 86
1. Phương trình Pitago . . . . . . . . . . . . . . . . . . . . . . . . . 86
2 Một số phương pháp giải phương trình nghiệm nguyên . . . . . . . . . . . . . . . 86

1
MỤC LỤC

I. Đưa về phương trình tích . . . . . . . . . . . . . . . . . . . . . . . . . . . . 86


II. Phương pháp lùi vô hạn . . . . . . . . . . . . . . . . . . . . . . . . . . . . 97
III. Phương pháp chọn môđunlô . . . . . . . . . . . . . . . . . . . . . . . . . . 104
IV. Phương pháp quy nạp . . . . . . . . . . . . . . . . . . . . . . . . . . . . . 110

hu
T
t
Tấ
n
yễ
gu
N

2
Chương 1

CÁC KHÁI NIỆM CƠ BẢN CỦA SỐ


HỌC

§1. Phép chia hết trong tập số nguyên

hu
I. Các khái niệm

T
Định lí 1. Cho hai số nguyên a và b (b 6= 0). Khi đó tồn tại duy nhất hai số nguyên q, r sao
cho
a = b · q + r, 0 ≤ r < |b|.
t (1)
Tấ
Lời giải.
Ta xét các trường hợp sau

• Nếu a ≥ b > 0, khi đó a − b ≥ 0. Nếu a − b ≥ b thì a − 2b ≥ 0 và quá trình trên tiếp tục
đến khi nào đó, ta có 0 ≤ a − qb < b. Đặt r = a − qb ta có a = qb + r với 0 ≤ r < b.
n

• Nếu 0 < a < b thì ta có a = 0 · b + a.


yễ

• Với trường hợp trong hai số a, b có một số âm thì ta đi xét trị tuyệt đối của số đó.
gu


Định nghĩa 1. Với hai số nguyên a, b thỏa đẳng thức (1) thì ta nói

• Số q được gọi là thương và r được gọi là số dư trong phép chia a cho b.


N

• Nếu r = 0, tức là a = bq thì ta nói a chia hết cho b hoặc b chia hết a, hay còn nói a là bội
.
của b hoặc b là ước của a. Kí hiệu a .. b hoặc b | a.
.
• Nếu r 6= 0 thì ta nói a không chia hết cho b, kí hiệu a 6 .. b hoặc b 6 | a.

II. Tính chất


Định lí 2. Cho a, b, c là các số nguyên. Khi đó, ta có các tính chất sau:

• Nếu a | b, b 6= 0 thì |a| ≤ |b|.

• Nếu a | b và a | cthì a | m · b + n · c với mọi số nguyên m và n.

• Nếu a | b và a | b ± c thì a | c.

• a | a (tính phản xạ).

3
1. PHÉP CHIA HẾT TRONG TẬP SỐ NGUYÊN

• Nếu a | b và b | c thì a | c(tính bắc cầu).

• Nếu a | b và b | a thì |a| = |b|.

III. Ví dụ minh họa

Ví dụ 1.1. Tìm các số nguyên x để biểu thức sau nhận giá trị nguyên

2x3 − x2 + x − 1
A= .
2x − 1

Lời giải.
Ta có
1 1
A = x2 + − .
2 2(2x − 1)
Suy ra

hu
1
2A = 2x2 + 1 − .
2x − 1
1
Vì A nguyên nên 2A nguyên, do đó nguyên hay 2x − 1 là ước của 1.

T
2x − 1
Suy ra x = 0 hoặc x = 1.
Với x = 0 ta có A = 1, với x = 1 ta có A = 1. t
Vậy x = 0,x = 1 là những giá trị cần tìm. 
Tấ
Ví dụ 1.2. Tìm các số tự nhiên x để biểu thức sau là số nguyên

2x3 − x2 + 5x + 4
A= .
x2 + 1
n

Lời giải.
yễ

Ta có
3x + 5
A = 2x − 1 + .
x2 + 1
gu

Vì x ∈ Z nên A ∈ Z khi và chỉ khi


3x + 5
∈Z
x2 + 1
Suy ra
N

3x + 5 ≥ x2 + 1 ⇔ x2 − 3x − 4 ≤ 0 ⇔ −1 ≤ x ≤ 4.
Mà x là số tự nhiên nên x ∈ {0; 1; 2; 3; 4}. Thay lần lượt các giá trị của x ta có x ∈ {0; 1; 4}.


Ví dụ 1.3. Chứng minh rằng


.
A = n(n2 + 1)(n2 + 4)..5

với mọi số nguyên dương n.

Lời giải.
Ta xét các trường hợp sau
.
• n = 5k, ta có A .. 5.
. .
• n = 5k ± 1, ta có n2 + 4 .. 5 nên A .. 5.

4
1. PHÉP CHIA HẾT TRONG TẬP SỐ NGUYÊN

. .
• n = 5k ± 2, ta có n2 + 1 .. 5 nên A .. 5.
.
Vậy A .. 5 với mọi số nguyên dương n. 

Ví dụ 1.4. Cho a,b là các số nguyên. Chứng minh rằng


. .
8a + 9b .. 31 ⇔ a + 5b .. 31.

Lời giải.
Ta có
5(8a + 9b) − 9(a + 5b) = 31a.
Do (5; 31) = (9; 31) = 1 nên
. .
8a + 9b .. 31 ⇔ a + 5b .. 31.


hu
Ví dụ 1.5. Cho các số nguyên x, y. Chứng minh rằng
.
a) xn − y n .. x − y với mọi số tự nhiên n.

T
.
b) xn + y n .. x + y với mọi số tự nhiên n lẻ.

Lời giải.
t
Tấ
a) Với mọi số tự nhiên n, ta có

xn − y n = (x − y)(xn−1 + y n−2 x + . . . + y n−1 ).


.
Suy ra xn − y n .. x − y.
n
yễ

b) Với n là số tự nhiên lẻ, ta có

xn + y n = (x + y)(xn−1 − xn−2 y + . . . + y n−1 ).


gu

.
Suy ra xn + y n .. x + y.


N

Ví dụ 1.6. Tìm số nguyên dương lớn nhấtx để 23x+26 chia hết 2014!.

Lời giải.  
2014
Từ 1 đến 2014 có = 87 số chia hết cho 23. Trong đó có ba số chia hết cho 232 là
23
232 , 2.232 , 3.232 .
.
Do đó 2014! .. 2390 và 2014! không chia hết cho 2391 . Suy ra x + 26 = 90 ⇒ x = 64. 
n+1 n
Ví dụ 1.7. Cho n ≥ 2 là số nguyên dương. Chứng minh rằng 22 + 22 + 1 có ít nhất 3
ước nguyên dương lớn hơn 1.

Lời giải.
T 
a có 2
a4 + a2 + 1 = a2 + 1 − a2 = a2 + a + 1 a2 − a + 1 .
 

5
1. PHÉP CHIA HẾT TRONG TẬP SỐ NGUYÊN

Suy ra
 n−1 4  n−1 2
2n+1 2n
2 +2 + 1 = 22 + 22 +1
 n n−1
  n n−1

= 22 − 22 + 1 22 + 22 + 1
 n   n−2 4  n−2 2 
2 2n−1 2 2
= 2 −2 +1 2 + 2 +1
 n   n−1   n−1 
2 2n−1 2 2n−2 2 2n−2
= 2 −2 +1 2 +2 +1 2 −2 +1 .

Từ đó, ta có đpcm.
.
Ví dụ 1.8. Cho n là số nguyên dương thỏa 3n − 1 .. 22014 . Chứng minh rằng n ≥ 22012 .

Lời giải.
Đặt n = 2k .m với m lẻ. Khi đó

hu
 k   k m−1  k m−2 
n 2 2 2 2k
3 −1= 3 −1 3 + 3 + ··· + 3 + 1 .

Vì m lẻ nên

T
 k
m−1  k m−2 k
32 + 32 + · · · + 32 + 1
là số lẻ. t
Do đó
. .
3n − 1 .. 22014 ⇔ 32 − 1 .. 22014 .
k
Tấ
Mà   22   k−1 
2k 2 2
3 − 1 = (3 − 1) (3 + 1) 3 + 1 3 + 1 · · · 3 +1
 2
  k−1

= 23 32 + 1 32 + 1 · · · 32 + 1 .

n

m 
Vì 32 + 1 m = 1,k − 1 chỉ chia hết cho 2 mà không chia hết cho 4 nên
yễ

.
32 − 1 .. 22014 ⇔ k + 2 ≥ 2014 ⇔ k ≥ 2012.
k

Suy ra n ≥ 22012 . 
gu

Ví dụ 1.9. Tìm tất cả các số nguyên a, b, c với 1 < a < b < c sao cho (a − 1) (b − 1) (c − 1)
là một ước của abc − 1.
N

Lời giải.
Đặt x = a − 1, y = b − 1, z = c − 1 ta có 0 < x < y < z, khi đó xyz là ước của
(x + 1) (y + 1) (z + 1) − 1 = xyz + xy + yz + zx + x + y + z.
Hay xyz là ước của xy + yz + zx + x + y + z.
Suy ra
xy + yz + zx + x + y + z 1 1 1 1 1 1
= + + + + + = f (x,y,z)
xyz x y z xy yz zx
là số nguyên dương.
Ta có
5
f (x,y,z) ≤ f (1,2,3) = 2 + < 3 ⇒ f (x,y,z) = 1
6
hoặc f (x,y,z) = 2.
Mặt khác
59
f (3,4,5) = < 1 ⇒ x ∈ {1,2} .
60
6
1. PHÉP CHIA HẾT TRONG TẬP SỐ NGUYÊN

• f (x,y,z) = 1 ⇔ xy + yz + zx + x + y + z = xyz. (1)

– x = 1, khi đó (1) ⇔ 2(y + z) + 1 = 0 vô nghiệm.


– x = 2, khi đó
 
y−3=1 y=4
(1) ⇔ 3 (y + z) + 2 = yz ⇔ (y − 3) (z − 3) = 11 ⇔ ⇔ .
z − 3 = 11 z = 14

• f (x,y,z) = 2 ⇔ xy + yz + zx + x + y + z = 2xyz. (2)

– x = 1, khi đó
 
y−2=1 y=3
(2) ⇔ 2(y + z) + 1 = yz ⇔ (y − 2) (z − 2) = 5 ⇔ ⇔
z−2=5 z=7

vô nghiệm.
– x = 2, khi đó (2) ⇔ 3 (y + z) + 2 = 3yz vô nghiệm.

hu
Vậy (a; b; c) = (2; 4; 8) , (3; 5; 16) . 

T
a2
Ví dụ 1.10. Tìm tất cả các cặp nguyên dương (a,b) sao cho là một số
2ab2 − b3 + 1
nguyên dương.

Lời giải.
t
Tấ
a2
Vì là số nguyên dương nên 2ab2 − b3 + 1 = b (2a − b) + 1 > 0.
2ab2 − b3 + 1
Suy ra 2a − b ≥ 0. Ta có a2 ≥ 2ab2 − b3 + 1. (*)

• Nếu b = 2a thì (*) đúng.


n

• Nếu
yễ

b > 2a ⇒ a2 ≥ b2 (2a − 1 − b) + b2 + 1 > b2 ⇒ a > b


Do đó, ta luôn có b = 2a hoặc a > b.
gu

Đặt
a2
= k ∈ Z+ .
2ab2 − b3 + 1
N

Hay a2 − 2ab2 k + kb3 − k = 0. (1)


Với mỗi k thì phương trình (1) có hai nghiệm nguyên không âm a1 ≥ a2 .
Theo định lí Viet, ta có
a1 + a2 = 2b2 k ⇒ a1 ≥ b2 k
k (b3 − 1)
a1 a2 = k b 3 − 1 ⇒ a2 =

≤ b.
a1
Suy ra a2 = 0 hoặc b = 2a2 .

• a2 = 0 ⇒ b3 − 1 = 0 ⇒ b = 1 ⇒ a = 2k.
b2 b4 b
• b = 2a2 ⇒ − k = 0 ⇒ b2 = 4k và a1 = − .
4 2 2
Suy ra k = m2 ⇒ a2 = m,a1 = 8m4 − m.
Vậy (a; b) ∈ {(2m; 1) , (m; 2m) , (8m4 − m; 2m)} với m ∈ Z+ . 

7
1. PHÉP CHIA HẾT TRONG TẬP SỐ NGUYÊN

IV. Bài tập


.
Bài 1.1. Tìm số tự nhiên n để n2 + 1 .. n + 1.
Lời giải.
Ta có
n2 + 1 2
=n−1+ .
n+1 n+1
Từ đây, suy ra n + 1 = 1 hoặc n + 1 = 2. Hay là n = 0,n = 1. 
Bài 1.2. Tìm số nguyên x sao cho

3x2 + 2x − 1
A=
x2 + 1

là số nguyên.
Lời giải.
Ta có

hu
2x − 4
A=3+ .
x2 + 1
2x − 4
A là số nguyên khi và chỉ khi là số nguyên. Suy ra

T
x2 + 1

|2x − 4| ≥ x2 + 1 ⇔ x2 + 2x − 3 ≤ 0 ⇔ −1 ≤ x ≤ 3.
t
Mà x nguyên nên x ∈ {−1; 0; 1; 2; 3}. Thử lại ta có x ∈ {−1; 0; 1; 2}. 
Tấ
Bài 1.3. Cho số nguyên x thỏa mãn 17|3x + 2. Chứng minh rằng 17 là ước của

6x3 + x2 − 5x + 15.
n

Lời giải.
yễ

Ta có
.
6x3 + x2 − 5x + 15 = 6x3 + x2 − 5x − 2 + 17 = (3x + 2)(2x2 − x − 1) − 17 .. 17.
gu


Bài 1.4. Chứng tỏ rằng với mọi số tự nhiên n, giữa n2 và (n + 1)2 có thể tìm thấy ba số tự nhiên
N

a, b, c sao cho c | a2 + b2 .
Lời giải.
Chọn a = n2 + 2,b = n2 + n + 1,c = n2 + 1 . 
Bài 1.5. Cho số tự nhiên n. Chứng minh rằng

n5 n4 n3 n
+ + −
5 2 3 30
là số nguyên.
Lời giải.
Ta có
n5 n4 n3 n 6n5 + 15n4 + 10n3 − n
+ + − =
5 2 3 30 30
Do
6n5 + 15n4 + 10n3 − n = n(n + 1)(2n + 1)(3n2 + 3n − 1)
. .
Mà n(n + 1)(2n + 1) .. 6. Ta chứng minh a(n) = 6n5 + 15n4 + 10n3 − n .. 5 (1).

8
1. PHÉP CHIA HẾT TRONG TẬP SỐ NGUYÊN

.
• n = 5k thì a(n) .. 5.
. .
• n = 5k + 1 hoặc n = 5k − 2 thì 3n2 + 3n − 1 .. 5 nên a(n) .. 5.
. .
• n = 5k − 1 thì n + 1 .. 5 nên a(n) .. 5.
. .
• n = 5k + 2 thì 2n + 1 .. 5 nên a(n) .. 5.

Từ đó ta có đpcm. 
Bài 1.6. Chứng minh rằng
120|n5 − 5n3 + 4n
với mọi số tự nhiên n.
Lời giải.
Ta có 120 = 23 .3.5 và

n5 − 5n3 + 4n = n(n2 − 1)(n2 − 4) = (n − 2)(n − 1)n(n + 1)(n + 2)

hu
Trong 5 số tự nhiên liên tiếp luôn tồn tại ít nhất một số chia hết cho 5, một số chia hết cho 3 và
có ít nhất hai số chẵn liên tiếp nên tích hai số chẵn này chia hết cho 8.

T
Do đó: n5 − 5n3 + 4n chia hết cho 120. 
Bài 1.7. Cho n là số nguyên dương. Chứng minh rằng
t .
(n + 1)(n + 2) . . . (2n) .. 2n .
Tấ
Lời giải.
Ta chứng minh bài toán bằng phương pháp quy nạp.
n

Với n = 1 ta thấy bài toán đúng.


Giả sử
.
yễ

(n + 1)(n + 2) . . . (2n) .. 2n
Ta chứng minh
.
(n + 2)(n + 3) . . . (2n).(2n + 1)(2n + 2) .. 2n+1 .
gu

Thật vậy
.
N

(n + 2)(n + 3) . . . (2n).(2n + 1)(2n + 2) = (n + 1)(n + 2) . . . (2n).(2n + 1).2 .. 2n+1 .

Vậy bài toán được chứng minh. 


Bài 1.8. Xác định tất cả các cặp số (a,b) nguyên dương sao cho ab2 + b + 7 chia hết a2 b + a + b.
Lời giải.
Ta có ab2 + b + 7 |a2 b + a + b nên

ab2 + b + 7 b(a2 b + a + b) − a(ab2 + b + 7) = b2 − 7a

• b2 = 7a ⇒ a = 7k 2 ,b = 7k với k ∈ N.
• b2 − 7a > 0 ⇒ ab2 + b + 7 ≤ b2 − 7a (vô lí).
Vì ab2 + b + 7 > b2 > b2 − 7a.
• b2 − 7a < 0 ⇒ ab2 + b + 7 ≤ 7a − b2 ⇒ b2 < 7 ⇒ b = 1,b = 2. 
. a + 8 = 57 a = 49
+) b = 1 ⇒ 7a − 1 = 7 (a + 8) − 57 .. a + 8 ⇒ ⇒
a + 8 = 19 a = 11

9
1. PHÉP CHIA HẾT TRONG TẬP SỐ NGUYÊN

. .
+) b = 2 ⇒ 7a − 4 = 2 (4a + 9) − (a + 22) .. 4a + 9 ⇒ a + 22 .. 4a + 9 .
Suy ra
13
a + 22 ≥ 4a + 9 ⇒ a ≤ ⇒ a ∈ {1,2,3,4} .
3
Thử lại ta thấy không có a thỏa bài toán.
Vậy (a; b) = (7k 2 ; 7k) , (49; 2) , (11; 2). 
Bài 1.9. Tìm tất cả các số nguyên dương n sao cho với mọi số nguyên lẻ a mà a2 < n thì a|n.
Lời giải.
Gọi a là số nguyên lẻ lớn nhất mà a2 < n, suy ra n ≤ (a + 2)2 .
Vì a,a − 2,a − 4 là các số lẻ không nên n chia hết cho a,a − 2,a − 4.
Suy ra
a (a − 2) (a − 4) ≤ (a + 2)2 ⇔ a3 − 7a2 + 4a − 4 ≤ 0 ⇒ a < 7.
Do đó a ∈ {1,3,5}.
• Nếu a = 1 ta có: 1 ≤ n ≤ 9, từ đây ta tìm được n = 3.

hu
• Nếu a = 3 ta có 9 < n ≤ 25, từ đây ta tìm được n ∈ {12; 15; 18; 21; 24}.

• Nếu a = 5 ta có 25 < n ≤ 49, từ đây ta tìm được n ∈ {30,45}.

T
Vậy n ∈ {3; 12; 15; 18; 21; 24; 30; 45}. 
Bài 1.10. Tìm tất cả các số nguyên dương (x,n) sao cho xn +2n +1 là một ước của xn+1 +2n+1 +1.
t
Lời giải.  
.. x+3=7 x=4
Tấ
2
• n = 1 ⇒ x + 5 = (x + 3) (x − 3) + 14 . x + 3 ⇒ ⇒ .
x + 3 = 14 x = 11
• Xét n ≥ 2.
+) Với x ∈ {1,2,3} ta có

1 + 2n + 1 < 1 + 2n+1 + 1 < 2 (1 + 2n + 1)


n

2n + 2n + 1 < 2n+1 + 2n+1 + 1 < 2 (2n + 2n + 1)


yễ

2 (3n + 2n + 1) < 3n+1 + 2n+1 + 1 < 3 (3n + 2n + 1)


Do đó xn + 2n + 1 không là ước của xn+1 + 2n+1 + 1 với x ∈ {1,2,3}.
gu

+) Xét x ≥ 4, ta có xn+1 + 2n+1 + 1 < x (xn + 2n + 1)


Ta chứng minh
xn+1 + 2n+1 + 1 > (x − 1) (xn + 2n + 1) . (1)
N

Thật vậy
(1) ⇔ xn+1 + 2n+1 + 1 > xn+1 + x.2n + x − xn − 2n − 1
⇔ xn + 2n+1 + 2n + 2 > x (2n + 1) (2)
Ta có
n xn xn 22n + x2
x = + ≥ .
2 2 2
Suy ra
n n+1 22n + x2 n+1 n 22n + 2.2n + 1 + x2
x +2 +2> +2 +2 +2>
2 2
n 2 2
(2 + 1) + x
= ≥ x (2n + 1) ⇒ (2) đúng.
2
Vậy
(x − 1) (xn + 2n + 1) < xn+1 + 2n+1 + 1 < x (xn + 2n + 1)
Suy ra xn + 2n + 1 không là ước của xn+1 + 2n+1 + 1. 

10
1. PHÉP CHIA HẾT TRONG TẬP SỐ NGUYÊN

Bài 1.11. Cho tam thức f (x) = ax2 + bx + c trong đó a,b,c ∈ Z, a chẵn và b lẻ. Chứng minh
rằng với mỗi số nguyên dương n, tồn tại số nguyên dương x sao cho

2n ax2 + bx + c .

Lời giải.
• n = 1 ta chọn x1 lẻ nếu c lẻ và x1 chẵn nếu c chẵn. Khi đó

21 ax21 + bx1 + c

• Giả sử
2n ax2n + bxn + c = P (xn )
với xn ∈ N∗, ta chỉ ra tồn tại xn+1 ∈ N∗ để

2n+1 ax2n+1 + bxn+1 + c = P (xn+1 )

hu
Nếu 2n+1 |P (xn ) thì ta chọn xn+1 = xn .
Ngược lại, P (xn ) = 2n .d với d ∈ N,d lẻ.
Ta có

T
P (x) − P (xn ) = (x − xn ) [a (x + xn ) + b]
với a (x + xn ) + b lẻ với mọi x ∈ N.
Đặt xn+1 = xn + 2n .k với k ∈ N ∗ ,k lẻ.
Ta có:
t
Tấ
P (xn+1 ) = P (xn ) + 2n .k [a (xn+1 + xn ) + b]
= 2n [d + k (a(xn+1 + xn ) + b)]
Do d + k (a(xn+1 + xn ) + b) là số chẵn nên ta có:
n

2n+1 |P (xn+1 ) .
yễ


Bài 1.12. Cho các số nguyên a,b,c với b 6= c. Chứng minh rằng nếu các phương trình ax2 +bx+c =
gu

0 và (c − b) x2 + (c − a) x + a + b có nghiệm chung thì a + b + 2cchia hết cho 3.


Lời giải.
Gọi x0 là nghiệm chung của hai phương trình đã cho.
N

Đặt
f (x) = ax2 + bx + c, g(x) = (c − b) x2 + (c − a) x + a + b
Ta có
f (x) − g(x) = (a + b − c) x2 + x − 1 .


Nếu a + b − c 6= 0thì x0 là nghiệm của phương trình x2 + x − 1 = 0 nên x0 là số vô tỉ.


Theo thuật toán Ơclit, ta có

f (x) = m x2 + x − 1 + nx + p


với m,n,p ∈ Z.
Ta có f (x0 ) = 0 ⇒ nx0 + p = 0 điều này vô lí.
.
Do đó, ta có a + b − c = 0 ⇒ a + b + 2c = 3c .. 3. 

11
1. PHÉP CHIA HẾT TRONG TẬP SỐ NGUYÊN

n2
Bài 1.13. Cho các số nguyên dương m,n thỏa m ≤ và nếu p là ước nguyên tố của m thì
4
.
p ≤ n. Chứng minh rằng n! .. m.
Lời giải.
Giả sử p là ước nguyên tố của m và k là số mũ cao nhất của p trong phân tích đó.
.
Ta chứng minh n! .. pk .
.
Với k = 1, vì p ≤ n nên n! .. p.
n2
Vì m ≤ nên
4
n2 p
pk ≤ ⇒ n ≥ 2 pk .
4
Ta chứng minh n ≥ kp, bằng cách chứng minh
k−2
p k
2 pk ≥ kp ⇔ p 2 ≥ . (∗)
2

hu
Với k = 2 thì (*) đúng.
Với k = 3 thì (*) đúng với p ≥ 3, còn với

T
.
p = 2 ⇒ n ≥ 5 ⇒ n! .. 8 = 23

Với k ≥ 4, áp dụng bđt Becnuli ta có t


Tấ
k−2
k−2 k
p 2 ≥1+ (p − 1) ≥
2 2
nên (*) đúng.
.
n

Do n ≥ kp nên n! .. pk . Bài toán được chứng minh. 


yễ

Bài 1.14. Tìm các số nguyên a,b sao cho



a2 + ab + 1 b2 + ab + a + b − 1.
gu

Lời giải.
Ta có
N

a2 + ab + 1|b2 + ab + a + b − 1 ⇔ a2 + ab + 1|b2 + 2ab + a2 + a + b = (a + b)(a + b + 1).

Nếu a + b = 0 hoặc a + b = −1 thì ta có lời giải cho bài toán.


Nếu a + b = 1 ⇒ a + 1|2 ⇒ a = −3,a = 0,a = −2,a = 1.
Xét a + b ∈
/ {−1,0,1}, khi đó vì

a2 + ab + 1,a + b = (a,a + b) = 1


nên ta có

a2 + ab + 1|a + b + 1 ⇒ a2 + ab + 1|a2 + ab − a − b = (a − 1) (a + b) .

Suy ra a2 + ab + 1|a − 1 (2).


Với a = 1 thì (2) hiển nhiên đúng.
Với |a + b| ≤ 1 thì (2) hiển nhiên đúng.

12
1. PHÉP CHIA HẾT TRONG TẬP SỐ NGUYÊN

a 6= 1
Xét , khi đó |a(a + b) + 1| ≤ |a − 1|.
|a + b| ≥ 2
Ta có
2 |a| − 1 ≤ |a(a + b)| − 1 ≤ |a(a + b) + 1| ≤ |a − 1|
Suy ra a = 0,a = −1,a = −2.
Vậy
(a,b) ∈ {(−k,k); (−k − 1,k); (−3,4); (−2,3); (1,0); (−1,3); (−1,4); (−2,4)}
với k ∈ Z. 
Bài 1.15. Cho n số nguyên dương lẻ. Chứng minh rằng 3n + 1 không chia hết cho n.
Lời giải.
.
Giả sử tồn tại n để 3n + 1 .. n. Gọi p là ước nguyên tố nhỏ nhất của n.
Khi đó
3n ≡ −1(modp) ⇒ 32n ≡ 1 (modp)
Mà 3p−1 ≡ 1 (modp) nên

hu
3(2n,p−1) ≡ 1 (modp)
.
Mà (n; p − 1) = 1 nên (2n; p − 1) = 2 ⇒ 32 − 1 = 8 .. p vô lí do p lẻ. 

T
Bài 1.16. Tìm các số nguyên a,b,c > 1 thỏa ab − 1 chia hết cho c, bc − 1 chia hết cho a và ca − 1
chia hết cho b.
Lời giải. t
Từ giả thiết, ta có a,b,c đôi một nguyên tố cùng nhau.
Từ đó, ta có
Tấ
.
(ab − 1) (bc − 1) (ca − 1) .. abc
Hay
.
ab + bc + ca − 1 .. abc.
n

Vì ab + bc + ca > 1 nên ta có
yễ

1 1 1
ab + bc + ca > abc ⇒ f (a,b,c) = + + > 1. (1)
a b c
gu

Không mất tính tổng quát, ta giả sử a < b < c. Ta có


1 1 1
f (3,4,5) = + + <1
3 4 5
N

1 1 1
nên a = 2. Khi đó + > . Vì
b c 2
1 1 9 1
+ = < ⇒2<b<4⇒b=3
4 5 20 2
1 1 1 1
⇒ > − = ⇒ 3 < c < 6 ⇒ c = 4,5.
c 2 3 6
Thử lại ta có (a,b,c) = (2,3,5) và các bộ hoán vị. 
Bài 1.17. Tìm tất cả các số nguyên dương m,n sao cho
m2 + n n2 + m

n2 − m m2 − n

13
1. PHÉP CHIA HẾT TRONG TẬP SỐ NGUYÊN

là các số nguyên.
Lời giải.
• m = n ta có
m2 + n m2 + m 2
2
= 2
=1+
n −m m −m m−1
nên m = 2,m = 3.
• m > n, khi đó
n2 + m
2
≥ 1 ⇔ n2 + m ≥ m2 − n ⇔ n + 1 ≥ m
m −n
Suy ra m = n + 1, khi đó

m2 + n (n + 1)2 + n 4n + 2
2
= 2
=1+ 2
n −m n −n−1 n −n−1
m2 + n
+) Với n = 1 ta có ∈ Z.
n2 − m

hu
+) Với n ≥ 2, suy ra

4n + 2 5 + 37
≥ 1 ⇔ n2 − 5n − 3 ≤ 0 ⇔ n ≤ .

T
2
n −n−1 2

Nên n = 2,3,4,5. Thử lại ta có n = 2 ⇒ m = 3.


• m < n tương tự như trên ta có được t
 
Tấ
m=2 m=1
và .
n=3 n=2

Vậy các cặp (m; n) cần tìm là:


n

(2; 2), (3; 3), (1; 2), (2; 1), (2; 3), (3; 2)
yễ


gu
N

14
2. HỢP SỐ - SỐ NGUYÊN TỐ

§2. Hợp số - số nguyên tố


I. Định nghĩa
Định nghĩa 1.

• Số nguyên tố là số nguyên dương lớn hơn 1 chỉ chia hết cho 1 và chính nó.

• Số nguyên dương khác 1 không là số nguyên tố được gọi là hợp số.

II. Đinh lí
Định lí 1. Tồn tại vô hạn số nguyên tố.
Chứng minh: Để chứng minh định lí này, ta cần bổ đề sau:

Bổ đề 1. Mọi số nguyên dương lớn hơn 1 đều có ít nhất một ước nguyên tố.

hu
Thật vậy: Giả sử tồn tại số nguyên dương n khác 1 và n không có ước nguyên tố. Trong các số
như vậy ta xét n là số nhỏ nhất. Vì n không là số nguyên tố nên tồn tại các số nguyên dương
a,b sao cho n = ab và 1 < a, b < n. Vì a < n nên a có ít nhất một ước nguyên tố. Rõ ràng ước

T
nguyên tố của a cũng là ước của n. Điều này trái với giả sử ở trên. Vậy bổ đề được chứng minh.
Trở lại bài toán:
Ta có thể chứng minh định lí trên theo hai cách sau:
t
Cách 1: Xét Pn = n! + 1, theo bổ đề trên ta có Pn có ít nhất một ước nguyên tố pn . Dễ thấy
Tấ
pn > n. Vậy mỗi số nguyên dương n ta có ít nhất một số nguyên tố pn > n. Từ đó, suy ra có vô
hạn số nguyên tố.
Cách 2: Gải sử có hữu hạn số nguyên tố p1 < p2 < · · · < pn . Xét số a = p1 p2 · · · pn + 1, ta có
a có một ước nguyên tố q. Dễ thấy q > pn (vì nếu q ≤ pn thì q sẽ trùng với một trong các số
p1 , p2 , . . . , pn , khi đó q là ước của 1).Điều này trái với việc giả sử trên. Vậy định lí được chứng
n

minh.

yễ

Định lí 2. Nếu số nguyên dương n là hợp số, thì n có ước nguyên tố không vượt quá n.
Chứng minh:
√ Vì n là hợp số, nên n = ab với a,b là các số nguyên 1 < a ≤ b < n. Suy ra n ≥ a2
hay a ≤ n. Định lí được chứng minh.
gu

III. Định lí cơ bản của số học


N

Định lí 3. Mọi số nguyên n > 1 đều biểu diễn được dưới dạng tích của các số nguyên tố. Phân
tích này là duy nhất nếu không tính thứ tự của các thừa số.
Chứng minh: Ta chứng minh tồn tại biểu diễn bằng qui nạp.
Với n = 2, n = 3, n = 4 = 2.2, n = 5, n = 6 = 2.3 đều biểu diễn dưới dạng tích các số nguyên tố.
Giả sử khẳng định đúng đến n˘1, tức mọi số nguyên không vượt quá n˘1 đều biểu diễn được
dưới dạng tích các số nguyên tố.
Xét số nguyên n.
Nếu n nguyên tố ta có ngay điều chứng minh.
Nếu n là hợp số thì n = n1 .n2 (1 < n1 , n2 < n), từ giả thiết qui nạp ta có n1 , n2 đều biểu
diễn được dưới dạng tích các số nguyên tố, như vậy n cũng biểu diễn được dưới dạng tích các số
nguyên tố.
Để chứng minh biểu diễn trên là duy nhất, ta dựa vào bổ đề sau:

Bổ đề 2. Nếu p là số nguyên tố và p|ab thì p|a hoặc p|b.

15
2. HỢP SỐ - SỐ NGUYÊN TỐ

Ta chứng minh biểu diễn duy nhất.


Giả sử tồn tại số nguyên dương có ít nhất hai cách biểu diễn khác nhau, trong các số như vậy ta
xét n là số nhỏ nhất. Ta có
n = p1 p2 . . . pr = q 1 q 2 . . . qs
với p1 ≤ p2 ≤ . . . ≤ pr và q1 ≤ q2 ≤ . . . ≤ qs là các số nguyên tố.
Giả sử p1 ≤ q1 . Khi đó p1 |q1 q2 . . . qs , suy ra tồn tại i để p1 |qi hay p1 = qi nên qi là q1 .
n n n
Xét = p2 . . . pr = q2 . . . qs . Do < n nên có một cách phân tích suy nhất. Do đó n cũng
p1 p1 p1
có một cách phân tích duy nhất. Điều này mẫu thuẫn với điều giả sử ở trên.
Vậy định lí được chứng minh.
Như vậy với mọi số nguyên dương n thì ta luôn phân tích được
n = pα1 1 .pα2 2 . . . . .pαk k
trong đó pi , i = 1,k là các số nguyên tố phân biệt và αi là các số nguyên dương.

hu
IV. Ví dụ minh họa

Ví dụ 2.1. Chứng minh rằng với mọi số tự nhiên n > 1 thì n5 + n4 + 1 không phải là số

T
nguyên tố.

Lời giải. t
Ta có:
Tấ
n5 + n4 + 1 = n2 + n + 1 n3 − n + 1 .
 

Mà n2 + n + 1 > 1 và n3 − n + 1 > 1 nên n5 + n4 + 1 là hợp số. 

Ví dụ 2.2. Tìm tất cả các cặp số nguyên dương (a; b) sao cho a4 + 4b4 là số nguyên tố.
n

Lời giải.
yễ

Ta có: 2
a4 + 4b4 = a2 + 2b2 − (2ab)2 = a2 + 2ab + 2b2 a2 − 2ab + 2b2 .
 

Từ đó ta tìm được a = b = 1. 
gu

Ví dụ 2.3. Cho n là số tự nhiên dương. Chưng minh rằng luôn tồn tại n số tự nhiên liên
tiếp sao cho chúng là hợp số.
N

Lời giải.
Đặt a = (n + 1)!. Xét n số a + 2, a + 3, . . . , a + n + 1. Ta thấy n số này đều là hợp số. 

Ví dụ 2.4. Chứng minh rằng: Có vô hạn số nguyên tố dạng 4k + 3.

Lời giải.
Giả sử có hữu hạn số nguyên tố dạng 4k + 3. Gọi các số đó là p1 ,p2 , . . . ,pn .
Xét số nguyên dương m = 4p1 p2 · · · pn + 3, khi đó m có dạng 4k + 3.
Nếu tất cả các ước nguyên tố của m đều có dạng 4k + 1 thì m có dạng 4k + 1 (vô lí). Do đó, m
có ít nhất một ước nguyên tố dạng 4k + 3, kí hiệu số nguyên tố này là p.Dễ thấy p > pn (vô lí).


Ví dụ 2.5. Cho p là số nguyên tố dạng 4k + 3. Chứng minh rằng x2 + y 2 chia hết cho p
khi và chỉ khi x và y chia hết cho p.

Lời giải.

16
2. HỢP SỐ - SỐ NGUYÊN TỐ

• Nếu x hoặc y chia hết cho p thì hiển nhiên số còn lại cũng chia hết cho p.

• Xét x,y cùng không chia hết cho p. Vì p là số nguyên tố nên (x; p) = (y; p) = 1.
Do đó, theo định lí Fecma ta có:

xp−1 ≡ y p−1 ≡ 1 (mod p)

hay
2k+1 2k+1
x4k+2 ≡ y 4k+2 (mod p) ⇔ x2 ≡ y2 (mod p).
.
Suy ra x ≡ y (mod p), do đó x2 + y 2 ≡ 2x2 (mod p) ⇒ 2 .. p vô lí.

Vậy ta có điều phải chứng minh. 


4! Với mọi số nguyên x thì x2 + 1 không có ước dạng 4k + 3.

hu
Ví dụ 2.6. Cho p, q, r là các số nguyên tố và n là số tự nhiên thỏa

pn + q n = r 2

T
Chứng minh n = 1.

Lời giải. t
Giả sử n ≥ 2.
Trong ba số p, q, r có một số chẵn.
Tấ
• r = 2, khi đó pn + q n = 4 điều này không xảy ra.

• p > q = 2, ta có:
pn + 2 n = r 2 .
n

– n lẻ. Suy ra :
yễ

(p + 2)(pn−1 − 2pn−2 + · · · + 2n−1 ) = r2


Vì p + 2 > 1 và pn−1 − 2.pn−2 + · · · + 2n−1 > 2n−1 > 1.
gu

Nên ta có r = p + 2, suy ra pn + 2n = (p + 2)2 = p2 + 4p + 4. Điều này không thể xảy


ra với n ≥ 3.
– n = 2k, ta có: p2k + 22k = r2 . Theo phương trình Pitago ta có:
N

pk = a2 − b2 , 2k = 2ab, r = a2 + b2

với a, b ∈ Z, a > b, (a,b) = 1.


Ta có: b = 1,a = 2k−1 , suy ra pk = 4k−1 − 1 < 4k ⇒ p = 3.
Hay 3k = 4k−1 − 1 phương trình này vô nghiệm.

Do đó ta có n = 1. 
p
Ví dụ 2.7. Cho p là số nguyên tố. Tìm tất cả các số nguyên k sao cho k 2 − pk là số
nguyên dương.

Lời giải.
Ta xét các trường hợp sau:

• p = 2 ta có: k 2 − 2k = (k − 1)2 − 1 không thể là một số chính phương lớn hơn 0.

• p ≥ 3.

17
2. HỢP SỐ - SỐ NGUYÊN TỐ

.
– k .. p, ta có k = np và k 2 − pk = p2 n(n − 1).
Do n(n − 1) > 0 không thể là số chính phương nên trường hợp này không tồn tại k.
– (k; p) = 1, suy ra (k; k − p) = 1. Do đó k(k − p) là số chính phương khi và chỉ khi

k = m2 ,k − p = n2

Suy ra p = m2 − n2 = (m + n)(m − n).


Mà p nguyên tố nên dẫn đến

m−n=1 p+1
⇒m=
m+n=p 2

(p + 1)2
và k = .
4
Thử lại ta thấy thỏa mãn.
(p + 1)2

hu
Vậy k = với p là số nguyên tố lẻ. 
4
p−1
Ví dụ 2.8. Chứng minh rằng với mọi số nguyên tố p thì p3 + không phải là tích

T
2
của hai số tự nhiên liên tiếp.

Lời giải.
t
Tấ
p−1
• Nếu p = 2 suy ra p3 + không nguyên.
2
p−1 p−1
• Nếu p = 4k + 1, suy ra p3 + = (4k + 1)3 + 2k là số lẻ nên p3 + không thể là
2 2
tích của hai số tự nhiên liên tiếp.
n

• Nếu p = 4k + 3. Giả sử
yễ

p−1
p3 + = x(x + 1) ⇔ 2p(2p2 + 1) = (2x + 1)2 + 1
2
gu

với x ∈ N.
.
Suy ra (2x + 1)2 + 1 .. p vô lí vì p = 4k + 3.
N

Từ các trường hợp trên, ta có điều phải chứng minh. 

Ví dụ 2.9. Nếu p là số nguyên tố có dạng 4k + 1 thì tồn tại hai số nguyên dương c, d sao
cho
p = c2 + d2 .

Lời giải.
Trước hết ta có bổ đề sau:
Bổ đề 3. Nếu p ≡ 1 (mod 4) là số nguyên tố thì
  2
p−1
! ≡ −1 (mod p).
2

18
2. HỢP SỐ - SỐ NGUYÊN TỐ

Thật vậy: Theo định lí Winsol thì

(p − 1)! ≡ −1 (mod p).


p − k ≡ (−1) .k (mod p)
nên
    p − 1   2
p−1 p−1 p−1
(p − 1)! = 1.2 . . . . p− . . . (p − 1) ≡ (−1) 2 ! ≡ −1 (mod p).
2 2 2

Chứng minh định lí:  


p−1
Vì p ≡ 1 (mod 4) nên tồn tại x sao cho x2 ≡ −1 (mod p) (Chẳng hạn x = ! ).
2
Đặt

t = [ p] ⇒ (t + 1)2 > p.

hu
Xét tập S = {a + xb |0 ≤ a, b ≤ t}, ta có |S| = (t + 1)2 > p .
Suy ra trong S có ít nhất một cặp đồng dư với nhau theo (mod p).
Hay

T
a + bx ≡ a1 + b1 x (mod p) ⇒ (a − a1 )2 ≡ (b − b1 )2 x2 ≡ − (b − b1 )2 (mod p)

Suy ra
t
(a − a1 )2 + (b − b1 )2 ≡ 0 (mod p).
Tấ
Vì 0 ≤ a, a1 , b, b1 ≤ t nên ta có

(a − a1 )2 + (b − b1 )2 = p.
n

Đặt c = |a − a1 | ,d = |b − b1 | ta có p = a2 + b2 .

yễ
gu
N

19
2. HỢP SỐ - SỐ NGUYÊN TỐ

V. Bài tập
Bài 2.1. Tìm tất cả các số nguyên tố p sao cho:

a) p + 10 và p + 14 cũng là số nguyên tố.

b) p + 2, p + 6, p + 8, p + 12, p + 14 đều là số nguyên tố.

Lời giải.

a) Ta có p = 2 không thỏa và p = 3 thỏa bài toán. Xét p ≥ 5, ta có các trường hợp sau

• p = 3k + 1, khi đó p + 14 = 3k + 15 = 3(k + 5) là hợp số.


• p = 3k + 2, khi đó p + 10 = 3(k + 4) là hợp số.

Vậy p = 3 là số cần tìm.

hu
b) Ta có p là số lẻ và p = 3 không thỏa, p = 5 thỏa bài toán. Xét p > 5. Xét p = 5k + r ta
thấy trường hợp này không thỏa. Vậy p = 5 là số cần tìm.

T
Bài 2.2. Cho số nguyên tố p sao cho 8p2 + 1 là số nguyên tố. Chứng minh rằng 8p2 − 1 cũng là
số nguyên tố.
Lời giải.
Ta xét các trường hợp sau
t
Tấ
.
• p = 2 ⇒ 8p2 + 1 = 33 .. 3

• p = 3 ⇒ 8p2 + 1 = 73 là số nguyên tố
n

.
• p ≥ 5, khi đó p2 = 3k + 1 ⇒ 8p2 + 1 = 3 (8k + 3) .. 3 nên 8p2 + 1 không là số nguyên tố.
yễ

Do đó, ta chỉ có p = 3 ⇒ 8p2 − 1 = 71 là số nguyên tố. 


Bài 2.3. Cho số nguyên dương n thỏa mãn 2n − 1 là số nguyên tố. Chứng minh rằng n là số
gu

nguyên tố.
Lời giải.
Giả sử n là hợp số, ta có n = ab với 2 ≤ a ≤ b < n. Khi đó
N

2n − 1 = 2ab − 1 = (2a − 1)(2a(b−1) + 2a(b−2) + · · · + 1)

là hợp số. Điều này trái với giả thiết.


Vậy n là số nguyên tố. 
Bài 2.4. Chứng minh rằng có vô hạn số nguyên tố dạng 3k − 1.
Lời giải.
Giả sử có hữu hạn số nguyên tố p1 , p2 , . . . , pn dạng 3k + 1. Xét a = 3p1 p2 . . . pn − 1,khi đó ta có
các khả năng sau

• a là số nguyên tố, khi đó a có dạng 3k − 1 và a > pn (vô lí).

• a là hợp số, khi đó a có ít nhất một ước nguyên tố pn+1 có dạng 3k − 1. Vì ngược lại, nếu
mọi ước của a đều có dạng 3k + 1 thì a có dạng 3k + 1. Dễ thấy pn+1 > pn điều này cũng
vô lí.

Vậy có vô hạn số nguyên tố dạng 3k − 1. 

20
2. HỢP SỐ - SỐ NGUYÊN TỐ

Bài 2.5. Tìm số nguyên tố p để 2p + p2 cũng là số nguyên tố.


Lời giải.
Ta xét các trường hợp sau

• p = 2, khi đó 2p + p2 = 8 là hợp số.

• p = 3, khi đó 2p + p2 = 17 là số nguyên tố.


.
• p > 3, khi đó 2p + p2 = (2p + 1) + (p2 − 1). Vì p lẻ và không chia hết cho 3 nên 2p + 1 .. 3
. .
và p2 − 1 .. 3. Suy ra 2p + p2 .. 3 nên 2p + p2 là hợp số.

Vậy p = 3 là số cần tìm. 


3m2 +6n−61
Bài 2.6. Tìm các số tự nhiên m, n sao cho x = 3 + 4 là số nguyên tố.
Lời giải.
Ta có 3m2 + 6n − 61 = 3k + 2 .
.
Nếu k ≥ 1 ta có x = 33k+2 + 4 = 9 · 27k + 4 .. 13.

hu
Suy ra k = 0 hay 3m2 + 6n − 61 = 2 ⇔ m2 + 2n − 21 = 0.
Vì m2 lẻ và m2 < 21 nên m2 = 1, m2 = 9.

• m = 1 ⇒ n = 10.

T
• m = 3 ⇒ n = 6.


t
Tấ
Bài 2.7. Tìm các số nguyên tố a, b, c sao cho

ab + bc + ca > abc.
n

Lời giải.
Giả sử a ≤ b ≤ c.
yễ

Ta có abc < ab + bc + ca < 3bc ⇒ a < 3 ⇒ a = 2.


Khi đó
2bc < 2(b + c) + bc ⇒ bc < 2(b + c) ≤ 4c ⇒ b ≤ 3 ⇒ b ∈ {2,3} .
gu

• b = 2 ⇒ 2c < 2(2 + c) đúng với mọi c ≥ 2, c nguyên tố.

• b = 3 ⇒ 3c < 2(3 + c) ⇒ c < 6 ⇒ c ∈ {3,5}.


N


Bài 2.8. Tìm tất cả các số tự nhiên a, b, c sao cho

a3 + b3 + c3 − 3abc

là số nguyên tố.
Lời giải.
Không mất tính tổng quát, ta giả sử a ≥ b ≥ c.
Ta có
a3 + b3 + c3 − 3abc = (a + b + c) a2 + b2 + c2 − ab − bc − ca .


Do đó a3 + b3 + c3 − 3abc là số nguyên tố khi xảy ra một trong các trường hợp sau:

• a + b + c = 1 và a2 + b2 + c2 − ab − bc − ca là số nguyên tố.
Từ a + b + c = 1 ⇒ a = 1, b = c = 0, khi đó a3 + b3 + c3 − 3abc = 1 không là số nguyên tố.

21
2. HỢP SỐ - SỐ NGUYÊN TỐ

• a2 + b2 + c2 − ab − bc − ca = 1 và a + b + c là số nguyên tố.
Ta có
a2 + b2 + c2 − ab − bc − ca = 1 ⇔ (a − b)2 + (b − c)2 + (c − a)2 = 2
Suy ra 
a=b 
a=b

b−c=1 ⇔ ⇒ a + b + c = 3b − 1
c=b−1
c − a = −1

Hoặc 
a − b = 1
b − c = 0 ⇔ b = c = a − 1.
a−c=1

Vậy các số tự nhiên cần tìm là (k; k, k − 1) với 3k − 1 là số nguyên tố.


Hoặc (k; k − 1; k − 1) với 3k − 2 là số nguyên tố.


hu
Bài 2.9. Cho a, b, c là các số nguyên khác 0, a 6= c thỏa :
a a2 + b 2
= 2 .
c c + b2

T
Chứng minh rằng a2 + b2 + c2 không phải là số nguyên tố.
Lời giải.
a 2 +b2
t
Đẳng thức = ab2 +c 2
2 tương đương (a − c)(b − ac) = 0.
c
Tấ
Vì a 6= c nên suy ra b2 = ac.
Khi đó:

a2 + b2 + c2 = a2 + ac + c2 = a2 + 2ac + c2 − b2 = (a + c)2 − b2 = (a + c − b)(a + c + b)


n

Dễ thấy a2 + b2 + c2 > 3. Do đó nếu a2 + b2 + c2 là số nguyên tố thì có 4 trường hợp có thể xảy


ra.
yễ

(1) a + c − b = 1 và a + b + c = a2 + b2 + c2 .

(2) a + c + b = 1 và a + c − b = a2 + b2 + c2 .
gu

(3) a + c − b = −1 và a + b + c = −(a2 + b2 + c2 ).

(4) a + c + b = −1 và a + b + c = −(a2 + b2 + c2 ).
N

Ở hai trường hợp đầu ta có


a2 + b2 + c2 − 2(a + c) + 1 = 0
hay
(a − 1)2 + (c − 1)2 + b2 = 1 ⇒ a = c = 1
(trái với giả thuyết) Ở hai trường hợp sau ta có (a + 1)2 + (c + 1)2 + b2 = 1, suy ra a = c = −1
(trái với giả thuyết). 
Bài 2.10. Tìm ba số nguyên tố p, q, r thỏa

pq + q p = r.

Lời giải.
Ta có r > 2 ⇒ r lẻ. Do đó p,q phải khác tính chẵn, lẻ nên trong hai số phải có một số bằng 2.
Giả sử p = 2, khi đó 2q + q 2 = r.

22
2. HỢP SỐ - SỐ NGUYÊN TỐ

• q = 3 ⇒ r = 17 thỏa.

• q > 3. Vì q lẻ nên q 2 = 3k + 1 và 2q = 3l − 1. Suy ra 2q + q 2 = 3 (k + l) là hợp số.

Vậy (3; 2; 17) và (2; 3; 17) là hai bộ cần tìm.



Bài 2.11. Chứng minh rằng nếu 1 + 2n + 4n là số nguyên tố thì n = 3k .
Lời giải.
k
Giả sử n = 3k .m, (m,3) = 1,m ≥ 1 và đặt a = 23 , ta có

1 + 2n + 4n = 1 + am + a2m .

• m = 3q + 1, ta có

1 + am + a2m = 1 + a.a3q + a2 .a6q = a2 a6q − 1 + a a3q − 1 + a2 + a + 1


 

Vì a3 − 1 = (a − 1) (a2 + a + 1) ⇒ 1 + am + a2m là hợp số.

hu
• m = 3q + 2, ta có
1 + am + a2m = 1 + a2 .a3q + a4 .a6q

T
= a4 a6q − 1 + a2 a3q − 1 + a4 + a2 + 1
 


a3 − 1 = (a − 1) a2 + a + 1
t 
Tấ

a4 + a2 + 1 = a2 + a + 1 a2 − a + 1
 

Nên 1 + am + a2m là hợp số.

Do vậy n = 3k . 
n

Bài 2.12. Cho p là số nguyên tố lẻ và hai số nguyên a,b thỏa p4 là ước của hai số a2 + b2 và
yễ

a (a + b)2 . Chứng minh rằng p4 là ước của a(a + b).


Lời giải.
Ta có
gu

. .
a (a + b)2 − a a2 + b2 = 2a2 b .. p4 ⇒ a2 b .. p4


.
Nếu ap thì b .. p4 ⇒ a2 + b2 p4 (vô lí).
N

Suy ra
. .
a .. pm ⇒ a (a + b)2 = pm (pm + b)2 .. p4 ⇒ m ≥ 2
.
hay a .. p2 .
. . .
Từ a2 + b2 .. p4 ⇒ b2 .. p4 ⇒ b .. p2 .
.
Từ đó, suy ra a (a + b) .. p4 .

Bài 2.13. Cho p, q là các số nguyên tố và phương trình x2 − px + q = 0 có nghiệm nguyên
dương. Tìm p và q.
Lời giải.
Gọi x0 là nghiệm nguyên dương của phương trình x2 − px + q = 0, ta có x0 |q nên x0 = 1 hoặc
x0 = q.

• x0 = 1 ta có 1 − p + q = 0 ⇔ p = q + 1, suy ra q = 2,p = 3.

• x0 = q ta có q 2 − pq + q = 0 ⇔ p = q + 1, suy ra q = 2,p = 3.

23
2. HỢP SỐ - SỐ NGUYÊN TỐ


Bài 2.14. Chứng minh rằng nếu m có ít nhất một ước nguyên tố khác 3 thì 1 + 2m + 4m là hợp
số.
Lời giải.
Ta xét các trường hợp sau

• m = 2k, ta có

1 + 2m + 4m = 1 + 22k + 24k = (22k + 1)2 − 22k = (22k + 2k + 1)(22k − 2k + 1)

là hợp số.

• m lẻ, ta có m = 3n .k với (k,3) = 1 và k ≥ 5. Khi đó

1 + 2m + 4m = 1 + ak + a2k
n
với a = 23 .

hu
+)k = 3t + 1, khi đó

a2k + ak + 1 = a2 (a6t − 1) + a(a3t − 1) + a2 + a + 1.

T
. . .
Vì a3t − 1 .. a3 − 1 .. a2 + a + 1 nên a2k + ak + 1 .. a2 + a + 1.
+) k = 3t + 1, khi đó
t
a2k + ak + 1 = a4 (a6t − 1) + a2 (a3t − 1) + a4 + a2 + 1.
Tấ
. . . .
Vì a6t − 1 .. a2 + a + 1, a3t − 1 .. a2 + a + 1 và a4 + a2 + 1 .. a2 + a + 1 nên a2k + ak + 1 .. a2 + a + 1

Do vậy ta luôn có 1 + 2m + 4m là hợp số. 


 
n

4 4 ..
Bài 2.15. Cho T = (x,y) | x,y ∈ N,0 ≤ 2x < y ≤ 100,x + y . 49 . Tìm |T |.
yễ

Lời giải. 
x2 ... 7
4 .. 4 .. .
4 4
Ta có x + y . 49 nên suy ra x + y . 7 hay nên x,y .. 7.
y 2 ... 7
gu

Do vậy x = 7x1 ,y = 7y1 , suy ra 0 ≤ 2x1 < y1 ≤ 14. Đặt x1 = a, 0 ≤ a ≤ 6. Với mỗi a ta có
14 − 2a giá trị của y1 nên số cặp (x1 ,y1 ) thỏa bài toán là
N

6
X
(14 − 2a) = 2 + 4 + 6 + . . . + 14 = 56.
a=0


Bài 2.16. Chứng minh rằng phương trình

x2 − 7 = y 3

không có nghiệm nguyên.


Lời giải.
Từ phương trình ta suy ra x,y khác tính chẵn, lẻ.
Phương trình đã cho trở thành

x2 + 1 = y 3 + 8 = (y + 2)(y 2 − 2y + 4). (1)

Ta xét các trường hợp sau

24
2. HỢP SỐ - SỐ NGUYÊN TỐ

. .
• y = 2k thì y 2 − 2y + 4 .. 4, nhưng x lẻ nên x2 + 1 6 .. 4. nên (1) vô nghiệm.

• y = 4k + 1, khi đó y + 2 = 4k + 2 là ước của x2 + 1 (vô lí). Nên trường hợp này (1) cũng
vô nghiệm.

• y = 4k + 3, khi đó y 2 − 2y + 4 ≡ 9 − 2.6 ≡ 3 (mod 4) và là y 2 − 2y + 4 là ước của x2 + 1


(vô lí).

Vậy phương trình đã cho vô nghiệm. 


Bài 2.17. Tìm các số nguyên tố a,b,c và số nguyên dương k sao cho

a2 + b2 + 16c2 = 9k 2 + 1.

Lời giải.
Từ phương trình ta suy ra a2 + b2 + c2 ≡ 1 (mod 3). Suy ra,trong ba số a,b,c có hai số chia hết
cho 3.

hu
• a = b = 3, ta có
18 + 16c2 = 9k 2 + 1 ⇔ (3k − 4c)(3k + 4c) = 17

T
Suy ra ( (
3k + 4c = 17 k=3
⇔ .
3k − 4c = 1
t c=2
Tấ
• c = 3, không mất tính tổng quát, ta giả sử a = 3. Khi đó, ta có

(3k − b)(3k + b) = 152 = 19.8


(
3k + b = 19
n

+) vô nghiệm.
3k − b = 8
yễ

( (
3k + b = 38 k=7
+) ⇔ .
3k − b = 4 b = 17
( (
gu

3k + b = 76 k = 13
+) ⇔ .
3k − b = 2 b = 37
(
3k + b = 152
N

+) vô nghiệm.
3k − b = 2


Bài 2.18. Chứng minh rằng không tồn tại số tự nhiên n để n7 + 7 là số chính phương.
Lời giải.

Bài 2.19 (Bulgaria MO 2014). Tìm các số nguyên tố p và q sao cho

p2 | q 3 + 1 và q 2 | p6 − 1.

Lời giải.
Nếu p = 3, ta có q 2 | 36 − 1 = 23 · 7 · 11 nên q = 2.
Xét p 6= 3, ta có p2 | (q + 1)(q 2 − q + 1).
Mà (q + 1,q 2 − q + 1) = (q + 1,3) = 1 hoặc 3. Suy ra hoặc p2 | q + 1 hoặc p2 | q 2 − q + 1.Từ đây,
suy ra p < q.

25
2. HỢP SỐ - SỐ NGUYÊN TỐ

Nếu q = p + 1 ta có p = 2,q = 3.
Xét q ≥ p + 2. Vì q 2 | (p − 1)(p + 1)(p2 + p + 1)(p2 − p + 1).
Do (q,p + 1) = (q,p − 1) = 1 và (p2 − p + 1,p2 + p + 1) = (p2 + p + 1,2p) = 1 nên ta có hoặc
q 2 | p2 + p + 1 hoặc q 2 | p2 − p + 1.
Mà q ≥ p + 2 nên q 2 ≥ (p + 2)2 > p2 + p + 1 > p2 − p + 1. Suy ra q 2 6 | p6 − 1.
Vậy (p,q) = (2,3); (3,2). 

hu
T
t
Tấ
n
yễ
gu
N

26
3. ƯỚC CHUNG LỚN NHẤT - BỘI CHUNG NHỎ NHẤT

§3. Ước chung lớn nhất - Bội chung nhỏ nhất


I. Ước chung lớn nhất
Định nghĩa 1. Ước chung lớn nhất của hai số nguyên a và b không đồng thời bằng 0 là số
nguyên lớn nhất chia hết a và b. Ta kí hiệu (a,b) hoặc gcd(a,b).
Nếu (a,b) = 1thì ta gọi a và b là hai số nguyên tố cùng nhau.
Định nghĩa 2. Giả sử a1 ,a2 , . . . ,an là các số nguyên không đồng thời bằng 0. Ước chung lớn
nhất của các số đó là số nguyên lớn nhất mà là ước của các số đã cho. Ta kí hiệu (a1 ,a2 , . . . ,an )
hoăc gcd(a1 ,a2 , . . . ,an )
Nếu (a1 ,a2 , . . . ,an ) = 1 ta nói các số đó nguyên tố cùng nhau.
Các số đã cho được gọi là đôi một nguyên tố cùng nhau nếu (ai ,aj ) = 1 với mọi i 6= j.
Nhận xét. Từ định nghĩa ta có các nhận xét sau:
 
a b
a) Nếu d = (a, b) thì , = 1.

hu
d d
b) (a + bc, b) = (a, b).

T
c) (a1 , a2 , . . . , an ) = (a1 , a2 , . . . , an−2 , (an−1 ,an ))
. .
d) Nếu ab .. c và (b, c) = 1 thì a .. c.
t
Định lí 1. Cho d = (a, b). Khi đó d là số nguyên dương nhỏ nhất biểu diễn được một tổ hợp
Tấ
tuyến tính của a và b. Tức là tồn tại hai số nguyên x, y để

d = ax + by.

Chứng minh: Đặt A = {ax + by > 0|x, y ∈ Z, ta có A 6= ∅ vì (−1)a + 0b ∈ A và 1.a + 0.b ∈ A.


n

Do đó A sẽ có phần tử nhỏ nhất, gọi d là phần tử nhỏ nhất đó.


Ta chứng minh d là ước của a và b.
yễ

Giả sử a = dq + r, ta có r = a − dq = a − q(ax + by) = (1 − qx)a − qby, suy ra r cũng biểu diễn


được qua tổ hợp tuyến tính của a và b. Mà 0 ≤ r < d nên ta có r = 0, hay d|a. Chứng minh
tương tự, ta cũng có d|b.
gu

Vì d = ax + by nên ta thấy d chia hết cho tất cả các ước chung của a và b.
Định lí 2 (Thuật toán Ơ-clít). Giả sử r0 = a,r1 = b là các số nguyên không âm, b 6= 0. Ta
áp dụng liên tiếp thuật toán chia
N

ri = ri+1 qi+1 + ri+2


với 0 < ri+2 < ri+1 với i = 0,1, . . . ,n − 2, rn = 0. Khi đó (a,b) = rn−1 .
Để chứng minh định lí này ta dựa vào nhận xét trên và thuật toán chia.

II. Bội chung nhỏ nhất


Định nghĩa 3. Bội chung nhỏ nhất của hai số nguyên a và b khác 0 là số nguyên dương nhỏ
nhất chia hết cho cả a và b. Kí hiệu [a,b] hoặc lcm(a,b).
Tương tự ta có thể định nghĩa bội chung nhỏ nhất cho n số nguyên a1 ,a2 , . . . , an khác 0 là số
nguyên dương nhỏ nhất chia hết cho tất cả n số đó. Kí hiệu [a1 ,a2 , . . . ,an ].
Nhận xét.
1) Cho các số nguyên a, b, c, k khác 0. Ta có
• [ka, kb] = k[a, b]

• [a, b, c] = [[a, b], c]

27
3. ƯỚC CHUNG LỚN NHẤT - BỘI CHUNG NHỎ NHẤT

• [a, b] · (a, b) = ab
. . .
• Nếu a .. b và a .. c thì a .. [b, c].

2) Giả sử
a = pα1 1 .pα2 2 . . . . .pαk k và b = pβ1 1 .pβ2 2 . . . . .pβkk .
với αi và βi là các số tự nhiên, pi là các số nguyên tố phân biệt. Khi đó
min{α1 ,β1 } min{α2 ,β2 } min{αk ,βk }
• (a,b) = p1 .p2 . . . . .pk
max{α1 ,β1 } max{α2 ,β2 } max{αk ,βk }
• [a,b] = p1 .p2 . . . . .pk
.
• a .. b ⇔ αi ≥ βi với mọi i = 1,k

• a có (α1 + 1)(α2 + 1) . . . (αk + 1) ước nguyên dương khác nhau.

hu
III. Ví dụ minh họa

Ví dụ 3.1. Chứng minh rằng (a2 , b2 ) = (a, b)2 .

T
Lời giải.  
a b
Đặt (a, b) = d, khi đó ,
= 1. t
d d
 2 2 2
Tấ
a b
Suy ra , = 1 nên (a2 ,b2 ) = d2 = (a,b)2 
d2 d2

Ví dụ 3.2. Tìm (a + b,a2 + b2 với a,b là hai số nguyên tố cùng nhau.


n

Lời giải.
yễ

Ta có
(a + b,a2 + b2 ) = (a + b,(a + b)2 − 2ab) = (a + b,2ab) = (a + b,2)
do (a + b,ab) = (a + b,a(a + b) − a2 ) = (a + b,a2 ) = (b,a2 ) = 1.
gu

Suy ra (a + b,a2 + b2 ) = 1 hoặc 2. 

Ví dụ 3.3. Cho hai số nguyên a,b nguyên tố cùng nhau. Tìm (a + b,a2 − ab + b2 ).
N

Lời giải.
Đặt d = (a + b, a2 − ab + b2 ), khi đó d là ước của

(a + b)2 − a2 + ab − b2 = 3ab.

Suy ra 
d | 3b (a + b) − 3ab = 3b2 2 2
 2 2

⇒ d 3a , 3b = 3 a , b =3.
d | 3a (a + b) − 3ab = 3a2
Suy ra d = 1, d = 3. 

Ví dụ 3.4. Cho a, a 6= 1 và m, n là các số nguyên dương. Chứng minh rằng

(am − 1, an − 1) = a(m, n)
− 1.

Lời giải.

28
3. ƯỚC CHUNG LỚN NHẤT - BỘI CHUNG NHỎ NHẤT

Đặt d = (m, n) ⇒ m = d · s, n = d · p.
Khi đó
.
am − 1 = ad·s − 1 .. ad − 1

.
an − 1 = ad·p − 1 .. ad − 1
Do đó
ad − 1 |(am − 1, an − 1) (1).
Theo định lí Bơzu, ta có mx + ny = d với x, y ∈ Z.
Vì d ≤ m, d ≤ n nên x, y không thể cùng là số nguyên dương. Ta giả sử x > 0, y ≤ 0.
Đặt t = (am − 1, an − 1). Suy ra

t |amx − 1 , t a−ny − 1 ⇒ t amx − 1 − ad a−ny − 1 = ad − 1



(2).

Từ (1) và (2) ta có đpcm. 

hu
Ví dụ 3.5. Chứng minh rằng nếu m,n là các số tự nhiên và m lẻ thì

(2m − 1,2n + 1) = 1.

Lời giải.

T
Đặt
t
Tấ
d = (2m − 1,2n + 1) ⇒ 2m − 1 = ad, 2n + 1 = bd
Suy ra
2mn = (ad + 1)n = kd + 1, 2mn = (bd − 1)m = td − 1
Do đó:
n

.
kd + 1 = td − 1 ⇔ d (k − t) = 2 ⇒ 2 .. d.
yễ

Mà d lẻ nên ta có d = 1. 

Ví dụ 3.6. Chứng minh rằng


gu

a · (a, b, c) ≥ (a, b) · (a, c).


N

Lời giải.
Đặt (a, b, c) = d.
Ta có a = da1 , b = db1 , c = cd1 với (a1 , b1 , c1 ) = 1.
Khi đó
(a, b) · (a, c) d (a1 , b1 ) . (a1 , c1 )
= = d (a1 , b1 ) · (a1 , c1 ) .
(a, b, c) (a1 , b1 , c1 )
Vì (a1 , b1 ) và (a1 , c1 ) nguyên tố cùng nhau và cùng là ước của a1 nên tích của chúng là ước của
a1 . Do đó
d (a1 , b1 ) · (a1 , c1 ) ≤ da1 = a.

n(n + 1)
Ví dụ 3.7. Cho n là số nguyên dương và đặt an = . Chứng minh rằng trong dãy
2
các số a1 , a2 , . . . , an , . . . tồn tại vô hạn các số đôi một nguyên tố cùng nhau.

Lời giải.

29
3. ƯỚC CHUNG LỚN NHẤT - BỘI CHUNG NHỎ NHẤT

Ta có a1 = 1, a2 = 3, a4 = 10 là các số đôi một nguyên tố cùng nhau.


Giả sử ta đã tìm được m số t1 , t2 , . . . , tm đôi một nguyên tố cùng nhau.
Đặt t = t1 t2 · · · tm . Ta xét a2t+1 = (t + 1)(2t + 1).

(2t + 1; t) = (t + 1; t) = 1
nên
(a2t+1 ; t) = 1 ⇒ (a2t+1 ; ti ) = 1
với i = 1,2, . . . ,m. Và a2t+1 > t, suy ra dãy t1 , t2 , . . . , tm , a2t+1 gồm các số đôi một nguyên tố
cùng nhau. 

a+1 b+1
Ví dụ 3.8. Cho các số nguyên dương a,bthỏa + là số nguyên, đặt d = (a; b).
√ b a
Chứng minh rằng d ≤ a + b.

Lời giải.
Ta có a = md,b = nc, khi đó

hu
a+1 b+1 md + 1 nd + 1 (m2 + n2 )d + m + n
+ = + =
b a nd md mnd

T
là số nguyên. Suy ra
. √
m + n .. d ⇒ d ≤ m + n ⇒ d ≤ d(m + n) = a + b.
p
t 
Tấ
Ví dụ 3.9. Tìm các số nguyên dương a, b, c sao cho a3 + b3 + c3 chia hết cho a2 b, b2 c và
c2 a.

Lời giải.
n

3 3 3 .. 3 ..
Gọi d = (a; b), suy ra
a +b + c . d ⇒ c . d hay d = (a, b, c).
yễ

a b c
Xét bộ (x; y; z) = ; ; ⇒ x, y, z đôi một nguyên tố cùng nhau và bộ (x; y; z) thỏa
d d d
bài toán.
gu


. .
x3 + y 3 + z 3 .. x2 , y 2 , z 2 ⇒ x3 + y 3 + z 3 .. x2 y 2 z 2
Giả sử x ≥ y ≥ z. Khi đó
N

y2z2
3x3 ≥ x3 + y 3 + z 3 ≥ x2 y 2 z 2 ⇒ y 2 z 2 ≤ 3x ⇒ x ≥ .
3
Mặt khác
. y4z4
y 3 + z 3 .. x2 ⇒ 2y 3 ≥ y 3 + z 3 ≥ x2 ≥ ⇒ yz 4 ≤ 18.
9
• z = 2 ⇒ 16y ≤ 18 ⇒ y = 1 vô lí.
• z = 1, ta thấy x = y = 1 là một bộ thỏa bài toán.
Xét y ≥ 2, khi đó
y2
2x3 ≥ x3 + y 3 + 1 ≥ x2 y 2 ⇒ x ≥
2

. y4
y 3 + 1 .. x2 ⇒ y 3 + 1 ≥ x2 ≥ ⇒ y ≤ 4.
4
Kiểm tra ta thấy chỉ có x = 3,y = 2 thỏa bài toán.

30
3. ƯỚC CHUNG LỚN NHẤT - BỘI CHUNG NHỎ NHẤT

Vậy (a, b, c) ∈ {(k; k; k) , (3k, 2k, k)}. 

Ví dụ 3.10. Cho n + 1 số nguyên dương a0 < a1 < a2 < . . . < an . Chứng minh rằng
1 1 1 1
+ + ... + ≤ 1 − n.
[a0 ,a1 ] [a1 ,a2 ] [an−1 ,an ] 2

Lời giải.
Ta chứng minh bài toán bằng phương pháp quy nạp.

• Với n = 1 thì bài toán luôn đúng.

• Giả sử bài toán đúng đến n, ta chứng minh bài toán đúng với n + 1. Thật vậy:

– Nếu an+1 ≥ 2n+1 thì [an ,an+1 ] ≥ 2n+1 nên


n n−1
X 1 X 1 1 1 1 1
= + ≤ 1 − n + n+1 = 1 − n+1 .

hu
k=0
[ak ,ak+1 ] k=0 [ak ,ak+1 ] [an ,an+1 ] 2 2 2

– Nếu an+1 < 2n+1 thì áp dụng tính chất (a,b).[a,b] = ab và (a,b) ≤ a − b với a > b ta có

T
1 (ak ,ak+1 ) ak+1 − ak 1 1
= ≤ = −
[ak ,ak+1 ] ak .ak+1
t ak ak+1 ak ak+1

Nên suy ra
Tấ
n n  
X 1 X 1 1 1 1 1
≤ − = − < 1 − n+1 .
k=0
[ak ,ak+1 ] k=0
ak ak+1 a0 an+1 2
n

Bài toán được chứng minh. 


yễ

IV. Bài tập


Bài 3.1. Cho (a, b) = 1. Tìm
gu

a) (3a + 5b, 8a + 13b) c) (18a + 5b, 11a + 3b)

d) (a2 + b2 , a3 + b3 ).
N

b) (a + b, a − b)

Lời giải.
1) Ta có (3a + 5b, 8a + 13b) = (3a + 5b, a + 2b) = (b, a + 2b) = (b, a) = 1.
2) Ta có (a + b, a − b) = (a + b, 2b) = 2 hoặc 1.
3) Ta có (18a + 5b, 11a + 3b) = (4a + b, 11a + 3b) = (4a + b, a) = (b, a) = 1.
4) Ta có (a + b)(a2 + b2 ) = a3 + b3 + a2 b + b2 a nên:

d = (a2 + b2 , a3 + b3 ) = (a2 + b2 , ab2 + a2 b) = (a + b)2 − 2ab, ab(a + b)




Vì (ab,a + b) = (a2 , a + b) = (a2 , b) = 1 nên d | ab hoặc d | a + b.


+) d | ab, suy ra d | a hoặc d | b. Mà d | (a + b)2 nên d = 1.
+) d | a + b, suy ra d | 2ab nên d = 1 hoặc d = 2. 
Bài 3.2. Chứng minh rằng các phân số sau là tối giản

31
3. ƯỚC CHUNG LỚN NHẤT - BỘI CHUNG NHỎ NHẤT

21n + 4 3n + 2
a) b) .
14n + 3 8n + 5
Lời giải.
1) Ta có 3(14n + 3) − 2(21n + 4) = 1 nên nếu d = (14n + 3, 21n + 4) thì d|1 hay d = 1.
21n + 4
Vậy là phân số tối giản.
14n + 3
3n + 2
2) Ta có 8(3n + 2) − 3(8n + 5) = 1 nên là phân số tối giản. 
8n + 5
Bài 3.3. Tìm các số nguyên dương a,b biết

a) a + b = 128 và (a, b) = 16 b) (a,b) = 12 và [a, b] = 432.

Lời giải.
1) Ta có a = 16a1 , b = 16b1 , (a1 ,b1 ) = 1 và a1 + b1 = 8. Do đó, ta tìm được các cặp (a, b) là
(16, 112), (48, 80), (112, 16), (80, 48).
2) Ta có a = 12a1 , b = 12b1 , (a1 ,b1 ) = 1 và ab = (a,b) · [a,b] = 12 · 432. Suy ra a1 b1 = 36 = 1 · 36 =

hu
4 · 9.
Từ đó, ta tìm được (a,b) = (12,432), (48,108), (432,12), (108,48). 
Bài 3.4. Chứng minh rằng dãy an = 2n − 3 với n ≥ 3 chứa vô hạn cặp số nguyên tố cùng nhau.

T
Lời giải.
Ta có
a2n = 22n − 3 = (2n − 3) (2n + 3) + 6 = (2n + 3) an + 6.
t
Suy ra
Tấ
(a2n ,an ) = (an ,6) = 1.

Bài 3.5. Chứng minh rằng dãy Mersen Mn = 2n − 1, n ∈ N∗ chứa vô hạn số hạng đôi một
nguyên tố cùng nhau.
n

Lời giải.
yễ


n
Bài 3.6. Chứng minh rằng dãy Fermat Fn = 22 +1 là dãy có các số hạng nguyên tố cùng nhau.
gu

Bài 3.7. (Anbania TST 2012 Tìm các cặp số tự nhiên (a; b) không nguyên tố cùng nhau thỏa
mãn
(a,b) + 9[a,b] + 9(a + b) = 7ab.
N

Lời giải.
Đặt d = (a,b) ta có a = dx,b = dy với (x,y) = 1, d > 1. Ta có [a,b] = dxy nên

1 + 9xy + 9(x + y) = 7dxy

Suy ra
7dxy ≤ xy + 9xy + 9(xy + xy) = 28xy ⇒ 2 ≤ d ≤ 4.

• d = 2 ta có
9(x + y) + 1 = 5xy ⇔ x = 2,y = 19.
Suy ra (a,b) = (4,38).

• d = 3 ta có
1 + 9(x + y) = 12xy
tường hợp này vô nghiệm.

32
3. ƯỚC CHUNG LỚN NHẤT - BỘI CHUNG NHỎ NHẤT

• d = 4 ta có x = y = 1 hay a = b = 4.

Vậy (a,b) = (4,38), (38,4), (4,4). 


Bài 3.8. (Germany 2008) Cho a,b,c là các số nguyên dương thỏa mãn a + b|ab và a + c|ac.
Chứng minh rằng (a,b,c) > 1
Bài 3.9. Tìm tất cả các bộ 3 số nguyên dương {m,n,l} thỏa
 2
m + n = (m,n)
m + l = (m,l)2
n + l = (n,l)2

Bài 3.10. (Baltic Way 2014) Cho m,n là các số nguyên dương và nguyên tố cùng nhau. Xác
định các giá trị có thể có của
2 2
(2m − 2n , 2m +mn+n − 1).

hu
Bài 3.11. (1998 Indian MO) Tìm các số nguyên dương (x,y,n) thỏa (x,n + 1) = 1 và

xn + 1 = y n+1 .

T
Bài 3.12. Cho dãy các số tự nhiên a1 ,a2 , . . . thỏa mãn
t
(ai ,aj ) = (i,j) ∀i 6= j.
Tấ
Chứng minh rằng ai = i với mọi i.
Bài 3.13. Tìm ước chung lớn nhất của các số

an = 23n + 36n+2 + 56n+2


n

với n = 0,1,2, . . . ,1999.


yễ

(2001 Junior Balkan Olympic toán)


gu

Bài 3.14. Cho n ∈ N∗, đặt ak = (n + 1) Cnk , k = 0,1,2, . . . ,n. Chứng minh rằng

[a0 ,a1 , . . . ,an ] = [1,2, . . . ,n + 1] .


N

Bài 3.15. Cho các số nguyên dương lẻ a1 ,a2 , . . . ,an . Chứng minh rằng
 
a1 + a2 a2 + a3 an + a1
(a1 ,a2 , . . . ,an ) = , ,..., .
2 2 2

Lời giải.  
a1 + a2 a2 + a3 an + a1
Đặt a = (a1 ,a2 , . . . ,an ) và b = , ,..., . Khi đó ai = a · αi với mọi i = 1,n
2 2 2

Bài 3.16. Cho n là số nguyên dương chẵn và a, b là hai số nguyên dương và nguyên tố cùng
nhau. Tìm a, b biết a + b là ước của an + bn .

33
4. ĐỒNG DƯ

§4. Đồng dư
I. Các khái niệm về đồng dư
Định nghĩa 1. Cho a, b, m là các số nguyên, m 6= 0. Nếu a − b chia hết cho m thì a được gọi
là đồng dư với b modulo m, ký hiệu a ≡ b (mod m).
Trong trường hợp ngược lại, ta kí hiệu a 6≡ b (mod m).
Vậy a ≡ b (mod m) ⇔ a = b + km với k ∈ Z.
Định lí 1. Cho a,b,c,d là các số nguyên, khi đó ta có các tính chất sau:

a) Nếu a ≡ b (mod m) thì b ≡ a (mod m).

b) Nếu a ≡ b (mod m) và b ≡ c (mod m) thì a ≡ c (mod m).

c) Nếu a ≡ b (mod m) và c ≡ d (mod m) thì a + c ≡ b + d (mod m).

d) Nếu a ≡ b (mod m) và c ≡ d (mod m) thì ac ≡ bd (mod m).

hu
Chứng minh: Ta chứng minh tính chất thứ 3 và 4.
Ta có a ≡ b (mod m) và c ≡ d (mod m) nên

T
a = b + km và c = d + tm, k,t ∈ Z.

c) Suy ra t
a + c = b + d + (k + t)m ⇒ a + c ≡ b + d (mod m).
Tấ
d) Suy ra
ac = (b + km)(d + tm) = bd + (bt + kd)m + ktm2
Do đó ac ≡ bd (mod m).
Định lí 2. Cho các số nguyên a, b. Khi đó:
n

a) Nếu a ≡ b (mod m), k nguyên dương thì ak ≡ bk (mod m).


yễ

b) Nếu a ≡ b (mod m) và d|m thì a ≡ b (mod d).


gu

c) Nếu a ≡ b (mod m) thì ac ≡ bc (mod cm) với mọi c 6= 0.


m
d) Nếu ab ≡ ac (mod m) và (a,m) = d thì b ≡ c (mod ).
d
N

Chứng minh: Ta chứng minh tính chất 4.


Đặt a = da1 , m = dm1 với (a1 ,m1 ) = 1. Ta có
. .
ab − ac = da1 (b − c) .. m = dm1 ⇒ a1 (b − c) .. m1
.
Do (a1 ,m1 ) = 1 nên suy ra b − c .. m1 hay b ≡ c (mod m1 ).
Định nghĩa 2. Cho số nguyên m, tập

a = {b|b ≡ a (mod m)}

được gọi là một lớp đồng dư môđunlô m.


Do vậy, ta chia các số nguyên thành m lớp theo môđunlô m là {0, 1, . . . m − 1}.
Định lí 3. Cho các số nguyên a,b. Khi đó:

a) Nếu a ≡ b (mod mi ) với ( i = 1,2, . . . ,n) thì a ≡ b (mod [m1 ,m2 , . . . ,mn ])

34
4. ĐỒNG DƯ

b) Cho P (x) là đa thức hệ số nguyên, khi đó nếu a ≡ b (mod m) thì P (a) ≡ P (b) (mod m).
Chứng minh:
. . . .
a) Ta có a − b .. m1 , a − b .. m2 , . . ., a − b .. mn nên a − b .. [m1 ,m2 , . . . , mn ].
Hay a ≡ b (mod [m1 ,m2 , . . . ,mn ]).

b) Giả sử P (x) = an xn + an−1 xn−1 + . . . + a1 x + a0 với ai ∈ Z.


Khi đó n
X
P (a) − P (b) = ai (ai − bi ).
i=1
i i
Vì a − b ≡ a − b ≡ 0 (mod m) nên ta có

P (a) − P (b) ≡ 0 (mod m).

hu
Ví dụ 4.1. Tìm số dự của phép chia 62015 cho 37.

Lời giải.
Ta có 62 ≡ −1 (mod 37) nên ta có

T
62015 = 6 · (62 )1007 ≡ 6(−1)1007 ≡ −6 (mod 37)

Vậy số dư trong phép chia 62015 cho 37 là 31.


t 
Tấ
Ví dụ 4.2. Chứng minh rằng 641|232 + 1.

Lời giải.
Ta có 641 = 5 · 27 + 1 = 24 + 54 . Do đó, ta suy ra
n

5 · 27 ≡ −1 (mod 641) và 54 ≡ −24 (mod 641)


yễ

Suy ra
54 · 228 ≡ 1 (mod 641)
gu

hay
−24 · 228 ≡ 1 (mod 641).

Vậy 641 232 + 1. 
N

7
Ví dụ 4.3. Tìm chữ số tận cùng của 77 .

Lời giải.
Ta có 77 ≡ −1 (mod 4) nên 77 = 4k + 3. Khi đó
7
77 = 74k+3 = (72 )2k+1 .7 ≡ (−1)2k+1 .7 ≡ −7 (mod 10)
7
Vậy chữ số tận cùng của 77 là 3. 

Ví dụ 4.4. Chứng minh rằng với mọi n là số tự nhiên thì



133 11n+2 + 122n+1 .

Lời giải.

35
4. ĐỒNG DƯ

Ta có
11n+2 + 122n+1 = 133 · 11n + 12 (144n − 11n ) .

144n − 11n ≡ 144 − 11 ≡ 133 ≡ 0 (mod 133)
nên ta suy ra
133 11n+2 + 122n+1 .


Ví dụ 4.5. Chứng minh rằng


19 · 8n + 17
luôn là hợp số với mọi số tự nhiên n.

Lời giải.
Ta xét các trường hợp sau

hu
• n = 2k, khi đó
19 · 8n + 17 ≡ 1 · (−1)2k + 2 ≡ 3 ≡ 0 (mod 3)
.
Hay 19 · 8n + 17..3.

T
• n = 4k + 1, ta có
t
19·8n +17 = 19·84k+1 +17 = 19·8·642k +17 ≡ 6·8·(−1)2k +17 ≡ 9+17 ≡ 26 ≡ 0 (mod 13).
Tấ
.
Suy ra 19 · 8n + 17..13.

• n = 4k + 3, tuong tự ta có
.
19.8n + 17..5.
n

Và do 19 · 8n + 17 ≥ 36 nên 19 · 8n + 17 là hợp số. 


yễ

Ví dụ 4.6. Cho p là số nguyên tố và {r1 , r2 , . . . , rm } là dãy tăng gồm các số nguyên


dương thỏa rim ≡ 1 (mod p) với i = 1, 2, . . . , m. Chứng minh rằng
gu

xm − 1 ≡ (x − r1 )(x − r2 ) . . . (x − rm ) (mod p)
N

với mọi x ∈ Z.

Lời giải.
Ta có m
X
m
f (x) = x − 1 = ai (x − r1 )(x − r2 ) . . . (x − ri ) + a0 .
i=1

Ta có am = 1. Vì f (ri ) = rim − 1 ≡ 0 (mod p) với mọi i = 0,1, . . . ,m − 1 nên ta suy ra ai ≡ 0


(mod p) với mọi i = 0,1, . . . ,m − 1.
Vậy xm − 1 ≡ (x − r1 )(x − r2 ) . . . (x − rm ) (mod p). 

Ví dụ 4.7. Cho các số nguyên a, b, c thỏa mãn 9 a2 + b2 + c2 . Chứng minh rằng

9 (a2 − b2 )(b2 − c2 )(c2 − a2 ).

Lời giải.
Xét số nguyên n.

36
4. ĐỒNG DƯ

• Nếu n = 9k thì n2 ≡ 0 (mod 9)

• Nếu n = 9k ± 1 thì n2 ≡ 1 (mod 9)

• Nếu n = 9k ± 2 thì n2 ≡ 4 (mod 9)

• Nếu n = 9k ± 3 thì n2 ≡ 0 (mod 9)

• Nếu n = 9k ± 4 thì n2 ≡ 7 (mod 9)

Đặt a2 ≡ r1 (mod 9), b2 ≡ r2 (mod 9), c2 ≡ r3 (mod 9), ta có r1 , r2 , r3 ∈ {0,1,4,7}.


.
Do đó a2 + b2 + c2 ..9 nên r1 + r2 + r3 ≡ 0 (mod 9). Do đó, ta có các trường hợp sau:

• r1 = r2 = r3 = 0.

• Trong ba số r1 , r2 , r3 có một số bằng 1 và hai số bằng 4.

• Trong ba số r1 , r2 , r3 có một số bằng 7 và hai số bằng 1.

hu
• Trong ba số r1 , r2 , r3 có một số bằng 4 và hai số bằng 7.

Tất cả các trường hợp trên ta đều suy ra 9 (a2 − b2 )(b2 − c2 )(c2 − a2 ). 

T
II. Đồng dự tuyến tính
t
Định nghĩa 3. Cho các số nguyên a,b,m. Khi đó phương trình
Tấ
ax ≡ b (mod m) (1)

được gọi là phương trình đồng dư tuyến tính.


Định lí 4. Cho a,b,m là các số nguyên, m > 0 và (a,m) = d. Khi đó
n

• Nếu d 6 | b thì (1) vô nghiệm.


yễ

• Nếu d | b thì (1) có đúng d nghiệm không đồng dư môđunlô m.


gu

Chứng minh: Số nguyên x là nghiệm của (1) khi và chỉ khi tồn tại số nguyên y sao cho

ax − my = b (2)
N

Do đó, nếu d 6 | b thì (2) vô nghiệm. Xét d|b.


Vì (a,m) = d nên tồn tại các số nguyên s,t sao cho

d = at + ms.

Mà d|b nên
b = ud = a(ut) + (us)m.
Do đó x0 = ut là một nghiệm của (1).
m
Ta chứng minh x = x0 + .k (3) với k ∈ Z đều là nghiệm của (1).
d
Thật vậy
a
ax ≡ ax0 + m k ≡ b (mod m).
d
Ngược lại, mọi nghiệm của (1) đều có dạng (3). Gọi x là nghiệm của (1), ta có
a m
ax − my = b ⇔ a(x − ut) = m(y − us) ⇔ (x − ut) = (y − us).
d d

37
4. ĐỒNG DƯ
a m m m
Do , = 1 nên x − ut hay x = ut + k với k ∈ Z.
d d d d
Ta chứng minh được (1) có đúng d nghiệm không đồng dư theo môđunlô m.

Ví dụ 4.8. Giải phương trình 3x ≡ 5 (mod 7).

Lời giải.
Ta có (3,7) = 1 nên phương trình luôn có nghiệm.Ta thấy x = 4 là một nghiệm của phương
trình. Do đó, nghiệm của phương trình đã cho là x = 4 + 7t, t ∈ Z. 
Nhận xét. Nếu (a,m) = 1 thì phương trình ax ≡ 1 (mod m) luôn có nghiệm. Nghiệm đó được
gọi là nghịch đảo của a thỏa môđunlô m.

III. Hệ thặng dư đầy đủ - Hệ thặng dư thu gọn


Định nghĩa 4. Cho tập A = {a1 , a2 , . . . , an }. Giả sử ri , 0 ≤ ri ≤ n − 1 là số dư của các số ai
khi chia cho n. Khi đó, nếu tập các số dư {r1 , r2 , . . . , rn } trùng với tập {0, 1, 2, . . . , n − 1} thì

hu
ta nói A là một hệ thặng dư đầy đủ (hay gọi tắt là HĐĐ (mod n)).
Vậy A lập thành một HĐĐ (mod n) nếu i 6= j thì ai 6= aj .
Định lí 5. Cho A = {a1 , a2 , . . . , an } là một HĐĐ (mod n). Khi đó ta có

T
• Với mọi m ∈ Z luôn tồn tại duy nhất i ∈ {1, 2, . . . , n} sao cho m ≡ ai (mod n).

• Với mọi a ∈ Z thì tập A + a = {a + a1 , a + a2 , . . . , a + an } lập thành một HĐĐ (mod n).
t
• Nếu c ∈ Z và (c,n) = 1 thì tập cA = {ca1 , ca2 , . . . , can } lập thành một HĐĐ (mod n).
Tấ
Định nghĩa 5. Cho tập B = {b1 , b2 , . . . , bk } với (bi , n) = 1. Gọi ri , 0 ≤ ri ≤ n − 1 là số dư của
bi khi chia cho n, ta có (ri , n) = 1. Nếu tập {r1 , r2 , . . . , rk } trùng với tập các số nguyên dương
không vượt quá n và nguyên tố cùng nhau với n thì tập B được gọi là hệ thặng dư thu gọn
n

mod n, gọi tắt là HTG mod n


Vậy tập B = {b1 , b1 , . . . , bk } lập thành HTG khi và chỉ khi
yễ

a) (bi ,n) = 1 với mọi i = 1,k.

b) bi 6= bj với mọi i 6= j.
gu

c) Với mọi số nguyên x mà (x,n) = 1 tồn tại duy nhất chỉ số i ∈ {1, 2, . . . , k} sao cho x ≡ bi
(mod n).
N

Ví dụ 4.9. Cho p là số nguyên tố, p > 3. Chứng minh rằng với mọi số nguyên dương
x, 1 < x < p − 1 tồn tại duy nhất số nguyên dương y < p, y 6= x sao cho xy ≡ 1 (mod p).

Lời giải.
Xét p số x, 2x, . . . , (p − 1)x, px ta thấy p số này lập thành một HĐĐ theo mod p. Do đó,
tồn tại duy nhất y ∈ {1, 2, . . . , p − 1} sao cho xy ≡ 1 (mod p). Gải sử x = y, khi đó x2 − 1 =
. . .
(x − 1)(x + 1) .. p. Suy ra x − 1 .. p hoặc x + 1 .. p, cả hai trường hợp này đều không xảy ra. 

Ví dụ 4.10. Cho đa thức P (x) = x3 − 11x2 − 87x + m trong đó m ∈ Z. Chứng minh rằng
.
với mọi m tồn tại số nguyên n sao cho P (n) .. 191.

Lời giải.
Trước hết ta có bổ đề sau
Bổ đề 1. Cho p là số nguyên tố có dạng 3k + 2. Khi đó x3 ≡ y 3 (mod p) ⇔ x ≡ y (mod p).

38
4. ĐỒNG DƯ

Chứng minh: Ta xét các trường hợp sau

• x ≡ 0 (mod p), suy ra y 3 ≡ 0 (mod p) nên y ≡ 0 (mod p). Do đó x ≡ y (mod p).

• x 6≡ 0 (mod p), suy ra y 6≡ 0 (mod p). Theo định lí Phecma nhỏ ta có

x3k+1 ≡ y 3k+1 ≡ 1 (mod p). (1)

Mà x3 ≡ y 3 (mod p) nên
x3k ≡ y 3k (mod p). (2)
Từ (1) và (2) suy ra x ≡ y (mod p).

Vậy bổ đề được chứng minh.


Trở lại bài toán:
Ta chứng minh với n1 , n2 ∈ N thì

P (n1 ) ≡ P (n2 ) (mod 191) ⇔ n1 ≡ n2 (mod 191). (3)

hu
Thật vậy, ta có
27P (n) = (3n − 11)3 − 19 · 191n + 113 + 27m

T
nên

P (n1 ) ≡ P (n2 ) (mod 191) ⇔ 27P (n1 ) ≡ 27P (n2 ) (mod 191)
t
⇔ (3n1 − 11)3 ≡ (3n2 − 1)3 (mod 191)
Tấ
⇔ 3n1 − 11 ≡ 3n2 − 11 (mod 191) ⇔ n1 ≡ n2 (mod 191).

Mà {1, 2, . . . , 191} lập thành HĐĐ theo mod 191 nên {P (1), P (2), . . . , P (191)} lập thành
HĐĐ theo mod 191. Do đó với mọi m luôn tồn tại n ∈ {1, 2, . . . , 191} để P (n) ≡ 0 (mod 191).

n
yễ

IV. Các định lí về đồng dư


1. Định lí Phecma nhỏ
gu

Định lí 6. Giả sử p là số nguyên tố và a là số nguyên dương mà p 6 | a. Khi đó, ta có

ap−1 ≡ 1 (mod p).


N

Chứng minh. Xét các số a, 2a, 3a, . . . , (p − 1)a. Ta thấy,trong dãy số trên không có số nào chia
hết cho p và không có hai số nào có cùng số dư khi chia cho p. Do đó, tập các số dư khi chia cho
p của các số trên là tập {1, 2, 3, . . . , p − 1}. Suy ra

a · 2a · 3a · · · (p − 1)a ≡ 1 · 2 · 3 · · · (p − 1) (mod p).

Hay
(p − 1)! · ap−1 ≡ (p − 1)! (mod p).
Mà ((p − 1)!, p) = 1 nên ta có
ap−1 ≡ 1 (mod p).

Nhận xét. Cho số nguyên tố p và số nguyên a. Khi đó

ap ≡ a (mod p).

39
4. ĐỒNG DƯ

Ví dụ 4.11. Chứng minh rằng

2340 ≡ 1 (mod 341).

Lời giải.
Ta có 341 = 11.31
Theo định lí Phecma nhỏ thì
.
210 − 1..11
nên
.
2340 − 1..11.
Mặt khác, cũng theo định lí Phecma thì

231 ≡ 2 (mod 31)

hu
nên
2341 = (231 )11 ≡ 211 ≡ 2.210 ≡ 2 (mod 31)

T
Mà (2,31) = 1 nên
2340 ≡ 1 (mod 31).
Từ đó, suy ra 2340 ≡ 1 (mod 341). t 
4 ! Tương tự như cách chứng minh bài toán trên, ta có kết quả tổng quát sau: Cho hai số nguyên
Tấ
tố phân biệt p và q. Nếu ap ≡ a (mod q) và aq ≡ a (mod p) thì

apq ≡ a (mod pq).


n

Ví dụ 4.12. Cho n ≥ 2, a > 0 là các số nguyên và p là số nguyên tố sao cho ap ≡ 1


(mod pn ). Chứng minh rằng nếu p > 2, thì a ≡ 1 (mod pn−1 ), và nếu p = 2, thì a ≡ ±1
yễ

(mod 2n−1 ).

Lời giải.
gu

Ta có ap ≡ 1 (mod pn ) với n ≥ 2, nên ap ≡ 1 (mod p).


Mặt khác, theo định lí nhỏ Fermat, thì
N

ap ≡ a (mod p), do đó a ≡ 1 (mod p).

Với a = 1, kết quả hiển nhiên đúng; mặt khác, đặt a = 1 + kpd , trong đó d ≥ 1 và p 6 | k. Khi đó
với p > 2, thì ap = 1 + kpd+1 + M p2d+1 với M là số nguyên. Do đó d + 1 ≥ n và a ≡ 1 (mod pn−1 ).
Trường hợp p = 2, ta có 2n a2 − 1 = (a − 1)(a + 1). Vì chúng khác 2, nên cả hai đều không là
bội của 4. Do đó a + 1 hoặc a − 1 không chia hết cho 2n−1 , nghĩa là a ≡ ±1 (mod 2n−1 ). 

Ví dụ 4.13. Tìm số các số n > 1 sao cho a25 − a chia hết cho n với mỗi số nguyên a.

Lời giải.
.
Gọi p là một ước nguyên tố của n, theo đề bài ta chọn a = p thì ta có p25 − p .. p .
Nhưng p2 6 p25 − p nên p2 6 n. Do đó, n là bội của các số nguyên tố phân biệt.
Mặt khác với a = 2 ta có 225 − 2 = 2 · 32 · 5 · 7 · 13 · 17 · 241.
Vì 325 ≡ −3 (mod 17) và 325 ≡ 32 (mod 241) nên n không chia hết cho 17 và 241.
.
Mà theo định lí Phecma thì a25 ≡ a (mod p) với p ∈ {2, 3, 7, 13} nên a25 − a..n với mọi n là ước
của 2 · 3 · 7 · 13. 

40
4. ĐỒNG DƯ

Ví dụ 4.14. Chứng minh rằng dãy an = 2n − 3 với n ≥ 2 chứa vô số cặp số nguyên tố


cùng nhau.

Lời giải.
Ta có a2 = 1, a3 = 5, a4 = 13 là các số đôi một nguyên tố cùng nhau.
Giả sử ta có a2 ,a2 , . . . , ak là các số đôi một nguyên tố cùng nhau. Ta chỉ ra được tồn tại ak+1 sao
cho a2 ,a2 , . . . , ak , ak+1 đôi một nguyên tố cùng nhau.
Gọi p1 ,p2 , . . . ,pt là tất cả các ước nguyên tố của dãy số a2 ,a2 , . . . , ak . Theo định lí Phecma nhỏ
.
2pi −1 − 1..pi ∀i = 1,t.

Suy ra
.
2(p1 −1)(p2 −1)...(pt −1) − 1..pi .
Đặt k + 1 = (p1 − 1)(p2 − 1) . . . (pt − 1) thì ak+1 ≡ −2 (mod pi ).
Suy ra a1 , a2 , . . . , ak , ak+1 đôi một nguyên tố cùng nhau. Theo nguyên lí quy nạp ta có bài toán

hu
được chứng minh. 

2. Định lí Ơ le

T
Định lí 7. Cho số nguyên dương n, kí hiệu ϕ (n) là số các số nguyên dương không vượt quá n
và nguyên tố cùng nhau với n. Khi đó với mọi số nguyên a mà (a, n) = 1 thì
t
aϕ(n) ≡ 1 (mod n).
Tấ
Chứng minh. Gọi r1 , r2 , . . . , rk là các số nguyên dương không vượt quá n và nguyên tố cùng
nhau với n.
Xét k số r1 a, r2 a, . . . , rk a. Vì (ri a, n) = 1 với mọi i = 1, 2, . . . , k. Do đó ta có
n

r1 a · r2 a · · · rk a ≡ r1 r2 · · · rk (mod n),
yễ

hay
aϕ(n) ≡ 1 (mod n).
gu

4! Với n = pα1 1 pα2 2 · · · pαk k thì ta có


    
N

1 1 1
ϕ (n) = n 1 − 1− ··· 1 − .
p1 p2 pk

Ví dụ 4.15. Chứng minh với mỗi số tự nhiên m > 0 thì tồn tại vô hạn số tự nhiên n sao
cho với moị số tự nhiên a nguyên tố cùng nhau với m ta luôn có an − 1 chia hết cho m.

Lời giải.
Theo định lý Euler ta có aφ(m) ≡ 1 (mod m) nên nếu chọn n = kφ(m) với k = 1, 2, 3, . . . thì
hiển nhiên n sẽ nhận vô hạn giá trị và ta có :
k
an = aφ(m) ≡ 1k = 1 (mod m).

Bài toán được chứng minh. 

41
4. ĐỒNG DƯ

Ví dụ 4.16. Chứng minh rằng với mỗi số nguyên dương s mà (s, 10) = 1, luôn tồn tại số
.
nguyên dương n có tổng các chữ số bằng s và n..s.

Lời giải.
Đặt n = 10s·ϕ(s) + 10(s−1)·ϕ(s) + . . . + 10ϕ(s) , n có tổng các chữ số bằng s.
. .
Theo định lí Ơ-le, ta có 10ϕ(s) − 1 .. s nên 10kϕ(s) − 1 .. s với k ∈ N∗ .
.
Suy ra n = 10s·ϕ(s) − 1 + 10(s−1)·ϕ(s) − 1 + . . . + 10ϕ(s) − 1 + s..s.
  


Ví dụ 4.17. Cho n là số tự nhiên và nn +1 có tập hợp các số nguyên tố là {p1 , p2 , . . . , pk }.


Chứng minh n | (p1 − 1) · · · (pk − 1).

Lời giải.
Ta có
nn + 1 = pα1 1 pα2 2 . . . pαk k .

hu

ϕ (nn + 1) = pα1 1 −1 (p1 − 1) pα2 2 −1 (p2 − 1) . . . pαk k −1 (pk − 1)
và (n; pi ) = 1 nên ta chứng minh n |ϕ(nn + 1) .

T
Theo thuật toán Eclic ta có ϕ (nn + 1) = nq + r và (nn + 1; n) = 1.
Suy ra
n
nϕ(n +1) ≡ 1 (mod nn + 1) ⇒ (nn )q · nr ≡ 1 (mod nn + 1).
t
Do đó (−1)q · nr ≡ 1 (mod nn + 1).
Tấ

|(−1)q nr | < nn + 1 ⇒ (−1)q nr − 1 = 0 ⇒ r = 0.

n

3. Định lí Wilson
yễ

Định lí 8. Cho n là số nguyên dương. Khi đó, n là số nguyên tố khi và chỉ khi

(n − 1)! ≡ −1 (mod n).


gu

Chứng minh.

• Giả sử n là số nguyên tố, ta chứng minh (n − 1)! ≡ −1 (mod n).


N

– n = 2 ta có 1! ≡ −1 (mod2).
– Xét n ≥ 3. Với mỗi a ∈ {1, 2, . . . , n − 1}, tồn tại nghịch đảo a0 ∈ {1, 2, . . . , n − 1} để
aa0 ≡ 1 (mod n).
n−3
Mà 1 và n − 1 là nghịch đảo của chính nó nên các số 2, 3, . . . , n − 2 chia thành
2
cặp, mà tích của chúng đồng dư 1 môđun n. Do đó

(n − 1)! ≡ 1. (n − 1) ≡ −1 (mod n).

• Giả sử (n − 1)! ≡ −1 (mod n), ta chứng minh n là số nguyên tố.


Giả sử n là hợp số, khi đó n = a · b với 2 ≤ a, b ≤ n − 1.
Vì a | n nên (n − 1)! ≡ −1 (mod a). Mà a | (n − 1)! nên a | 1 vô lí.
Vậy n là số nguyên tố.

42
4. ĐỒNG DƯ

Ví dụ 4.18. Cho n ∈ N, n ≥ 3. Chứng minh nếu n + 2 là một số nguyên tố thì n! − 1 là


một hợp số.

Lời giải.
Vì n + 2 là số nguyên tố nên theo định lý Wilson ta có (n + 1)! ≡ −1( mod n + 2) hay (n + 1)! + 1
chia hết cho n + 2. Mặt khác, ta lại có (n + 1)! + 1 = (n + 2) · n! − n! + 1 nên suy ra n! − 1 chia
hết cho n + 2. Hơn nữa do n ≥ 3 ta luôn có n! − 1 > n + 2 nên ta có n! − 1 là hợp số. Bài toán
được chứng minh. 

Ví dụ 4.19. Cho p là số nguyên tố lẻ. Chứng minh rằng với mọi số nguyên dương n < p
ta có
(n − 1)! · (p − n)! ≡ (−1)n (mod p).

Lời giải.

hu
Theo định lí Wilson ta có (p − 1)! ≡ −1 (mod p).

(p − 1)! = (n − 1)! · n · (n + 1) · · · (p − n) = (n − 1)! (p − (p − n)) (p − (p − n − 1)) · · · (p − 1)

T
và p − k ≡ −k (mod p) với mọi k = 1, 2, . . . , n − p nên
(p − 1)! ≡ (n − 1)! · (−1)n−p · (p − n) (p − n − 1) · · · 1 ≡ (−1)p−n (n − 1)! (p − n)! (mod p).
Do p lẻ nên ta có
t
(−1)p−n (n − 1)! (p − n)! ≡ −1
Tấ
(mod p).
n
Hay (n − 1)! (p − n)! ≡ (−1) (mod p) (đpcm). 

Ví dụ 4.20. Cho p là số nguyên tố lẻ và a1 , a2 , . . . , ap là p số nguyên phân biệt sao cho


không có hai số nào có hiệu chia hết cho p. Chứng minh rằng tồn tại chỉ số i ∈ {1,2, . . . ,p}
n

sao cho
ai + a1 .a2 . . . ap ≡ 0 (mod p2 ).
yễ
gu

Lời giải.
Vì a1 , a2 , . . . , ap có số dư khác nhau khi chia cho p nên tồn tại i ∈ {1, 2, . . . , p} sao cho ai ≡ 0
(mod p). Khi đó
N

a1 a2 . . . ai−1 · ai+1 . . . ap ≡ (p − 1)! ≡ −1 (mod p),


hay
.
a1 a2 · · · ai−1 · ai+1 · · · ap + 1 .. p.
Suy ra
ai + a1 · a2 · · · ap ≡ 0 (mod p2 ).
Bài toán được chứng minh. 

4. Định lí thặng dư trung hoa

Định lí 9. Cho n ≥ 2 số nguyên dương đôi một nguyên tố cùng nhau m1 , m2 , . . . , mn và n số


nguyên bất kì a1 , a2 , . . . , an . Khi đó hệ đồng dư tuyến tính


 x ≡ a1 (mod m1 )

x ≡ a (mod m )
2 2

 .........

x ≡ an (mod mn )

43
4. ĐỒNG DƯ

có nghiệm duy nhất theo môđunlô M = m1 m2 · · · mn .


M .
Chứng minh. Đặt Ni = với mọi i = 1,n. Khi đó Ni .. mj với mọi i 6= j đồng thời (N1 , mi ) = 1,
mi
nên tồn tại ti sao cho Ni · t1 ≡ 1 (mod mi ). Đặt Ai = Ni · ti và
x0 = A1 a1 + A2 a2 + · · · + An an .
Ta có x0 ≡ Ai ai ≡ ai (mod mi ), hay x0 là một nghiệm của hệ phương trình đã cho.
Giả sử x1 cũng là một nghiệm của hệ, khi đó ta có x1 ≡ ai ≡ x0 (mod mi ) nên ta có
x1 ≡ x0 (mod M ).

Ví dụ 4.21. Giải hệ phương trình đồng dư



x ≡ 1 (mod 2)

x ≡ 2 (mod 3)

hu

x ≡ 3 (mod 5).

T
Lời giải.
Ta có M = 30 nên N1 = 15, t1 = 1; N2 = 10, t2 = 1; N3 = 6, t3 = 1. Do đó
t
x0 = 1 · 15 · 1 + 1 · 10 · 2 + 1 · 6 · 3 = 53.
Tấ
Vậy x ≡ 53 ≡ 23 (mod 30) là nghiệm của hệ. 

Ví dụ 4.22. Giải phương trình đồng dư x2 ≡ 1 (mod 77).

Lời giải.
n

Ta có 77 = 7 · 11. Do đó
yễ

(
x2 ≡ 1 (mod 7)
x2 ≡ 1 (mod 77) ⇔ (∗)
x2 ≡ 1 (mod 11).
gu

Ta có
x2 ≡ 1 (mod 7) ⇔ x ≡ ±1 (mod 7)
N

x2 ≡ 1 (mod 11) ⇔ x ≡ ±1 (mod 11)


Do đó hệ đã cho tương đương với
(
x ≡ 1 (mod 7)
(1.1)
x ≡ 1 (mod 11)
(
x ≡ 1 (mod 7)
(1.2)
x ≡ −1 (mod 11)
(
x ≡ −1 (mod 7)
(1.3)
x ≡ 1 (mod 11)
(
x ≡ −1 (mod 7)
(1.4)
x ≡ −1 (mod 11)

Giải các hệ trên ta có x ≡ 1 (mod 77), x ≡ 43 (mod 77), x ≡ 34 (mod 77), x ≡ −1 (mod 77).


44
4. ĐỒNG DƯ

Ví dụ 4.23. Cho hai số nguyên dương p, q nguyên tố cùng nhau. Chứng minh rằng tồn
tại số nguyên k sao cho (pq − 1)n k + 1 là hợp số với mọi số nguyên dương n.

Lời giải.
Vì (p, q) = 1 nên theo định lí thặng dư Trung hoa, tồn tại k0 thỏa mãn
(
k0 ≡ 1 (mod p)
k0 ≡ −1 (mod q).

Ta xét các trường hợp sau


• Nếu n chẵn thì

(pq − 1)n k0 + 1 ≡ (−1)n k0 + 1 ≡ k0 + 1 ≡ 0 (mod q).

• Nếu n lẻ thì

hu
(pq − 1)n k0 + 1 ≡ (−1)n k0 + 1 ≡ −k0 + 1 ≡ 0 (mod p).

Do đó (pq − 1)n k0 + 1 luôn là hợp số với mọi n. 

T
Ví dụ 4.24. Chứng minh rằng với mọi số nguyên dương n, luôn tồn tại các số nguyên
x, y sao cho
.
x2 − 34y 2 + 1 .. n.
t
Tấ
Lời giải.
Ta có
x2 − 34y 2 + 1 = x2 − 25y 2 − 9y 2 + 1 = x2 − (5y)2 − (3y)2 − 1 .
 
n

• Nếu (n, 3) = 1 thì tồn tại y0 để 3y0 ≡ 1 (mod n). Ta chọn x, y sao cho y = y0 và x ≡ 5y0
yễ

(mod n) thì
x2 − (5y)2 − (3y)2 − 1 ≡ 0 (mod n).
 
gu

• n = 3k · m với (m, 3) = 1. Ta chọn y1 , x1 sao cho

3y1 ≡ 1 (mod m), x1 ≡ 5y1 (mod m).


N

Khi đó
.
x21 − 34y12 + 1 .. m.
Chọn x1 , y2 sao cho
5y2 ≡ 1 (mod 3k ), x2 ≡ 3y2 (mod 3k ).
Khi đó
x22 − 34y22 + 1 = x22 − (5y2 )2 − (3y2 )2 − 1 ≡ 0 (mod 3k ).
 

Theo định lí thặng dự Trung hoa tồn tại x0 , y0 sao cho


( (
x0 ≡ x1 (mod m) y0 ≡ y1 (mod m)
và .
x0 ≡ x2 (mod 3k ) y0 ≡ y2 (mod 3k )

Khi đó
.
x20 − 34y02 + 1 .. n.

45
4. ĐỒNG DƯ

V. Bài tập
Bài 4.1. Chứng minh rằng :
.
a) Nếu (a,35) = 1 thì a12 − 1 .. 35.
.
b) Nếu (a,42) = 1 thì a6 − 1 .. 168.
.
c) Nếu (a,133) = 1 thì a18 − 1 .. 133.

Lời giải.

a) Ta có (a,5) = (a,7) = 1 nên theo định lí Phecma nhỏ ta có:

a4 ≡ 1 (mod 5) ⇒ a12 ≡ 1 (mod 5)

a6 ≡ 1 (mod 7) ⇒ a12 ≡ 1 (mod 7).

hu
.
Từ đó, suy ra a12 ≡ 1 (mod 35) hay a12 − 1 .. 35.

b) Ta có 168 = 23 · 3 · 7 và (a,2) = (a,3) = (a,7) = 1 nên theo định lí Phecma nhỏ ta có:

T
a2 ≡ 1 (mod 3) ⇒ a6 ≡ 1 (mod 3), a6 ≡ 1 (mod 7).
.
Vì a lẻ nên a2 − 1 = (a − 1) (a + 1) .. 8. Suy ra a6 ≡ 1 (mod 8).
.
t
Tấ
Do vậy a6 ≡ 1 (mod168) hay a6 − 1 .. 168.

c) Ta có 133 = 7.19 và (a,7) = (a,19) = 1 nên theo định lí Phecma nhỏ ta có

a18 ≡ 1 (mod 1)9, a6 ≡ 1 (mod 7) ⇒ a18 ≡ 1 (mod 7).


n

.
Suy ra a18 ≡ 1 (mod 133) hay a18 − 1 .. 133.
yễ


20112017
Bài 4.2. Tìm hai chữ số cuối cùng của 3 .
gu

Lời giải.
2017
Ta cần tìm số dư của 32011 khi chia cho 100.
Ta có ϕ (100) = 40 nên theo định lí Ơ-le ta có 340 ≡ 1 (mod 100).
N

Lại có ϕ (40) = 16 nên 201116 ≡ 1 (mod 40).


Mà 2017 = 16.126 + 1 nên 20112017 ≡ 2011 ≡ 11 (mod 40) hay 20112017 = 40k + 11.
2017
Suy ra 32011 ≡ 340k+11 ≡ 311 ≡ 47 (mod 100).
2017
Vậy hai chữ số cuối cùng của 32011 là 47. 
.
Bài 4.3. Cho p, q là hai số nguyên tố lẻ phân biệt thỏa mãn q − 1 .. p − 1 và số nguyên a thỏa
.
(a, pq) = 1. Chứng minh rằng aq−1 − 1 .. pq.
Lời giải.
Theo định lí Phecma nhỏ ta có

aq−1 ≡ 1 (modq) và ap−1 ≡ 1 (mod p).


.
Vì q − 1 .. p − 1 nên aq−1 ≡ 1 (mod p). Mà (p,q) = 1 nên aq−1 ≡ 1 (mod pq). 

46
4. ĐỒNG DƯ

Bài 4.4. Cho số nguyên dương lẻ a > 1. Tìm số nguyên dương n nhỏ nhất sao cho 22015 là một
ước của an − 1.
Lời giải.
Ta có ϕ (22015 ) = 22014 .
Đặt 22014 = n · m + r, 0 ≤ r < n.
Theo định lí Ơ-le ta có :
.
aϕ(2 ) − 1 .. 22015 .
2015

Suy ra
.
2nm+r − 1 = 2r (2mn − 1) + 2r − 1 .. 22015 .
. .
Mà 2mn − 1 .. 22015 nên 2r − 1 .. 22015 , suy ra r = 0 hay n là ước của 22015 , từ đó n = 2k với
k ≤ 2015. Khi đó  k−1 
k
a2 − 1 = (a − 1) (a + 1) a2 + 1 · · · a2 + 1 .


t
Với t ≥ 0 ta có a2 + 1 chia hết cho 2 mà không chia hết cho 4.

hu
Đặt a = 2p · q + 1 với p ≥ 1, q lẻ thì a − 1 chia hết cho 2p mà không chia hết cho 2p+1 .
k
Suy ra a2 − 1 chia hết cho 2k+p không chia hết cho 2k+p+1 .
Suy ra k + p = 2015 ⇔ k = 2015 − p. Vậy n = 22015−p là số cần tìm. 

T
Bài 4.5. Cho n là số nguyên dương. Chứng minh rằng

7 | n3 + 3n ⇔ 7 | n3 · 3n + 1.
t
Tấ
Lời giải.
Dễ thấy 7 6 | n và
n3 3n + n3 − n3 .3n + 1 = n6 − 1.
 

Theo định lí Phecma nhỏ ta có 7 | n6 − 1. Do đó


n

7 | n3 + 3n ⇔ 7 | n3 .3n + 1.
yễ


Bài 4.6. Giải các hệ phương trình đồng dư
gu

( 
x ≡ 4 (mod 5) x ≡ 1 (mod 8)

a) .
x ≡ −1 (mod 7) c) x ≡ 3 (mod 9) .
N


x ≡ −2 (mod 11)

b) x2 ≡ 1 (mod 6). d) x2 ≡ 1 (mod 144).

Lời giải.
HS tự làm. 
Bài 4.7. Chứng minh rằng với mọi số nguyên dương k luôn tồn tại k số nguyên dương liên tiếp
không là lũy thừa của một số nguyên dương nào khác.
Lời giải.
Xét k số nguyên tố p1 < p2 < · · · < pk . Gọi x0 là nghiệm của hệ phương trình đồng dư


 x ≡ p1 (mod p21 )

x ≡ p 2 − 1 (mod p22 )



x ≡ p3 − 2 (mod p23 )

·········



(mod p2k ).

x ≡ p − k + 1
k

47
4. ĐỒNG DƯ

Khi đó ta thấy
x0 , x0 + 1, x0 + 2, . . . , x0 + k − 1
không có số nào là lũy thừa của một số nguyên. 
r
pαi i , khi đó phương trình đồng dư
Q
Bài 4.8. Cho n =
i=1

P (x) ≡ 0 (mod n) (1)

có nghiệm khi và chỉ khi tất cả các phương trình đồng dư

P (x) ≡ 0 (mod pαi i ) (2)

có nghiệm. Hơn nữa nếu gọi Ni là số nghiệm của phương trình đó thì phương trình P (x) ≡ 0
(mod n) có đúng N1 · · · Nr nghiệm.
Lời giải.

hu
• Giả sử phương trình (1) có nghiệm x = x0 . Khi đó, do P (x0 ) ≡ 0 (mod n) nên P (x0 ) ≡ 0
(mod pα1 i ), hay các phương trình (2) có nghiệm x0 .

• Giả sử ứng với mỗi i = 1,2, . . . ,r phương trình P (x) ≡ 0 (mod pαi i ) có nghiệm xi . Theo

T
định lí thặng dư Trung hoa thì tồn tại x0 thỏa mãn hệ phương trình

x ≡ x1 (mod pα1 1 )



x ≡ x (mod pα2 )
 t
2 2
.
Tấ

 ·········

x ≡ xr (mod pαr r )

Khi đó P (x0 ) ≡ P (xi ) (mod pαi i ) với mọi i = 1, r, nên P (x0 ) ≡ 0 (mod n).
n


yễ

Bài 4.9. Cho n là số nguyên dương lớn hơn 1, hãy tính số nghiệm của phương trình đồng dư
x(x − 1) ≡ 0 ( mod n).
Lời giải.
Ta viết n dưới dạng phân tích thừa số nguyên tố n = ni=1 ai . Ta đi tìm số nghiệm của phương
gu

Q
trình đồng dư: x(x − 1) ≡ 0 ( mod pai i ). Chú ý rằng x và x − 1 nguyên tố cùng nhau nên phương
trình đồng dư này chỉ có 2 nghiệm x ≡ 0 (mod pai i ) hoặc x ≡ 1 (mod pai i ), từ đó suy ra phương
trình này có tất cả 2r nghiệm . 
N

Bài 4.10. Chứng minh rằng với mọi số nguyên dương n luôn tồn tại số nguyên k sao cho 2n k + 1
là hợp số.
Lời giải.
Gọi Fn là số hạng thứ n của dãy Fibonaci, ta có (Fi ,Fj ) = 1 vói mọi i 6= j; F1 , F2 , F3 , F4 là các
số nguyên tố và F5 = p · q với p = 641, q = 6700417.
Gọi k là nghiệm của hệ phương trình

k ≡ 1 (mod Fm ) với m = 1,2,3,4

k ≡ 1 (mod p) .

k ≡ −1 (mod q)

Gải sử n = 2m · t với t là số tự nhiên lẻ.

• m ≤ 4 ta có
mt
2n = 22 ≡ −1 (mod Fm ) ⇒ 2n · k + 1 ≡ −k + 1 ≡ 0 (mod Fm ).

48
4. ĐỒNG DƯ

• m = 5 ta có
mt
22 ≡ −1 (mod F5 ) ⇒ 2n k + 1 ≡ −k + 1 ≡ 0 (mod p).

• m > 5 ta có

2n ≡ 1 (mod F5 ) ⇒ 2n k ≡ −1 (mod q) ⇒ 2n k + 1 ≡ 0 (mod q).


125 n 5 5
Bài 4.11. Cho dãy số an = · 5 + · (−1)n − . Chứng minh rằng nếu số nguyên dương
6 3 2
..
chẵn h thỏa mãn an+h − an . 3996 với mọi số nguyên dương n thì h ≥ 108.
Lời giải.
Ta có 3996 = 22 · 33 · 37 và
n h

(−1)n · 10  125 · 5n 125 · 5 5 − 1
(−1)h − 1 + (5h − 1) =

an+h − an = .

hu
3 3 3
Suy ra
.

5h − 1 .. 4

T


. 
.
an+h − an .. 4 · 27 · 37 ⇔ 5h − 1 .. 81
.

5 − 1 .. 37

 h
t
(do (125 · 5n , 4) = (125 · 5n , 81) = (125 · 5n , 37) = 1) .
Tấ
.
Gọi k là số nguyên dương nhỏ nhất thỏa mãn 5k − 1 .. 37.
. .
Theo định lí Phecma nhỏ ta có 536 − 1 .. 37 nên suy ra 36 .. k. Do đó k ∈ {1, 2, 3, 4, 12, 18, 36}
thử trực tiếp ta thấy chỉ có k = 36 thỏa mãn, suy ra
n

. .
5h − 1 .. 37 ⇒ h .. 36. (1)
yễ

Chứng minh tương tự, ta cũng có:


. .
5h − 1 .. 81 ⇒ h .. ϕ(81) = 54.
gu

(2)
.
Từ (1) và (2) ta suy ra h .. [36,54] = 108 ⇒ h ≥ 108. 
N

Bài 4.12. Chứng minh rằng với mọi số nguyên tố p thì

pp+1 + (p + 1)p

không là số chính phương.


Lời giải.
Với p = 2 bài toán đúng.
Giả sử p ≥ 3 và pp+1 + (p + 1)p = t2 với vài số nguyên dương t .
Suy ra
p+1 p+1
  
t − p 2  t + p 2  = (p + 1)p .


p+1 p+1 p+1 p+1
   
t − p 2 , t + p 2  = t − p 2 , 2p 2  = 2

49
4. ĐỒNG DƯ

nên ta có
p+1 p+1
t ± p 2 = 2 u và t ∓ p 2 = 2v p .
p−1 p

Với u, v là hai số nguyên dương thỏa mãn 2uv = p + 1 và (u,v) = 1.


Suy ra
p+1

p 2 = 2p−2 up − v p .

up ≡ u (mod p), v p ≡ v (mod p) và 2p−1 ≡ 1 (mod p),
nên
2p−1 · up − 2v p ≡ u − 2v ≡ 0 (mod p)
Suy ra u ≡ 2v (mod p) . Lại có 2uv = p + 1 nên

u = 2v ⇒ p + 1 = 4v 2 ⇒ p = (2v − 1) (2v + 1) .

hu
Từ đây ta tìm được v = 1 ⇒ p = 3 ⇒ t2 = 145 (vô lí). Vậy bài toán được chứng minh. 
Bài 4.13 ( Olympic Toán Ban-căng 1999). Cho số nguyên tố p > 2 có dạng 3m + 2. Gọi

S = y 2 − x3 − 1 |0 ≤ x, y ≤ p − 1 ; x, y ∈ Z .


T
Chứng minh rằng tối đa có p phần tử của S chia hết cho p.
Lời giải. t
Bổ đề 2. Cho số nguyên tố p và số nguyên dương k > 1, nếu (k,p − 1) = 1 thì
Tấ
xk ≡ y k (mod p) ⇔ x ≡ y (mod p)

với mọi x,y.


Chứng minh.
n

• Nếu y ≡ 0 (mod p) thì kết quả là đúng.


yễ

• Xét y 6≡ 0 (mod p), khi đó


k
xk ≡ y k (mod p) ⇒ (xy 0 ) ≡ 1 (mod p),
gu

với y 0 là nghịch đảo của y theo môđun p, tức là yy 0 ≡ 1 (mod p).


Do đó, để chứng minh bổ đề, ta chứng minh: Nếu ak ≡ 1 (mod p) thì a ≡ 1 (mod p).
N

Thật vậy: Dễ thấy a 6≡ 0 (mod p).


Gọi d là số nguyên dương nhỏ nhất thỏa mãn ad ≡ 1 (mod p).
Vì ak ≡ 1 (mod p) nên d | k. Mặt khác ap−1 ≡ 1 (mod p) nên d|p − 1.
Do (k, p − 1) = 1 nên ta có d = 1 hay a ≡ 1 (mod p).
Bổ đề được chứng minh.
Trở lại bài toán:
Vì (3, p − 1) = 1 nên theo bổ đề trên ta có

1 , 2 , . . . , (p − 1)3 , p3
 3 3

có tập các số dư trung với tập


{1, 2, . . . , p − 1, p} .
Do đó, với mỗi 0 ≤ y ≤ p − 1, tồn tại suy nhất x ∈ {0, 1, 2, . . . , p − 1} sao cho

x3 ≡ y 2 − 1 (mod p) hay p|y 2 − x3 − 1.


Do đó trong tập S chứa nhiều nhất p phần tử chia hết cho p. 

50
4. ĐỒNG DƯ

2n + 1
Bài 4.14 (IMO lần thứ 31). Tìm các số nguyên dương n > 1 sao cho là số nguyên.
n2
Lời giải.
Gọi p là một ước nguyên tố nhỏ nhất của n và d là số nguyên dương nhỏ nhất sao cho ad ≡ 1
(mod p).
Ta có n lẻ p | 2n + 1 nên p | 22n − 1, suy ra d | 2n.
Mặt khác p | 2p−1 − 1 nên d | p − 1. Từ đó, suy ra d|(2n, p − 1) = 2 nên p = 3.
Do đó n = 3k .m với k ≥ 1, m lẻ và (m,3) = 1. Ta có
k k−1 k−1
x3 + 1 = (x + 1)(x2 − x + 1)(x2·3 − x3 + 1) . . . (x2·3 − x3 + 1).
Nên
km k−1 m k−1 m
23 + 1 = (2m + 1)(22m − 2m + 1)(22.3m − 23m + 1) · · · (22.3 − 23 + 1). (1)
Vì 22t − 2t + 1 ≡ 3 (mod 9) nên 22t − 2t + 1 chia hết cho 3 mà không chia hết cho 9. Suy ra
k−1 k−1
3k (22m − 2m + 1)(22.3m − 23m + 1) · · · (22.3 m − 23 m + 1).

hu
Mặt khác m lẻ nên 2m + 1 chia hết cho 3 mà không chia hết cho 9. Suy ra 2n + 1 chia hết cho 3k+1
và không chia hết cho 3k+2 , nên ta có k + 1 ≥ 2k hay k = 1. Suy ra n = 3m, do đó 9m2 |23m + 1.
Nếu m ≥ 5 thì ta gọi q là ước nguyên tố nhỏ nhất của m và t là số nguyên dương nhỏ nhất sao
cho q|2t − 1. Ta có q > 3, q|26m − 1 và q|2q−1 − 1 nên d|(6m,q − 1) suy ra d = 1,2,3,6. Hay q là

T
ước của một trong các số 1,3,7,63 = 32 .7. Mặt khác, với m > 3 thì 23m + 1 ≡ 2 (mod 7) nên ta
có điều vô lí. Do đó, m = 1 hay n = 3.
Vậy n = 3 là số cần tìm. t 
Bài 4.15. Cho số nguyên a > 1. Chứng minh rằng trong tập {a2 + a − 1, a3 + a2 − 1, . . .} chứa
Tấ
một tập con vô hạn với hai phần tử bất kì của nó nguyên tố cùng nhau.
Lời giải.
Ta có
a2 + a − 1, a3 + a2 − 1 = a2 + a − 1, a − 1 = (1, a − 1) = 1.
 
n

Giả sử ta chọn được n1 , n2 , . . . , nk thuộc tập đã cho và chúng đôi một nguyên tố cùng nhau.
yễ

Đặt N = n1 n2 · · · nk và nk+1 = aϕ(N )+1 + aϕ(N ) − 1. Khi đó (nk+1 , ni ) = 1 với mọi i = 1, 2, . . . , k.


Khi đó tập {n1 , n2 , . . . , nk+1 } đôi một nguyên tố cùng nhau. Từ đó ta có đpcm. 
Bài 4.16. Cho p là số nguyên tố dạng 4k + 1. Có tồn tại hay không số tự nhiên n sao cho
gu

.
n2 + 2n .. 2p.
Lời giải.
Do p = 4k + 1 nên ta có
N

4k ≡ −1 (mod p)
4k − 1 ≡ −2 (mod p)
4k − 2 ≡ −3 (mod p)
......
2k + 1 ≡ −2k (mod p).
Suy ra
(2k + 1)(2k + 2) · · · (4k) ≡ (2k)! ⇒ (4k)! ≡ [(2k)!]2
Vì p nguyên tố nên theo Willson ta có :
(4k)! ≡ −1 (mod p)
Suy ra [(2k)!]2 + 1 ≡ 0 (mod p).
Xét số n = 4k(2k)! . Khi đó theo định lý Fermat nhỏ ta có:
2n − 1 = [2(2k)! ]4k − 1 = [2(2k)! ]p−1 − 1 ≡ 0 (mod p).

51
4. ĐỒNG DƯ

Xét tổng :

n2 + 2n = [4k(2k)!]2 + 1 ≡ (4k)2 [(2k)!2 + 1] − [(4k)2 − 1] ≡ 0 (mod p).

Lại có n2 + 2n ≡ 0 (mod 2) và (2,p) = 1.


.
Suy ra n2 + 2n .. 2p.
.
Vậy tồn tại n để n2 + 2n .. 2p. 
Bài 4.17 (IMO Shortlist 1998). Xác định tất cả các số nguyên dương n sao cho với n này
tồn tại m ∈ Z thỏa mãn
2n − 1 | m2 + 9.

Lời giải.
Viết n = 2a · b với b là số tự nhiên lẻ.

• Giả sử b ≥ 3, khi đó 2b − 1 | 2n − 1 nên 2b − 1 | m2 + 9. Do 2b − 1 ≡ −1 (mod 4) nên 2b − 1

hu
có ước nguyên tố p có dạng 4k + 3. Ta có p | m2 + 9 nên −9 là chính phương môđun p. Suy
ra
     p − 1  2 p−1
−9 −1 9 3
1= = = (−1) 2 = (−1) 2 .

T
p p p p
p−1
Do đó là số chẵn, hay p ≡ 1 (mod 4) (vô lí).
2 t
• b = 1, ta có n = 2a nên
Tấ
a a−1
2n − 1 = 22 − 1 = (2 − 1)(2 + 1)(22 + 1) · · · (22 − 1).
n m
Do (22 + 1, 22 + 1) = 1 với m 6= n, nên theo định lí thặng dư Trung hoa tồn tại x0 thỏa
mãn hệ
n


2
x ≡ 2 (mod 2 + 1)

yễ


2 22

x ≡ 2 (mod 2 + 1)



2 3
x ≡ 22 (mod 22 + 1)

gu

·········





x ≡ 22a−2 (mod 22a−1 + 1)

Suy ra
N

 k−1 2 k k
x20 + 1 ≡ 22 + 1 ≡ 22 + 1 ≡ 0 (mod 22 + 1) ∀k = 1, 2, . . . , a − 1.

Đặt m = 3x0 ta có
k
22 + 1 | 9(x20 + 1) = m2 + 9 ∀k = 0,1, . . . , a − 1.

Suy ra 2n − 1 | m2 + 9.


Bài 4.18 (Hàn Quốc 1999). Tìm tất cả các số tự nhiên n thỏa mãn 2n − 1 chia hết cho 3 và
tồn tại số tự nhiên m sao cho
2n − 1
| 4m2 + 1.
3

Lời giải.

52
4. ĐỒNG DƯ

Ta có
2n − 1 ≡ (−1)n − 1 (mod 3).
Suy ra n là số chẵn, do đó n = 2a · b với b là số tự nhiên lẻ và a là số nguyên dương.
Nếu b ≥ 3 thì ta có
2u − 1 | 2n − 1 | 4m2 + 1.
Hơn nữa 2u − 1 ≡ −1 (mod 4), nên 2u − 1 có một ước nguyên tố p có dạng 4k + 3. Khi đó
p | 4m2 + 1 nên p | 2m và p | 1, điều này vô lí. Do đó b = 1 hay n = 2a . Khi đó
2n − 1 2 a−1
= (22 + 1)(22 + 1) · · · (22 + 1).
3
Gọi x0 là nghiệm của hệ đồng dư


 x ≡ 0 (mod 2)
x ≡ 2 (mod 22 + 1)





x ≡ 22 (mod 222 + 1)

hu

2 3


 x ≡ 22 (mod 22 + 1)

·········

T


a−2 a−1

x ≡ 22 (mod 22 + 1).

. x0
Ta có x0 chẵn và x20 + 1 .. 2n − 1. Đặt m = , ta có
t
2
Tấ
2n − 1 | x20 + 1 = (2m)2 + 1 = 4m2 + 1.


Bài 4.19. Cho S = {p1 , . . . ,pr } là tập r số nguyên tố phân biệt, và P (x) là đa thức hệ số nguyên
n

sao cho với mọi n đều tồn tại pi trong S sao cho pi | P (n) .Chứng minh rằng tồn tại i sao cho
pi | P (n), ∀n ∈ N .
yễ

Lời giải.
Giả sử với mỗi pi trong S tồn tại ai sao cho p 6 | P (ai ). Theo định lí thặng dư Trung hoa, tồn
tại x0 thỏa mãn hệ phương trình đồng dư x ≡ ai (mod pi ). Khi đó, không tồn tại p nào trong S
gu

để p | P (x0 ). Điều này trái với giả thiết đề bài.


Vậy bài toán được chứng minh. 
Bài 4.20 (VMO 2008). Đặt m = 20072008 . Hỏi có tất cả bao nhiêu số tự nhiên n mà n < m
N

và n(2n + 1)(5n + 2) chia hết cho m?


Lời giải.
Ta có m = a·b với a = 34016 , b = 2232008 . Hơn nữa (n, 2n+1) = 1; (n, 5n+2) = (n, 2) = 1 hoặc 2;
(2n + 1, 5n + 2) = (2n + 1, n) = 1. Do đó m | n(2n + 1)(5n + 2) khi và chỉ khi mọt trong các
trường hợp sau xảy ra
.
1) n .. m.
.
2) 2n + 1 .. m.
.
3) 5n + 2 .. m.

n ... a

4) .
..
2n + 1 . b

53
4. ĐỒNG DƯ

n ... b

5) .
..
2n + 1 .a

n ... a

6) .
..
5n + 2 .b

n ... b

7) .
..
5n + 2 .a

..

2n + 1 .a
8) .
..
5n + 2 .b

..

2n + 1 .b

hu
9) .
..
5n + 2 .a

Theo định lí thặng dư Trung hoa, mỗi trường hợp trên có duy nhất một số tự nhiên n < m. Do

T
đó có tất cả 9 số tự nhiên n thỏa yêu cầu bài toán. 
Bài 4.21 (Trường hè toán học 2015). Ta gọi một số là lũy thừa đúng nếu nó có dạng am
với a,m nguyên lớn hơn 1. Tìm tất cả các số nguyên dương n lớn hơn 1 sao cho tồn tại
t
các số nguyên dương b1 ,b2 , . . . ,bn không đồng thời bằng nhau để với mọi k nguyên dương thì
Tấ
(b1 + k) (b2 + k) · · · (bn + k) là lũy thừa đúng?
Lời giải.
Ta chứng minh mọi hợp số n thỏa mãn yêu cầu bài toán.
Điều kiện đủ. Giả sử n là hợp số, n = rs với r > 1,s > 1. Ta chọn các số như sau
n

b1 = b2 = . . . = br = 1; br+1 = br+2 = . . . = bn = 2.
yễ

Khi đó, với mọi k ta có (b1 + k) (b2 + k) · · · (bn + k) = (k + 1)r (k + 2)(s−1)r là lũy thừa đúng bậc
r.
gu

Điều kiện cần. Ta chứng minh nếu n nguyên tố thì n không thỏa mãn.
Thật vậy, giả sử tồn tại bộ số (b1 ,b2 , . . . ,bn ) thỏa mãn điều kiện đề bài, có thể giả sử b1 ,b2 , . . . ,bt
là các số đôi một phân biệt, còn mỗi một trong các số bt+1 ,bt+2 , . . . ,bn là sự lặp lại của các số
trước, ta có ngay t > 1 vì các số không đồng thời bằng nhau.
N

Giả sử trong các số b1 ,b2 , . . . ,bn có si số bằng bi , trong đó

1 ≤ i ≤ t và s1 + s2 + · · · + st = n.

Theo định lý phần dư Trung Hoa thì với mọi a1 ,a2 , . . . ,at nguyên tố cùng nhau và mọi số nguyên
không âm r1 ,r2 , . . . ,rt với 0 ≤ ri < ai ,i = 1,t, tồn tại số nguyên dương m sao cho trong phép chia
cho a1 ,a2 , . . . ,at cho số dư tương ứng là r1 ,r2 , . . . ,rt .
Xét t số nguyên tố phân biệt p1 ,p2 , . . . ,pt , mỗi số đều lớn hơn tất cả các số bi và đặt ai = p2i ,ri =
pi − bi với i = 1,t. Các số p2i đôi một nguyên tố cùng nhau và 0 < ri < pi < p2i nên điều kiện của
định lý phần dư Trung Hoa được thỏa mãn.
Xét số nguyên dương m thỏa mãn điều kiện m chia ai dư ri , ta sẽ chứng minh rằng nếu
(b1 + m) (b2 + m) · · · (bn + m) = uv thì v = 1.
Xét chỉ số i bất kỳ với 1 ≤ i ≤ t. Số bi + m khi chia cho p2i cho số dư ri + bi = pi nên suy ra
bi + m chia hết cho pi nhưng không chia hết cho p2i .
Với j 6= i và 1 ≤ j ≤ t, ta có 0 < |bi − bj | < pi nên suy ra bj + m không chia hết cho pi .
Do đó trong phân tích tiêu chuẩn của (b1 + m) (b2 + m) · · · (bn + m), mỗi số pi sẽ có mặt với số

54
4. ĐỒNG DƯ

mũ si . Vì tích này bằng uv nên v là ước số của tất cả các si và cũng là ước của tổng của chúng,
tức là n. Ngoài ra n là số nguyên tố và v < n nên v = 1. Do vậy số n không thỏa mãn. Bài toán
được chứng minh hoàn toàn.
Nhận xét. Đây là một ứng dụng hay của định lý thặng dư Trung Hoa. Bước đầu tiên kiểm tra
với hợp số thì khá rõ, đến số nguyên tố thì cần xử lý nhiều hơn, khai thác tính có nghiệm của
các hệ phương trình đồng dư để chỉ ra phản ví dụ.

Bài 4.22. Cho số nguyên dương n. Chứng minh rằng tồn tại vô hạn cặp số nguyên dương (a,b)
với a,b > n sao cho
n
Y n
Y n
Y
(a + i) | b (b + 2016) ; (a + i) 6 | b; (a + i) 6 | (b + 2016) .
i=1 i=1 i=1

Lời giải.
Ta sẽ chứng minh bài toán tổng quát sau

hu
Cho trước các số nguyên dương m,n,k. Gọi k1 ,k2 , . . . ,km là các số nguyên dương.
Chứng minh rằng tồn tại vô hạn cặp số nguyên dương (a,b) sao cho
m
Y m
Y m
Y
(a + ki ) | b (b + k) nhưng (a + ki ) 6 | b và (a + ki ) 6 | (b + k) .

T
k=1 k=1 k=1

Chứng minh. Đặt k < p1 < p2 < · · · < pm là m số nguyên tố phân biệt và kí hiệu
t m
Y
M= (a + ki ).
Tấ
k=1

Theo định lý phần dư Trung Hoa, tồn tại vô hạn số nguyên dương a > n sao cho
a ≡ −ki (mod pi ), ∀i = 1,m.
n

Do đó M ≡ 0 (mod p1 p2 . . . pm ). Ta viết M = pα1 1 · pα2 2 · · · pαmm · q1β1 · q2β2 · · · qsβs với αi ≥ 1; i = 1,m
yễ

và βj ≥ 1; j = 1,s và q1 ,q2 , . . . ,qs là s ước nguyên tố của M khác với p1 ,p2 , . . . ,pm .
Theo định lý phần dư Trung Hoa, tồn tại vô hạn số nguyên dương b > m sao cho
b ≡ 0 (mod M )

gu

pαmm
b ≡ −k (mod pαmm ).

N

Từ đó suy ra M 6 | b; M 6 | b + k và M | b(b + k). Bài toán được chứng minh hoàn toàn. 
Bài 4.23 (Iran TST 2015). Cho dãy b1 < b2 < b3 < . . . là dãy các số tự nhiên mà mỗi số hạng
là tổng của hai số chính phương. Chứng minh rằng tồn tại vô hạn m sao cho bm+1 − bm = 2015.
Lời giải.
Xét 2014 số nguyên tố p1 ,p2 , . . . ,p2014 sao cho pi ≡ 3 (mod 4),∀i
( = 1,2014.
x ≡ pi − i (mod p2i )
Theo định lý phần dư Trung Hoa, tồn tại vô hạn x sao cho:
x ≡ 2 (mod 8).
Khi đó x = 2 (4k + 1) và x + 2015 = 4h + 1. Do các số có dạng 4k + 1 đều viết được thành tổng
của hai số chính phương nên tồn tại m để bm+1 = x + 2015.
x
Ta có = a2 + b2 ⇒ x = (a − b)2 + (a + b)2 , suy ra tồn tại h để b` = x.
2  ..
x + i.pi


Ta sẽ chứng minh ` = m. Thật vậy do x + i 6 ...p2 suy ra không tồn tại u,v mà x + i =

 i
pi ≡ 3 (mod 4)

2 2
u + v . Do đó ` = m, tức là bm+1 − bm = 2015. 

55
4. ĐỒNG DƯ

Bài 4.24 (APMO 2011). Chứng minh rằng với mỗi k nguyên dương, k ≥ 2, luôn tồn tại một
a1 a2 an
cấp số cộng , , . . . , các số hữu tỷ, trong đó ai ,bi ∈ Z+ , (ai ,bi ) = 1 và các số a1 ,b1 ,a2 ,b2 , . . . ,an ,bn
b1 b2 bn
đều phân biệt.
Lời giải.
Gọi p1 ,p2 , . . . ,pk là k số nguyên tố phân biệt sao cho k < pk < pk−1 < · · · < p1 .
Đặt N = p1 p2 . . . pk . Theo định lý phần dư Trung Hoa, tồn tại x nguyên dương sao cho

x ≡ −i (mod pi ) với mọi i = 1,k và x > N 2 .


x+1 x+2 x+k
Xét dãy gồm k số sau: , ,..., . Hiển nhiên dãy này là một cấp số cộng.
N N N
.
Ta có ngay, với mỗi i = 1,k thì x + i .. pi .
. .
Nếu tồn tại j 6= i mà x + i .. pj thì |i − j| .. pj . Tuy nhiên pj > k > |i − j|, nên ta có điều mâu
thuẫn.
x+i N
Do đó x + i chia hết cho pi nhưng không chia hết cho pj với j 6= i. Đặt ai = ; bi =
pi pi

hu
x+i ai
với i = 1,k. Khi đó = và (ai ,bi ) = 1. Cuối cùng ta còn phải chứng minh các số
N bi
a1 ,b1 ,a2 ,b2 , . . . ,an ,bn phân biệt. Ta có x > N 2 suy ra

T
x+i N2 N
ai = > >N > = bj với mọi i,j.
pi pi pj

Rõ ràng các số bi phân biệt. Ta có aj =


x+j
t>
x+i
>
x+i
= ai ,∀i < j.
Tấ
pj pj pi
Vậy a1 ,b1 ,a2 ,b2 , . . . ,an ,bn phân biệt. 
Bài 4.25 (Tạp chí Crux 2015). Cho p nguyên tố lẻ. Chứng minh rằng tồn tại một số tự nhiên
x sao cho x và 4x đều là căn nguyên thủy modulo p.
n

Lời giải.
Do p nguyên tố nên luôn tồn tại căn nguyên thủy modulo p. Gọi a là một căn nguyên thủy
yễ

modulo p. Khi đó tồn tại số nguyên dương r sao cho ar ≡ 2 (mod p), suy ra a2r ≡ 4 (mod p).
Đặt p1 ,p2 , . . . ,p` là các ước nguyên tố phân biệt của p − 1. Với mỗi 1 ≤ k ≤ `, kí hiệu sk là một
số nguyên thỏa mãn sk 6≡ 0 (mod pk ) và sk 6≡ −2r (mod pk ). Theo định lý phần dư Trung Hoa,
gu

tồn tại một số tự nhiên m sao cho

sk ≡ m (mod pk ), ∀1 ≤ k ≤ `.
N

Khi đó cả m và m + 2r đều không chia hết cho pk , suy ra cả m và m + 2r đều nguyên tố cùng
nhau với p − 1.
Từ đó các số m,2m,3m, . . . ,(p − 2)m đều không chia hết cho p − 1, cũng vậy các số (m + 2r),
2 (m + 2r), 3 (m + 2r) , . . . ,(p − 2) (m + 2r) đều không chia hết cho p − 1. Do a là căn nguyên
thủy modulo p nên ta được

am ,a2m , . . . ,a(p−2)m 6≡ 1 (mod p);


am+2r ,a2(m+2r) , . . . ,a(p−2)(m+2r) 6≡ 1 (mod p).

Từ đó suy ra am và am+2r ≡ 4am (mod p) là căn nguyên thủy modulo p. Do đó số x = am là số


thỏa mãn yêu cầu bài toán. 
Bài 4.26. Tồn tại hay không tập hợp X thoả mãn đồng thời hai điều kiện sau:
i. Tập X gồm 2016 số tự nhiên phân biệt.
ii. Tổng của một số phần tử bất kì trong X đều có dạng luỹ thừa bậc lớn hơn 1 của một số
nguyên dương.

56
4. ĐỒNG DƯ

Lời giải.
Ta sẽ chứng minh tồn tại tập X thoả mãn bài toán. Thật vậy, đặt n = 1 + 2 + 3 + · · · + 2016 và
gọi p1 ,p2 , . . . ,pn(là n số nguyên tố phân biệt nào đó. Với mỗi i ∈ {1,2, . . . ,n}, ta xét hệ phương
x ≡ −1 (mod pi )
trình đồng dư (hệ gồm n phương trình)
x ≡ 0 (mod pj ), 1 ≤ j ≤ n; j 6= i
Theo định lý phần dư Trung Hoa thì với mỗi i ∈ {1,2, . . . ,n}, hệ trên sẽ có nghiệm là ai ∈ Z+ .
Đặt d = 2a2 · 3a3 · 4a4 · · · nan , khi đó các số a2 ,a3 , . . . ,an đều chia hết cho p1 , suy ra

ak = p1 · bk , ∀k = 2,n.
p
Do đó d = 2b2 · 3b3 · 4b4 · · · nbn 1 , suy ra d là một luỹ thừa bậc p1 của một số nguyên dương.
Tương tự thì các số a2 + 1,a3 ,a4 , . . . ,an đều chia hết cho p2 , suy ra 2d = 2a2 +1 · 3a3 · 4a4 · · · nan là
một luỹ thừa bậc p2 của một số nguyên dương.
Cứ như vậy thì các số d,2d,3d, . . . ,2016d là luỹ thừa bậc p1 ,p2 , . . . ,pn tương ứng.
Xét tập X = {d,2d,3d, . . . ,2016d}, khi đó ta thấy ngay tập X thoả mãn các điều kiện của bài
toán. 

hu
Bài 4.27. Cho tập S = {a1 ,a2 , . . . ,an } gồm các số nguyên dương và đa thức P (x) ∈ Z [x]. Biết
rằng với mọi số nguyên dương k đều tồn tại một chỉ số i ∈ {1,2, . . . ,n} sao cho ai | P (k). Chứng
minh rằng tồn tại một chỉ số i0 sao cho ai0 | P (k) với mọi số nguyên dương k.

T
Lời giải.
Giả sử với mỗi i đều tồn tại một số nguyên dương bi sao cho ai 6 | P (bi ). Gọi P (S) = {p1 ,p2 , . . . ,ps }
là tập các ước nguyên tố của các phần tử của S và mi là số nguyên dương nhỏ nhất sao cho tồn
t
mi
tại một số nguyên dương ki thỏa  mãn pi 6 | Pm(k i ) Theo định lý phần dư Trung Hoa thì tồn tại
Tấ

 K ≡ k1 (mod p1 ) 1

K ≡ k (mod pm2 )

2 2
số nguyên dương K sao cho

 . . .

K ≡ ks (mod pm s ).
s

n

mi
Ta có pi 6 | P (K) với mọi i = 1,2, . . . ,s. Hơn nữa, từ giả thiết phản chứng suy ra với mỗi i đều
ri
tồn tại bi sao cho ai 6 | P (bi ) nên phải có một ước nguyên tố pij nào đó của ai sao cho pij j k ai
yễ

ri mi ri
nhưng pij j k P (bi ). Ta có ngay mij ≤ rij , mà pi j 6 | P (K) nên pi j 6 | P (K), suy ra ai 6 | P (K),
với mọi i = 1,2, . . . ,n, mẫu thuẫn giả thiết. Bài toán được chứng minh hoàn toàn. Ta đưa ra
gu

thêm một bài toán tương tự. 


Bài 4.28. Cho tập S = {a1 ,a2 , . . . ,an } gồm các số nguyên. Chứng minh rằng tồn tại số nguyên
b sao cho tập bS = {ba1 ,ba2 , . . . ,ban } chứa toàn những lũy thừa bậc lớn hơn 1 của một số nguyên
N

nào đó.
Lời giải.
Gọi P (S) = {p1 ,p2 , . . . ,pk } là tập các ước nguyên tố của các phần tử của S. Khi đó với mọi
r r ri
ai ∈ S đều có dạng ai = p1i1 · p2i2 · · · pk k , trong đó rij có thể bằng 0. Ta sẽ chứng minh tồn tại
một số nguyên b dạng b = ps11 · ps22 · · · pskk sao cho bai là lũy thừa với số mũ lớn hơn 1. Điều này
tương đương với tồn tại số nguyên tố qi sao cho qi | sj + rij ,với mọi i = 1,2, . . . ,k. Thật vậy, chọn
q1 ,q2 , . . . ,qk là các số nguyên tố bất kì. Theo định lý phần dư Trung Hoa tồn tại số s1 thỏa mãn
hệ đồng dư 

 s1 + r11 ≡ 0 (mod q1 )

s + r ≡ 0 (mod q )
1 21 2

 . . .

s1 + rk1 ≡ 0 (mod qk ).

Khi đó, qi | s1 + ri1 với mọi i = 1,2, . . . ,k. Tương tự, tồn tại s2 ,s3 , . . . ,sk thỏa mãn các điều kiện
tương tự. Do đó số b = ps11 · ps22 · · · pskk thỏa mãn yêu cầu bài toán. 

57
5. HÀM SỐ HỌC

§5. Hàm số học


I. Hàm phần nguyên
Định nghĩa 1. Phần nguyên của số thực x là số nguyên lớn nhất không vượt quá x. Kí hiệu
[x]. Vậy [x] là số nguyên duy nhất có tính chất

[x] ≤ x < [x] + 1.

Hiệu x − [x] được gọi là phần lẻ (phần thập phân) của x, kí hiệu {x}.
Ví dụ: [2,1] = 2, {2,1} = 0,1; [−3,9] = −4, {−3,9} = 0,1.
Từ định nghĩa ta có một số tính chất cơ bản sau:
Định lí 1.

a) Nếu x ≥ y thì [x] ≥ [y], hay hàm phần nguyên không giảm.

hu
b) [x + n] = [x] + n với mọi n ∈ Z.

c) [x + y] ≥ [x] + [y].

T
d) [2x] + [2y] ≥ [x] + [y] + [x + y].
  h i
[x] x
e) = .
d d t
Tấ
f) Cho x là số thực dương vàhd ilà số nguyên dương. Khi đó số các số nguyên dương là bội của
x
d và không vượt quá x là .
d
Chứng minh. Hai tính chất a) và b) ta dễ dàng suy ra từ định nghịa.
n

c) Đặt n = [x], m = [y] ta có x = n + {x}, y = m + {y}. Khi đó


yễ

[x + y] = [m + n + {x} + {y}] = m + n + [{x} + {y}] ≥ m + n = [x] + [y].

d) Ta có
gu

[2x] + [2y] = 2m + 2n + [2{x}] + [2{y}],



[x] + [y] + [x + y] = 2m + 2n + [{x} + {y}].
N

Do đó ta chứng minh
[2{x}] + [2{y}] ≥ [{x} + {y}]. (1)
1
• Nếu {x}, {y} < thì V T (1) = V P (1) = 0.
2
1 1
• Nếu {x} ≥ hoặc {y} ≥ thì ta có
2 2
[2{x}] + [2{y}] ≥ 1 ≥ [{x} + {y}],

nên (1) đúng.


hxi
e) Đặt m = , ta có
d
 
x [x]
m ≤ < m + 1 ⇒ md ≤ x < d(m + 1) ⇒ md ≤ [x] < (m + 1)d ⇒ m ≤ < m + 1,
d d

58
5. HÀM SỐ HỌC
 
[x]
hay = m.
d
f) Giả sử có m số là bội của d không vượt quá x là d, 2d, . . . , md. Do đó ta có
x hxi
md ≤ x < (m + 1)d ⇒ m ≤ < m + 1 ⇒ m = .
d d

Định lí 2. Trong sự phân tích số n! ra thừa số nguyên tố

n! = pα1 1 pα2 2 · · · pαk


k , αi > 0

thì số mũ αi của pi nào đó sẽ là


     
n n n
αi = + 2 + ··· + k ,
pi pi pi

với n < pk+1 . Hay ta có thể viết

hu
∞  
X n
αi = .
k=1
pki

Chứng minh: Giả sử p là một ước của n!.

T
Ta có
  n  
" #

n n
n! = 1.2 . . . p.(p + 1) . . . 2p . . . 3p . . . .
t p...n = p p !q = pm .m!q
p p
Tấ
 
n
với m = và (p,q) = 1. Tương tự
p
m#  
"

m 0
m! = p p !q
n

p
yễ

với (p, q 0 ) = 1.
Suy ra
n# "m#  
"
n
" # "
n# 
m + n
gu

n! = p p p p !qq 0 = p p p2 !qq 0 ,
p p 2

với (p, qq 0 ) = 1.
N

Cứ tiếp tục như thế ta thu được số mũ của p:


     
n n n
α= + 2 + ... + k .
p p p

√
n

Ví dụ 5.1. Tìm tất cả các số nguyên dương n sao cho 111 chia hết 111.

Lời giải.
Các ước nguyên dương của 111 là 1, 3, 37, 111. Như thế chúng ta có các trường hợp sau
√ 
• n 111 = 1 hay 1 ≤ 111 < 2n , ta có n ≥ 7.
√ 
• n 111 = 3 hay 3n ≤ 111 < 4n , ta có n = 4.
√ 
• n 111 = 37 hay 37n ≤ 111 < 38n , điều này không thể xảy ra.
√ 
• n 111 = 111 hay n = 1.

59
5. HÀM SỐ HỌC

Bởi vậy n = 1, n = 4 hoặc n ≥ 7.




Ví dụ 5.2. Giải phương trình trên tập số thực: [x [x]] = 1.

Lời giải.
Từ định nghĩa của [x [x]] = 1, ta có:
1 ≤ x [x] < 2.
Ta xét các trường hợp sau

• x ∈ (−∞; −1). Thì [x] ≤ −2 và x [x] > 2 điều này dẫn tới mâu thuẫn.

• x = −1 ⇒ [x] = −1. Thì x [x] = 1 nên x = −1 là một lời giải.

• x ∈ (−1; 0). Chúng ta có [x] = −1 nên x [x] = −x < 1, (loại).

• x ∈ [0; 1), thì [x] = 0 và x [x] = 0 < 1, nên chúng ta không có lời giải trong trường hợp này.

hu
• x ∈ [1; 2) chúng ta có [x] = 1 và x [x] = x thoả bài toán.

• x ≥ 2 thì [x] ≥ 2 và x [x] ≥ 4 mâu thuẫn với yêu cầu đề bài.

T
Vì vậy x ∈ {−1} ∪ [1; 2).
t 

Ví dụ 5.3. Chứng minh rằng


Tấ
h√ √ i h√ i
n+ n+1 = 4n + 2 .

với mọi số nguyên dương n.


n

Lời giải.
yễ

Ta chứng minh √ √ √
n+ n+1≤ 4n + 2. (1)
gu

Thật vậy
p
(1) ⇔ 2n + 1 + 2 n(n + 1) ≤ 4n + 2
p
⇔ 2 n(n + 1) ≤ 2n + 1
N

⇔ 4n2 + 4n ≤ 4n2 + 4n + 1 luôn đúng.

Suy ra h√ √ i h√ i
n+ n+1 ≤ 4n + 2 .
√ √  √ 
Giả sử n+ n+1 < 4n + 2 , khi đó tồn tại số nguyên m sao cho
√ √ √
n + n + 1 < m ≤ 4n + 2
p
⇔2 n(n + 1) < m2 − (2n + 1) ≤ 2n + 1
2
⇔ (2n + 1)2 − 1 < m2 − (2n + 1) ≤ (2n + 1)2 .

Suy ra 2
m2 − (2n + 1) = (2n + 1)2 ⇒ m2 = 2(2n + 1). (2)
Do m2 ≡ 0, 1 (mod 4) và 2(2n + 1) ≡ 2 (mod 4) nên (2) vô lí. 

60
5. HÀM SỐ HỌC

Ví dụ 5.4. Chứng minh rằng


.
(2m)! · (2n)! .. (m + n)! · n! · m!.

với mọi số nguyên dương m, n.

Lời giải.
Xét p là số nguyên tố bất kì, khi đó số mũ của p trong phân tích tiêu chuẩn của (m + n)! · n! · m!
là ∞      
X m+n m n
α= + +
k=1
pk pk pk

và số mũ cảu p trong phân tích tiêu chuẩn của (2n)! · (2m)! là


∞    
X 2n 2m
β= + .
pk pk

hu
k=1


[2m] + [2n] ≥ [m] + [n] + [m + n],

T
nên ta có β ≥ α, hay m! · n! · (m + n)! là ước của (2n)! · (2m)!. 

Ví dụ 5.5. Cho m, n là các số nguyên dương sao cho m không có ước nguyên tố nhỏ hơn
t
hoặc bằng n. Chứng minh rằng
Tấ
a = (m − 1)(m2 − 1) · · · (mn−1 − 1)

chia hết cho n!.


n

Lời giải.
Xét p là số nguyên tố không vượt quá n. Khi đó (m, p) = 1 nên theo định lí Phecma nhỏ ta có
yễ

.
mk(p−1) − 1 .. p.
gu

 
n−1
Mà trong tập {1, 2, . . . , n − 1} có chia hết cho p − 1. Do đó, số mũ của p trong phân
p−1
tích tiêu chuẩn của a là
N

 
n−1
α= .
p−1
Mặt khác, số mũ của p trong phân tích tiêu chuẩn của n! là
r   r
X n X n n
β= ≤ < .
k=1
pk k=1
p k p−1

Ta chứng minh α ≥ β.
n−1
Đặt t = , ta có n − 1 = t(p − 1) + s, 0 ≤ s < p − 1. Khi đó
p−1
n s+1
=t+ ≤ t + 1.
p−1 p−1

Suy ra α < t + 1 nên α ≥ t = β. 

61
5. HÀM SỐ HỌC

Ví dụ 5.6. Cho p là số nguyên tố lẻ và q là số nguyên không chia hết cho p. Nếu f : Z∗+ →
R là một hàm thoả
f (k)
i) không là số nguyên với k = 1,2, . . . ,p − 1;
p
ii) f (k) + f (p − k) là số nguyên chia hết cho p với mọi k = 1,2, . . . ,p − 1.

Chứng minh rằng


p−1   p−1
X q pX p−1
f (k) = f (k) − . (1)
k=1
p q k=1 2

Lời giải.
Từ ii) ta có
qf (k) qf (p − k)
+ ∈ Z. (2)

hu
p p
qf (k)
Và từ (1), ta có ∈
/ Z và
p

T
qf (p − k)

/ Z, k = 1,2, . . . ,p − 1.
p

Do đó
t
Tấ
   
qf (k) qf (p − k)
0< + < 2.
p p
Nhưng từ (2), ta có    
qf (k) qf (p − k)
+ ∈ Z,
n

p p
yễ

nên    
qf (k) qf (p − k)
+ = 1, k = 1,2, . . . ,p − 1.
p p
gu

Lấy tổng hai lần ta có


p−1  
X qf (k) p−1
= .
p 2
N

k=1

Nên
p−1 p−1  
X p X q p−1
f (k) − f (k) = .
k=1
q k=1
p 2

Từ đó ta có đpcm.


II. Số các ước của số tự nhiên


Cho n là số nguyên dương. Kí hiệu d(n) là số các ước nguyên dương của n.
Định lí 3. Giả sử n = pα1 1 · pα2 2 · · · pαk k . Khi đó

d(n) = (α1 + 1) (α2 + 1) · · · (αk + 1) .

62
5. HÀM SỐ HỌC

Chứng minh. Trước hết ta có d(ab) = d(a)d(b) với a, b là hai số nguyên dương nguyên tố cùng
nhau.
Hơn nữa d (pα ) = α + 1, nên

d(n) = d (pα1 1 ) d (pα2 2 ) · · · d (pαk k ) = (α1 + 1) (α2 + 1) · · · (αk + 1) .


Định lí 4. Với mọi số nguyên dương n ta có bất đẳng thức d(n) < 2 n.
n
Chứng minh. Nhận thấy, nếu a là ước của n thì cũng là ước của n. Đặt
a
√ √
S1 = {a : a ≤ n và a|n}, S2 = {b : b > n và b|n}.
n n
Ta thấy với a ∈ S1 thì = ∈ S2 và ngược lại, với b ∈ S2 thì ∈ S1 . Từ đó, ta có |S1 | = |S2 |.
√ ¯a √ b
Mà |S1 | < n nên d(n) = 2|S1 | < 2 n.

hu
Ví dụ 5.7. Giả sử tồn tại số tự nhiên n để d(n2 ) = kd(n). Chứng minh rằng k là số lẻ.

Lời giải.
Giả sử n = pα1 1 pα2 2 · · · pαk k , khi đó n2 = p2α 1 2α2
· · · p2α 2

T
1 p2 k . Do đó d(n ) = d(n) tương đương với
k

(2α1 + 1) (2α2 + 1) · · · (2αk + 1) = k (α1 + 1) (α2 + 1) · · · (αk + 1) . (1)

Do vế trái của (1) là số lẻ, nên ta có k là số lẻ.


t 
Tấ
Ví dụ 5.8. Tìm tất cả n sao cho d(n3 ) = 5d(n).

Lời giải.
Giả sử n có phân tích tiêu chuẩn:
n

n = pa11 pa22 · · · pakk


yễ

Từ đó, giả thiết của bài toán trở thành:

(3a1 + 1)(3a2 + 1) · · · (3ak + 1) = 5(a1 + 1)(a2 + 1) · · · (ak + 1)


gu

Tương đương với:


(3a1 + 1)(3a2 + 1) · · · (3ak + 1)
= 5. (∗)
(a1 + 1)(a2 + 1) · · · (ak + 1)
N

Ta thấy rằng với ∀i, i = 1,2, . . . ,k thì:


3ai + 1
≥ 2.
ai + 1

Suy ra nếu k ≥ 3 thì hiển nhiên là V T∗ > 5. Mâu thuẫn.


Do đó k ≤ 2.
Với k = 1 thì ta có thể dễ dàng kiểm tra không tồn tại a1 thỏa mãn.
Với k = 2, phương trình (∗) trở thành:

(2a1 − 1)(2a2 − 1) = 5.

Suy ra: (a1 ,a2 ) = (1,3); (3,1).


Vậy tất cả các số cần tìm có dạng: n = pq 3 với mọi số nguyên tố p, q bất kỳ. 

63
5. HÀM SỐ HỌC

III. Hàm tổng các ước nguyên dương của n


1. Định nghĩa

Định nghĩa 2. Cho n là số nguyên dương. Khi đó, tổng các ước nguyên dương của n được kí
hiệu σ(n) (đọ là Xích-ma của n). Vậy
X
σ(n) = d
d|n

Ví dụ: σ(5) = 1 + 5, σ(6) = 1 + 2 + 3, σ(12) = 1 + 2 + 3 + 4 + 6 + 12.


Định lí 5. Nếu a, b là hai số nguyên tố cùng nhau thì σ(a · b) = σ(a) · σ(b).
Chứng minh. Giả sử a có tất cả các ước là a1 , a2 , . . . ,ak và b có tất cả các ước b1 , b2 , . . . , bt .
Khi đó các ước của ab là ai bj với 1 ≤ i ≤ k, 1 ≤ j ≤ t. Suy ra
k X t k
! t
!
X X X
σ(ab) = ai b j = ai aj = σ(a)σ(b).

hu
i=1 j=1 i=1 j=1

Định lí 6. Giả sử n = pα1 1 pα2 2 · · · pαk k là phân tích tiêu chuẩn của n. Khi đó

T
 α1   α2   αk 
p1 − 1 p2 − 1 pk − 1
σ(n) = ··· .
p1 − 1 tp2 − 1 pk − 1
Tấ
Chứng minh. Theo định lí trên ta có

σ(n) = σ (pα1 1 ) σ (pα2 2 ) · · · σ (pαk k ) .

Hơn nữa
pα+1 − 1
n

σ(pα ) = 1 + p + p2 + · · · + pα = .
p−1
yễ

Suy ra
pα1 1 − 1 pα2 2 − 1 pαk k − 1
    
σ(n) = ··· .
p1 − 1 p2 − 1 pk − 1
gu

Ví dụ 5.9. Chứng minh rằng σ(n) = n + 1 khi và chỉ khi n là số nguyên tố.
N

Lời giải.

• Nếu n là số nguyên tố, khi đó n chỉ có hai ước là 1 và n. Do đó σ(n) = n + 1.

• Giả sử σ(n) = n + 1. Nếu n là hợp số thì n có ít nhất ba ước 1, a và n. Do đó

σ(n) ≥ 1 + a + n > 1 + n.

Điều này trái với giả thiết, do đó n là số nguyên tố.


n
Ví dụ 5.10. Chứng minh rằng σ(n) là số lẻ khi và chỉ khi n là số chính phương hoặc
2
là số chính phương.

Lời giải.

64
5. HÀM SỐ HỌC

• Giả sử n = m2 hoặc n = 2m2 và


n = 2α pα1 1 pα2 2 · · · pαk k ,
trong đó α ∈ N, αi là các số tự nhiên chẵn và pi là các số nguyên tố lẻ. Khi đó
σ(n) = σ (2α ) σ (pα1 1 ) · · · (pαk k ) .
Do σ (2α ) = 2α+1 − 1 là số lẻ và
σ (pαi i ) = 1 + pi + · · · + pαi i
là số lẻ.
• Giả sử σ(n) là số lẻ và n = 2α pα1 1 pα2 2 · · · pαk k là phân tích tiêu chuẩn của n. Do
σ(n) = σ (2α ) σ (pα1 1 ) σ (pα2 2 ) · · · σ (pαk k )
và σ(n) là số lẻ nên σ (pαi i ) là số lẻ, hay αi là số chẵn. Do đó, nếu α là số chẵn thì n là số
n

hu
chính phương, nếu α là số lẻ thì là số chính phương.
2

Định nghĩa 3. Nếu số tự nhiên n thỏa mãn σ(n) = 2n thì n được gọi là số hoàn hảo.

T
Chẳng hạn 6, 28 là các số hoàn hảo.

Ví dụ 5.11. Cho k là số tự nhiên sao cho 2k − 1 là số nguyên tố. Chứng minh rằng
t
n = 2k−1 2k − 1 là số hoàn hảo.
Tấ
Lời giải.
Ta có
 2k − 1
σ(n) = σ 2k−1 σ 2k − 1 = · (2k − 1 + 1) = 2k (2k − 1) = 2n.

2−1
n

Do đó n là số hoàn hảo. 
yễ

Ví dụ 5.12. Chứng minh rằng nếu n là số hoàn hảo chẵn thì

n = 2k 2k+1 − 1 ,

gu

trong đó k ≥ 1 và 2k+1 − 1 là số nguyên tố.


N

Lời giải.
Vì n là số chẵn nên ta viết n = 2a · b với a ≥ 1 và b là số lẻ. Khi đó
σ(n) = σ(2a )σ(b) = (2a+1 − 1)σ(b).
Mà σ(n) = 2n nên ta có
21+a · b = (2a+1 − 1)σ(b). (1)
.
Do 2a+1 − 1 là số lẻ, nên từ (1) ta có σ(b) .. 2a+1 . Do đó σ(b) = t · 2a+1 , khi đó b = (2a+1 − 1)t.
Nếu t > 1 thì
σ(b) = σ(2a+1 − 1)σ(t) ≥ 2a+1 (t + 1) > 2a+1 · t (vô lí).
Do đó t = 1, hay σ(b) = 2a+1 và b = 2a+1 − 1, suy ra b là số nguyên tố.
Vậy n = 2a (2a+1 − 1) với a ≥ 1 và 2a+1 − 1 là số nguyên tố. 

IV. Bài tập


Bài 5.1. Chứng minh rằng với mọi số tự nhiên n, ta luôn có

65
5. HÀM SỐ HỌC
" #  "r #
(n − 1)2 h n i  n2  √

1 3 1
a) + = . (1) b) n+ = n− + .
4 2 4 2 4 2

Lời giải.
a) Ta xét hai trường hợp:
+) n = 2k thì
   2
1 2 n
V T (1) = k (k − 1) + + [k] = k (k − 1) + k = k = = V P (1).
4 4

+) n = 2k + 1 thì
   2
2 1 2 n
V T (1) = k + k + =k +k = = V P (1).
2 4
"r #

 
1 3 1
b) Đặt m = n+ , t= n− + ta có

hu
2 4 2
2

  
1 1 1
m≤ n+ <m+1⇒ m− ≤n< m+

T
2 2 2
1 1
⇒ m2 − m + ≤ n < m2 + m +
t 4 4
⇒ m2 − m + 1 ≤ n ≤ m2 + m.
Tấ
Tương tự ta cũng có t2 − t + 1 ≤ n ≤ t2 + t. Từ đó suy ra m = t.

Bài 5.2. Giải phương trình 3x2 − 10[x] + 3 = 0.
n

Lời giải.
Ta có 10[x] = 3x2 + 3x. Mà [x] ≤ x, nên ta có
yễ

1
3x2 + 3 ≤ 10x ⇔ ≤ x ≤ 3 ⇒ 1 ≤ [x] ≤ 3.
3
gu

r
2 7
• [x] = 1, ta có 3x − 7 = 0 ⇔ x = .
3
r
17
N

2
• [x] = 2, ta có 3x − 17 = 0 ⇔ x = .
3
• [x] = 3, ta có 3x2 − 27 = 0 ⇔ x = 3.

p
Bài 5.3 (2004 Romanian). Tìm tất cả các số thực x > 1 sao cho [xn ] là số nguyên dươngn

với mọi số nguyên n, n ≥ 2.


Lời giải.
p
Đặt n [xn ] = an , khi đó [xn ] = ann và ann ≤ xn < ann + 1.

Do đó an ≤ x < n ann + 1 nên [x] = an .
Chúng ta thấy, bài toán thoả mãn với mọi số nguyên x,x ≥ 2.
Giả sử có một giá trị của x thoả bài toán và x không phải là số nguyên.
Đặt x = a + α,a ∈ Z,a ≥ 1,0 < α < 1.
Khi đó: an < (a + α)n < an + 1, bởi vậy
 α n 1
1< 1+ < 1 + n ≤ 2. (∗)
a a
66
5. HÀM SỐ HỌC

Mà theo bất đẳng thức Bernoulii, ta có:


 α n α
1+ ≥1+n >2
a a
với n đủ lớn. Dẫn đến sự mâu thuẫn với (*).
Vậy bài toán được giải quyết. 
n h
X √ i √

Bài 5.4. Cho n ∈ N . Tính S = k theo n và [ n].
k=1
Lời giải. h i

Để ý rằng k = a ⇔ a2 ≤ k < (a + 1)2 , nên sẽ có (a + 1)2 − a2 số nguyên dương mà phần

nguyên của căn bậc hai của nó bằng a. Đặt [ n] = p, ta có

S = [1] + [2] + · · · + [n]


= 22 − 12 · 1 + 32 − 22 · 2 + · · · + p2 − (p − 1)2 (p − 1) + n − p2 + 1 p
    

hu
= 22 · 1 + 32 · 2 + · · · + p2 (p − 1) − 13 − 23 − · · · − p3 + (n + 1)p
= (n + 1)p − 12 − 22 − · · · − p2
p(p + 1)(2p + 1)
= (n + 1)p − .

T
6

2
 
k
Bài 5.5. Tìm số hạng phân biệt của dãy số hữu hạn
t 1998
, với k = 1,2, . . . ,1997.
Tấ
Lời giải.
Ta có:
9982 9992
   
= 498 < 499 = .
1998 1998
n

Vì vậy chúng ta chỉ cần tính số hạng phân biệt của dãy khi k = 1,2,...,998 và k = 999,1000, . . . ,1997.
Nhận xét thấy với k = 1,2, . . . ,997 thì
yễ

(k + 1)2 k2 2k + 1
− = < 1,
1998 1998 1998
gu

vì vậy với k = 1,2, . . . ,998 , mỗi số


 2 
9982
 
1
N

= 0,1, . . . ,498 =
1998 1998
 2 
k
xuất hiện ít nhất một lần trong dãy và tổng cộng có 499 số hạng phân biệt.
1998
Với k = 999,1000, . . . ,1997, chúng ta có

(k + 1)2 k2 2k + 1
− = > 1,
1998 1998 1998
k2
 
vì vậy số gồm các số hạng phân biệt ( với k = 999, . . . ,1997) và có tất cả
1998
1997 − 999 + 1 = 999

số.
Vậy có tất cả 1498 số phân biệt.


67
5. HÀM SỐ HỌC

Bài 5.6. Xác định số các số thực a thoả phương trình


hai hai hai
+ + = a.
2 3 5

Lời giải.
hai hai hai
Ta có , , là các số nguyên, a cũng vậy.
2 3 5
Đặt a = 30p + q với p,q là các số nguyên 0 ≤ q < 30. Khi đó:
hai hai hai
+ + =a
2 h q3 i h q5 i h q i
⇔ 31p + + + = 30p + q
2h i 3h i 5h i
q q q
⇔p=q− − − .
2 3 5
Như vậy, mỗi giá trị của q cho ta một giá trị của p (tương ứng một giá trị của a). Vì vậy có tất
cả 30 giá trị của a thoả yêu cầu bài toán. 

hu
Bài 5.7 (Hermite). Cho n là số nguyên dương. Chứng minh rằng với mỗi số thực x ta có
   
1 n−1
[nx] = [x] + x + + ... + x + .

T
n n

Lời giải.    t

1 n−1
Đặt f (x) = [x] + x + + ... + x + − [nx] . Ta có:
Tấ
n n
          
1 1 1 1 1 n−1 1
f x+ = x+ + x+ + + ... + x + + − n x+
n n n n n n n
   
1 2
n

= x+ + x+ + . . . + [x + 1] − [nx + 1] .
n n
yễ

Áp dụng [x + k] = [x] + k với k ∈ Z, ta được


 
1
f x+ = f (x) với ∀x ∈ R.
gu

n
1
Do đó f là hàm tuần hoàn với chu kì .
n
N

1
Vì vậy ta chỉ cần nghiên cứu f (x) trong 0 ≤ x < .
  n
1
Nhưng f (x) = 0 với mọi x ∈ 0; , nên f (x) = 0, ∀x ∈ R và chứng minh hoàn tất. 
n
Bài 5.8 (Gauss). Cho p, q là hai số nguyên tố cùng nhau. Khi đó
p−1  
X q (p − 1)(q − 1)
k = .
k=1
p 2

Lời giải.
Áp dụng kết quả ví dụ 5.6 với f (x) = x ta có
p−1  
X q q (p − 1)p p − 1 (p − 1)(q − 1)
k = − = .
k=1
p p 2 2 2

68
5. HÀM SỐ HỌC

Bài 5.9 (2002 German Mathematical Olympiad). Cho p là số nguyên tố lẻ. Chứng minh
rằng
p−1  3 
X k (p − 1)(p − 2)(p + 1)
= .
k=1
p 4

Lời giải.
Áp dụng kết quả ví dụ 5.6 với f (x) = x3 ta có
p−1 
q (p − 1)2 p2 p − 1 (p − 1)(p2 q − pq − 2)

3q
X
k = − = .
k=1
p p 4 2 4

Cho q = 1 ta có điều phải chứng minh. 


   
x−p −x − 1
Bài 5.10. Tính các giá trị có thể nhận của S = + với x ∈ R, p ∈ Z∗ .
p p
Lời giải.

hu
Đặt phần lẻ của x là a, nghĩa là a = x − [x], a ∈ [0,1). Ta có

x = [x] + a = kp + r + a, với k,r ∈ Z, 0 ≤ r < |p|.

T
Khi đó
   
kp + r + a − p −kp − r − a − 1
S= +
p
t p
Tấ
   
r+a −r − a − 1
=k−1+ −k+
p p
   
r+a −r − a − 1
= −1 + + .
p p
n

Chú ý các điều kiện của r và a, ta có thể đánh giá các phân số trong kí hiệu phần nguyên theo
yễ

3 trường hợp sau


 
r+a
• Trường hợp 1: p > 0. Khi đó r + a < p nên = 0, và
gu

p
  (
−r − a − 1 −2 nếu r = p − 1, a > 0
=
p −1 nếu ngược lại.
N

Suy ra S = −3 hoặc S = −2.

• Trường hợp 2: p = −1. Khi đó r = 0 nên


(
0 nếu a = 0
S = −1 + [−a] + [a + 1] =
−1 nếu a ∈ (0,1).

r+a
• Trường hợp 3: p < −1. Nếu r = a = 0 thì S = −1. Ngược lại −1 < < 0 nên
  p
r+a
= −1, suy ra
p
  (
r+a+1 −1 nếu r = −p − 1,a > 0
S = −1 − 1 + =
−p −2 nếu ngược lại.

69
5. HÀM SỐ HỌC

Vậy S ∈ {−3, − 2, − 1,0}. 


Bài 5.11. Cho p, q nguyên dương và nguyên tố cùng nhau. Chứng minh
           
p 2p (q − 1)p q 2q (p − 1)q (p − 1)(q − 1)
+ + ··· + = + + ··· + = . (1)
q q q p p p 2

Lời giải.
Giả sử p = mq + r, 0 ≤ r < q. Khi đó với mỗi k = 1,2, . . . , q − 1 ta có
       
kp (q − k)p kr kr
+ = km + + p − km −
q q q q
   
kr kr
= km + + p − km + −
q q
   
kr kr
=p+ + − = p − 1.
q q

hu
Do đó, nếu đặt S là giá trị của vế trái của (1) thì
q−1    
X kr (q − k)r
2S = + = (p − 1)(q − 1).

T
i=1
q q

Từ đó ta có điều phải chứng minh. t 


Bài 5.12. Số nguyên dương n được gọi là đẹp nếu nó là hợp số và không chia hết cho 2 hoặc 3
Tấ
hoặc 5. Hỏi có bao nhiêu số đẹp nhỏ hơn 1000?
Lời giải.
Gọi A, B, C lần lượt là tập các số nguyên dương nhỏ hơn 1000 chia hết cho 2, chia hết cho 3 và
chia hết cho 5. Dễ thấy
n

     
999 999 999
|A| = = 499, |B| = = 333, |C| = = 199,
2 3 5
yễ

     
999 999 999
|A ∩ B = = 166, |B ∩ C| = = 66, |C ∩ A| = = 99,
6 15 10
gu

 
999
|A ∩ B ∩ C| = = 33.
30

Gọi X là các số nguyên dương là bội của 2 hoặc 3 hoặc 5 và nhỏ hơn 1000 thì X = A ∪ B ∪ C,
N

nên
|X| = |A| + |B| + |C| − |A ∩ B| − |B ∩ C| − |C ∩ A| + |A ∩ B ∩ C| = 733.
Ngoài ra, trong các số nguyên dương nhỏ hơn 1000 còn cso 165 số nguyên tố khác 2, 3, 5, và số
1 không phải là số đẹp, nên số các số đẹp nhỏ hơn 1000 là

999 − 733 − (165 + 1) = 100.


Bài 5.13. Tìm số nguyên dương n lớn nhất thỏa mãn: trong n số nguyên dương đầu tiên, số các
số chia hết cho 3 bằng số các số chia hết cho 5 hoặc 7.
Lời giải.
hni hn
Trong số nguyên dương đầu tiên, số các số chia hết cho 3, cho 5, cho 7 và cho 35 lần lượt là
ni hni h n i
, , , . Theo giả thiết, ta cần tìm n lớn nhất thỏa mãn
3 5 7 35
hni hni hni h n i
= + − . (1)
3 5 7 35
70
5. HÀM SỐ HỌC

Giả sử n = 35k + r, k ∈ N, r ∈ {0,1,2, . . . , 34}. Khi đó, phương trình (1) tương đương
  h
2k + r ri h ri h ri
11k + = 7k + + 5k + − k+ ,
3 5 7 35

hay   h i h i
2k + r r r
= + . (2)
3 5 7
Ta có   h i h i
2k + r 2k + r r r r r 12r
−1< = + ≤ + = ,
3 3 5 7 5 7 35
r
suy ra 2k < + 3 < 4, do đó k < 2. Do n lớn nhất nên ta chọn k = 1, hay n = 35 + r. Thay
35
lần lượt r = 34,33 . . . ,1 vào phương trình (2), ta thấy r = 30 thỏa mãn.
Vậy n = 65 là số cần tìm. 
Bài 5.14. Giải phương trình nghiệm nguyên dương

hu
 2  2
x y x2 + y 2
+ = + xy. (1)
y x xy

T
Lời giải.
Ta có t
x2 y 2
 2
y2 x2 + y 2 x2 + y 2

x
Tấ
+ ≥ + = + xy > − 1 + xy
y x y x xy xy
⇒ x3 + y 3 > x2 + y 2 − xy + x2 y 2
⇒ x3 + y 3 − xy − x2 y 2 > (x − y)2 > 0
⇒ x2 − y x − y 2 > 0.
 
n

• Trường hợp 1: y > x2 . Khi đó y 2 > x4 > x nên bất phương trình trên được thỏa mãn. Vì
yễ

x, y nguyên dương nên y ≥ x2 + 1.

– Dễ thấy y = x2 + 1 và x = 1 thỏa mãn (1).


gu

– Nếu y = x2 + 1 và x > 1 thì (1) trở thành

x2
  4
x + 2x2 + 1
  4
x + 3x2 + 1
 
N

+ = + x3 + x
x2 + 1 x x (x2 + 1)
   2 
3 1 2x + 1
⇔ 0 + x + 2x =x+ + x3 + x
x x3 + x
 2 
2x + 1
⇔ = 0.
x3 + x

2x2 + 1
Đẳng thức cuối luôn đúng với x > 1, vì 3 < 1 ⇔ x(x − 1)2 > 1.
x +x
 2
2 x
– Nếu y > x + 1 thì = 0, do đó (1) tương đương
y
 2  2
x + y2

y
= xy.
x xy

71
5. HÀM SỐ HỌC

Ta có
y2
 2  2
x + y2 x2 + y 2

y
−1< = + xy < + xy
x x xy xy
⇒ y 3 − xy < x2 + y 2 + x2 y 2
⇒ y 2 y − x2 − 1 − xy < x2 .


Điều này vô lí vì

x2 > y 2 y − x2 − 1 − xy > y 2 − xy = y(y − x) > y ≥ x2 + 2.




Tóm lại trường hợp này cho ta nghiệm (x,y) = (x,x2 + 1) với x nguyên dương.

• Trường hợp 2: y < x2 . Khi đó, từ bất phương trình ban đầu là có x > y 2 . Lập luận tương
tự trường hợp 1 ta có nghiệm (x,y) = (y 2 + 1,y).

hu
Bài 5.15. Cho r nguyên dương thỏa mãn ∀m,n ∈ Z, nếu m | n thì [rm] | [rn]. Chứng minh r là
số nguyên.
Lời giải.

T
Giả sử r không là số nguyên.
 Khi đó ta có thể chọn a nguyên dương đủ lớn để [ar] > 1 và ar
1 1 1 1
không là số nguyên. Đặt k thì k ≤ < k + 1, hay < ar − [ar] ≤ .
ar − [ar] t ar − [ar] k+1 k
Suy ra
Tấ
k+1
1 < (k + 1) (ar − [ar]) ≤ ≤ 2. (1)
k
1
Dấu “=” của (1) xảy ra khi = [ar] = k = 1 hay ar − [ar] = 1 (vô lí). Nói cách khác, (2) không
ar
xảy ra dấu “=”, ta có thể viết lại
n

k+1
yễ

1 < (k + 1) (ar − [ar]) ≤ ≤ 2.


k
⇒ (k + 1) [ar] + 1 < (k + 1)ar < (k + 1) [ar] + 2,
gu

dẫn đến
[(k + 1)ar] = (k + 1) [ar] + 1.
Từ đó suy ra [ar] không chia hết [(k + 1)ar], mâu thuẫn với giả thiết, vì a chia hết (k + 1)a.
N

Vậy r phải là số nguyên. 


Bài 5.16. Tìm số nguyên dương n sao cho d(n) = 6
Lời giải.
Ta có 6 = 6.1 = 3.2
Từ công thức tính d(n) suy ra n có nhiều nhất 2 ước nguyên tố là p, q.
Giả sử n = pa q b , suy ra 1 + a = 2,1 + b = 3 hoặc 1 + a = 6,1 + b = 1
Suy ra dạng của n là pq 2 hoặc p5 với p, q nguyên tố.
Thử lại thấy cả 2 dạng này đều thỏa mãn. 
Bài 5.17. Với số nguyên dương n, kí hiệu d(n) là số các ước nguyên dương của n (kể cả 1 và
chính nó). Xác định tất cả các số nguyên dương n sao cho n = d(n)2 .
Lời giải.
Kí hiệu các số nguyên tố p1 = 2,p2 = 3,...
∞ ∞
p2a
Q Q
Với số chính phương n, ta có n = i
i
và d(n) = (2ai + 1)
i=1 i=1
Suy ra d(n) là số lẻ và từ đó n cũng là số lẻ, suy ra a1 = 0

72
5. HÀM SỐ HỌC

d(n) ∞ (2a + 1)
Q i
Theo điều kiện bài toán, √ = 1 ta có ai =1
n i=1 p i
Bởi bất đẳng thức Bernoulli ta có pai i ≥ (pi − 1) ai + 1 > 2ai + 1 với số nguyên tố pi > 3, là ước
của n.
Áp dụng bất đẳng thức ta được 3a2 ≥ 2a2 + 1, đẳng thức xảy ra khi a2 ∈ {0,1}, các trường hợp
còn lại xảy ra dấu bằng chỉ khi a3 = a5 = ,,, = 0
Suy ra n ∈ {1,9} là các giá trị cần tìm. 
Bài 5.18. Chứng minh rằng với mọi số nguyên dương k lẻ, luôn tồn tại số tự nhiên n để
d(n2 ) = kd(n).
Lời giải.
Đặt n = pα1 1 pα2 2 · · · pαr r . Khi đó

d(n2 ) = kd(n) ⇔ (2α1 + 1) (2α2 + 1) · · · (2αr + 1) = k (α1 + 1) (α2 + 1) · · · (αr + 1) . (1)

Ta chứng minh với k = 2m + 1 luôn tồn tại các số nguyên dương α1 , α2 , . . . , αr thỏa mãn (1)
bằng quy nạp theo m.

hu
• Với m = 1, ta có
(2 · 4 + 1)(2 · 2 + 1)
k=3= .
(4 + 1)(2 + 1)

T
• Giả sử với mọi k = 2m + 1, m < M đều có thể biểu diễn được dưới dạng (1). Ta chứng
minh k = 2M + 1 cũng biểu diễn được dưới dạng (1).
t k+1 k+1
Giả sử k + 1 = 2a · b với b là số tự nhiên lẻ, ta có b = a
≤ < k. Đặt
Tấ
2 2
α1 = 2a b − 20 b − 20
r2 = 2a+1 · b − 21 b − 21
α3 = 2a+2 · b − 22 b − 22
n

·········
yễ

αa = 22a−1 · b − 2a−1 · b − 2a−1 .

Đặt n1 = pα1 1 pα2 2 · · · pαa a . Khi đó ta có


gu

d(n21 ) (2a+1 b − 21 b − 21 + 1)(2a+2 b − 22 b − 22 + 1) · · · (22a b − 2a b − 2a + 1)


k1 = = a
d(n1 ) (2 b − 20 b − 20 + 1)(2a+1 b − 21 b − 21 + 1) · · · (22a−1 b − 2a−1 b − 2a−1 + 1)
22a b − 2a b − 2a + 1 2a b − 1
N

= = .
2a b − b b
d(n22 )
Vì b < k nên theo giả thiết quy nạp, tồn tại n2 = q1β1 q2β2 · · · qrβr để b = . Bằng cách
d(n2 )
chọn pi và qi là các số nguyên tố đôi một khác nhau và n = n1 · n2 ta có

d(n2 ) d(n21 ) d(n22 )


= · = k1 · b = 2a b − 1 = k.
d(n) d(n1 ) d(n2 )

Vậy bài toán được chứng minh. 


Bài 5.19. Tìm tất cả các số nguyên dương n để (d(n))3 = 4n.
Lời giải.
Giả sử n = pα1 1 pα2 2 · · · pαk k · · · trong đó p1 < p2 < · · · < pk < · · · là tất cả các số nguyên tố và
α1 , α2 , . . . , αk , . . . là các số tự nhiên. Ta có

d(n) = (α1 + 1) (α2 + 1) · · · (αk + 1) · · ·

73
5. HÀM SỐ HỌC


4n = 22+α1 pα2 2 · · · pαk k · · · .
Do 4n là lập phương của số tự nhiên nên tồn tại các số tự nhiên β1 , β2 , . . . , βk , . . . để
(
α1 = 1 + 3β1
αi = 3βi ∀ i ≥ 2.

Khi đó
d(n) = (3β1 + 2) (3β2 + 1) · · · (3βk + 1) · · · ,
.
do đó d(n) ≡ 2 (mod 3). Mà 4n = d3 (n) nên n 6 .. 3, hay α2 = 0. Do đó ta có

2 + 3β1 pβ3 3 · pβ4 4 · · · pβkk · · ·


= . (1)
21+β1 (1 + 3β3 )(1 + 3β4 ) · · · (1 + 3βk ) · · ·

Vì pi ≥ 5 với ∀i ≥ 3 nên ta có

hu
pβi i ≥ (1 + 4)βi ≥ 1 + 4βi .
Đẳng thức xảy ra khi βi = 0. Do đó V P (1) ≥ 1, nên

T
2 + 3β1
≥ 1 ⇔ 2 + 3β1 ≥ 21+β1 . (2)
21+β1
Ta có
n n
Cn1
t
Cn2
n(n − 1) n2 + n + 2
Tấ
2 = (1 + 1) ≥ 1 + + =1+n+ = .
2 2
Nên từ (2) ta có
2 + 3β1 ≥ β12 + β1 + 2 ⇔ β12 − 2β1 ≤ 0 ⇔ 0 ≤ β1 ≤ 2.
n

• β1 = 0, khi đó V T (1) = 1 nên βi = 0 ∀ i ≥ 3. Ta có n = 2.


5
yễ

• β1 = 1, khi đó V T (1) = .
4

pβ3 3 52
gu

25 5 5
– Nếu β3 > 1 thì > = > . Do đó V P (1) > nên (1) không xảy
1 + 3β3 1+3·2 7 4 4
ra.
pβ3 3 5
N

– Xét β3 = 1, khi đó = , do đó βi = 0 ∀i ≥ 4. Nên ta có n = 2000.


1 + 3β3 4
• β1 = 2 ta có V T (1) = 1 nên βi = 0 ∀i ≥ 3. Do đó n = 27 = 128.

Vậy n = 2, n = 128, n = 2000 là những số cần tìm. 


Bài 5.20. Tìm n nguyên dương sao cho σ(n) = 12.
Lời giải.
Giả sử n = pa11 pa22 · · · pakk trong khai triển thừa số nguyên tố.
σ(n) = 1 + p1 + p21 + · · · + pa11 1 + p2 + p22 + · · · + pa22 · · · 1 + pk + p2k + · · · + pakk


= 12 = 1 + 11 = (1 + 2) (1 + 3)
Suy ra n = 11 hoặc n = 2.3 = 6. 

Bài 5.21. Cho n là số nguyên dương. Chứng minh rằng nếu n là hợp số thì σ(n) > n + n.
Lời giải. √
Do n là hợp số nên tồn tại số nguyên
√ dương m sao cho n ≤ m ≤ n và m là ước của n.
Suy ra σ(n) ≥ m + 1 + n > n + n. 

74
5. HÀM SỐ HỌC

Bài 5.22. Cho số nguyên dương n, chứng minh σ(1) + σ(2) + ... + σ(n) ≤ n2 (1)
Lời giải. hni
Khi ta viết vế trái của (1) thành tổng các ước thì số d xuất hiện đúng lần.
n n
d
h n i P n
d = n2 , ta có điều phải chứng minh.
P
Suy ra σ(1) + σ(2) + ... + σ(n) = d ≤ 
d=1 d d=1 d

Bài 5.23. Cho n là số nguyên dương thỏa mãn 24|n + 1. Chứng minh rằng tổng tất cả các ước
của n chia hết cho 24.
Lời giải.
.
Theo giả thiêt 24|n + 1, suy ra n là số lẻ, n không chia hết cho 3 (do 24..3 và 1 không chia hết
cho 3) và n không là số chính phương.
P  n P d2 + n
Suy ra σ(n) = d+ = .
d|n d d|n d
√ √
d< n d< n
d là ước(của n, n là số lẻ, n không chia hết cho 3 nên d là số lẻ không chia hết cho 3.
d2 ≡ 1 (mod8)

hu
suy ra 2 ⇒ d2 ≡ 1 (mod24) ⇒ d2 + n ≡ 1 + n ≡ 0 (mod24)
d ≡ 1 (mod3)
d2 + n ..
do (n,24) = 1 ⇒ (d,24) = 1 ⇒ .24.
d

T
Ta có điều phải chứng minh. 
Bài 5.24. Với số nguyên√ dương √ n, kí hiệu σ(n) là tổng tất cả các ước của n (kể cả 1 và chính
nó). Chứng minh k n < σ(n) < 2kn với k là số các ước của n.
Lời giải.
t
Tấ
Xét các ước của n là 1 = d1 < d2 < ... < dk = n và di dk+1−i = n.
n
(do nếu d là ước của n thì cũng là ước của n).
d
Pk Pk d +d
i k+1−i Pk p √
Khi đó S = di = > di dk+1−i = k n (chú ý các số di là phân biệt)
2
n

i=1 i=1 vi=1


k
u k
P uP 2 r
d u di
yễ

i
k S i=1 S2 √
≤ i=1 =
t
d2i , ta có =
P
Đặt S2 = ⇒ S ≤ kS2
i=1 k k k k
S2 k d2 k 1 n 1
gu

P i P P
Hơn nữa 2 = 2
= 2
≤ 2
<2
n i=1 n i=1 dk+1−i j=1 j
√ √ √
Suy ra S ≤ kS2 < 2kn2 < 2kn. Ta có điều phải chứng minh. 
N

Bài 5.25. Chứng minh rằng nếu số nguyên dương lẻ n là số hoàn hảo thì n được khai triển thừa
số nguyên tố dạng n = pa q12b1 q22b2 ...qt2bt với a và p cùng chia 4 dư 1 và t ≥ 2.
Lời giải.
Phân tích số n thành tích các thừa số nguyên tố n = pa11 pa22 ...pakk .
k
(1 + pi + p2i + ... + pai i ) = 2pa11 pa22 ...pakk là số chẵn và
Q
Do n là số hoàn hảo nên σ(n) = 2n ⇔
i=1
không chia hết cho 4, suy ra tồn tại đúng 1 số i sao cho 1 + pi + p2i + ... + pai i ≡ 2 (mod4) Suy ra
ai là số lẻ (nếu ai là số chẵn thì vế trái là tổng của lẻ số lẻ, là 1 số lẻ).
Đặt ai = 2x + 1 với x là số nguyên.
Do p2i ≡ 1 (mod4), viết lại phương trình trên được (x + 1) (p1 + 1) ≡ 2 (mod4), suy ra x là số
chẵn, hay ai ≡ 1 (mod4).
Với i 6= j,1 ≤ j ≤ k, ta có 1 + pj + p2j + ... + paj i ≡ 1 (mod2), suy ra aj là số chẵn.
Viết lại dạng của n, ta được n = pa q12b1 q22b2 ...qt2bt .
Chứng minh t ≥ 2. 
Giả sử t = 1, ta có (1 + p + p2 + ... + pa ) 1 + q + q 2 + ... + q 2b = 2pa q 2b

75
5. HÀM SỐ HỌC

1 1
a+1 2b+1 q−
q − 2b
p −1q −1 p a q p q 53 15
Hay = 2pa q 2b ⇒2= < ≤ = , mâu thuẫn.
p−1 q−1 p−1 q−1 p−1q−1 42 8

Bài 5.26 (Nghệ An). Số nguyên dương m gọi là số hoàn hảo nếu tổng các ước nguyên dương
của nó là 2m. Tìm các số nguyên dương n sao cho 1 + nn là số hoàn hảo.
Lời giải.

• n lẻ, suy ra nn + 1 là số hoàn hảo chẵn. Do đó

nn + 1 = 2p−1 (2p − 1),

với p và 2p − 1 là các số nguyên tố.


Ta thấy n = 1 không thỏa. Với n > 1 thì

(n + 1) nn−1 − nn−2 + · · · − n + 1 = 2p−1 (2p − 1).




hu
Suy ra n + 1 = 2p−1 , khi đó 2n + 1 = 2p − 1. Do đó, ta có

nn + 1 = (n + 1)(2n + 1) = 2n2 + 3n + 1. (1)

T
+) n = 3 thỏa.
+) n > 3, suy ra n ≥ 5 nên
t
nn + 1 ≥ n5 + 1 ≥ 125n2 + 1 > 2n2 + 3n + 1.
Tấ
Suy ra vô lí.

• n chẵn. Giả sử n không chia hết cho 3, ta có


n

nn + 1 ≡ 2 (mod 3),
yễ

suy ra nn + 1 không là số chính phương.


Xét k là một ước của m = nn + 1, ta có
m ..
gu

k+ .3.
k
Suy ra  m  ..
N

X X
2m = k= k+ .3.
√ k
k|m k|m, k< m

. .
Điều này vô lí, do đó n..3. Kết hợp n chẵn, ta có n..6. Đặt a = n 6 , khi đó
n

nn + 1 = a6 + 1 = (a2 + 1)(a4 − a2 + 1).

Do 3 6 |a2 + 1 nên (a2 + 1,a4 − a2 + 1) = 1. Do đó

2m = σ(a2 + 1).σ(a4 − a2 + 1).

Lại có m lẻ nên trong hai số σ(a2 + 1) và σ(a4 − a2 + 1) có đúng 1 số lẻ và một số chẵn.


Mặt khác a2 + 1 và a4 − a2 + 1 là số lẻ nên trong hai số a2 + 1 và a4 − a2 + 1 có một số là
số chính phương. Tuy nhiên

a2 < a2 + 1 < (a + 1)2 và (a2 − 1)2 < a4 − a2 + 1 < (a2 )2 ,

suy ra điều vô lí.

76
5. HÀM SỐ HỌC

Vậy n = 3. 
Bài 5.27 (VMO 2016). Số nguyên dương n được gọi là số hoàn chỉnh nếu n bằng tổng các
ước số dương của nó (không kể chính nó).

a) Chứng minh rằng nếu n là số hoàn chỉnh lẻ thì n có dạng

n = ps m2

trong đó p là số nguyên tố có dạng 4k + 1, s là số nguyên dương có dạng 4h + 1 và m là


số nguyên dương không chia hết cho p.
n(n + 1)
b) Tìm tất cả các số nguyên dương n > 1 sao cho n − 1 và đều là các số hoàn chỉnh.
2
Lời giải.

a) Giả sử n = pα1 1 pα2 2 · · · pαk k , trong đó pi là các số nguyên tố lẻ. Vì n là số hoàn chỉnh nên

hu
1 + p1 + p21 + · · · + pα1 1 · · · 1 + pk + p2k + · · · + pαk k = 2n.
 

Vì n là số lẻ nên trong các thừa số 1 + p1 + p21 + · · · + pα1 1 , 1 + p2 + p22 + · · · + pα2 2 , . . .,

T
1 + pk + p2k + · · · + pαk k có đúng một thừa số là số chẵn. Do đó, trong các số α1 , α2 , . . ., αk
có đúng một số lẻ, còn các αi còn lại đều là số chẵn, giả sử αt lẻ. Do đó n = ps m2 .
Nếu p = 4k + 3, khi đó t
αt αt
3αt +1 − 1
Tấ
X X
pit ≡ (4k + 3)i ≡ (mod 4).
i=1 i=1
2

3αt +1 − 1 3αt +1 − 1
Vì 3n ≡ ±1 (mod 4), nên ≡ 0 (mod 4) hoặc ≡ 1 (mod 4). Tuy nhiên
2 2
n

αt
pit ≡ 2 (mod 4) nên dẫn tới vô lí. Do đó p = 4k + 1, suy ra
P
yễ

i=1

αt
X αt
X
pit ≡ (4k + 1)i ≡ αt + 1 ≡ 2 (mod 4),
gu

i=1 i=1

Dẫn tới αt ≡ 1 (mod 4) hay αt = 4h + 1.


N

b) Ta xét các trường hợp sau: 


TH1. Nếu n lẻ thì n − 1 chẵn nên ta có n − 1 = 2k 2k+1 − 1 với k ≥ 1, k ∈ N, 2k+1 − 1
nguyên tố.
n(n + 1)
Với k = 1 thì n = 7. Thử lại thấy = 28 là số hoàn chỉnh.
2
n(n + 1) n(n + 1)
Với k ≥ 2 thì n ≡ 1 (mod 4). Khi đó là số lẻ. Theo câu a) thì = ps m 2
2 2
với các điều kiện s, m, p như câu a). Từ đây ta suy ra
n+1
= 22k+1 − 2k + 1 · 22k − 2k−1 + 1 = ps m2 .
 

2
n+1
Do (n, n + 1) = 1 nên ít nhất một trong hai số n, phải là số chính phương.
2

77
5. HÀM SỐ HỌC

• n là số chính phương, khi đó tồn tại số tự nhiên x sao cho

n = x2 ⇔ 2k 2k+1 − 1 = x2 − 1 = (x − 1)(x + 1).




Do 2k+1 − 1 là số nguyên tố và x là số lẻ nên 2(2k+1 − 1) là ước của x − 1 hoặc x + 1.


Hơn nữa 2(x − 1) > x + 1 nên ta có
2
(x − 1)(x + 1) > 2 2k+1 − 1 > 2k 2k+1 − 1 = (x − 1)(x + 1).


Điều này vô lí.


n+1
• là số chính phương. Chứng minh tương tự như trên ta cũng dẫn đến vô lí.
2
TH2. Nếu n chẵn thì n − 1 lẻ. Do đó n − 1 = ps m2 suy ra n ≡ 2 (mod 4) vì p ≡ 1 (mod 4).
n(n + 1)
Do đó lẻ nên
2

hu
n(n + 1) (ps m2 + 1) (ps m2 + 2)
= = q t h2 ,
2 2
với q là số nguyên tố có dạng 4k + 1, t là số nguyên dương có dạng 4l + 1 và h không chia

T
hết cho q.
n
Do (n,n + 1) = 1 nên từ phương trình, một trong hai số , n + 1 là số chính phương. Chung
t 2
quy là ta đi giải hai phương trình nghiệm nguyên dương ps m2 + 1 = x2 hoặc ps m2 + 2 = x2 .
Hai phương trình này đều vô nghiệm nguyên dương vì ps m2 + 1 ≡ 2 (mod 4), ps m2 + 2 ≡ 3
Tấ
(mod 4).
Vậy n = 7 là đáp án duy nhất cho bài toán.


n
yễ
gu
N

78
Chương 2

PHƯƠNG TRÌNH DIOPHANTINE

§1. Một số phương trình Diophantine cổ điển

hu
I. Phương trình bậc nhất
1. Phương trình bậc nhất hai ẩn

T
Là phương trình có dạng
ax + by = c (2.1)
trong đó a, b, c là các số nguyên khác 0. t
Định lí sau cho chúng ta điều kiện của a, b, c để phương trình (2.1) có nghiệm nguyên x, y.
Tấ
Định lí 1. Phương trình (2.1) có nghiệm khi và chỉ khi (a,b) là ước của c.
Chứng minh. Dễ thấy nếu phương trình có nghiệm thì (a,b) là ước của c.
Giả sử d = (a,b) là ước của c. Khi đó a = da1 , b = db1 , c = dc1 với (a1 ,b1 ) = 1.
Xét a1 số kb1 với k = 0, 1, . . . , a1 − 1. Do (a1 ,b1 ) = 1 nên a1 số trên có số dư khác nhau khi chia
n

cho a1 , do đó tồn tại số k sao cho kb1 ≡ c1 (mod a1 ), hay tồn tại số nguyên l để
kb1 − c1 = la1 ⇔ kb1 + (−l)a1 = c1 .
yễ

Suy ra phương trình (2.1) có nghiệm nguyên.


gu

Nhận xét. Ta thấy (x; y) = (k; −l) là một nghiệm của phương trình (2.1) và (dk; −dl) cũng là
nghiệm của (2.1). Câu hỏi đặt ra là phương trình (2.1) có bao nhiêu nghiệm và có xác định được
các nghiệm của (2.1) hay không? Định lí sau sẽ cho ta câu trả lời.
N

Định lí 2. Nếu phương trình (2.1) có một nghiệm nguyên (x0 ; y0 ) thì nó có vô số nghiệm và
nghiệm nguyên (x; y) của nó được xác định bởi công thức
b


 x = x0 + t

 d
, t ∈ Z. (2.2)
a


y = y − t

0
d
Chứng minh.
Trước hết ta có x, y được xác định bởi (2.2) thỏa phương trình (2.1). Thật vậy
 
b  a 
ax + by = a x0 + t + b y0 − t = ax0 + by0 = c.
d d
Giả sử (x0 ; y0 ) là một nghiệm của phương trình, đặt d = (a,b) ta có a = a1 d, b = b1 d, (a1 ,b1 ) = 1

ax + by = c = ax0 + by0 ⇔ a1 (x − x0 ) = −b1 (y − y0 ).

79
1. MỘT SỐ PHƯƠNG TRÌNH DIOPHANTINE CỔ ĐIỂN

Suy ra b1 |x − x0 , hay x − x0 = a1 t suy ra y − y0 = −b1 t. Do vậy


(
x = x 0 + a1 t
.
y = y0 − b1 t

4! Để tìm nghiệm (x0 ; y0 ) của phương trình (2.1) (với (a,b) = 1) ta có thể làm như sau:
Theo định lí Ơle aφ(b) ≡ 1 (mod b) nên aφ(a) − 1 = kb, do đó
c · aφ(a) + b(−kc) = c.
(
x0 = c · aφ(b)−1
Ta chọn
y0 = −kc.

Ví dụ 1.1. Giải phương trình nghiệm nguyên 4x − 5y = 7.

Lời giải.

hu
Ta có (4,5) = 1 (
và (x0 ; y0 ) = (8; 5) là một nghiệm của phương trình. Do đó nghiệm của phương
x = 8 + 5t
trình đã cho là , t ∈ Z. 
y = 5 + 4t

T
4! Ta có thể tìm nghiệm của phương trình trên như sau:
y+7
Từ phương trình ta có x = y + , suy ra y + 7 = 4t, hay y = −7 + 4t và x = −7 + 5t.
4
(
t
x = −7 + 5t
Tấ
Vậy nghiệm của phương trình là , t ∈ Z.
y = −7 + 4t

2. Phương trình bậc nhất nhiều ẩn


n

Là phương trình có dạng


a1 x1 + a2 x1 + · · · an xn = b. (2.3)
yễ

Trong đó a1 , a2 , . . . , an , b là các số nguyên khác 0.


Tương tự như phương trình bậc nhất hai ẩn, định lí sau sẽ cho chúng ta điều kiện cần và đủ để
phương trình (2.3) có nghiệm.
gu

Định lí 3. Phương trình (2.3) có nghiệm nguyên x1 , x2 , . . . , xn khi và chỉ khi (a1 ,a2 , . . . ,an ) là
ước của b.
Điều kiện cần của định lí khá hiển nhiên. Để chứng minh điều kiện đủ ta sử dụng phương pháp
N

quy nạp toán học.

Ví dụ 1.2. Giải phương trình nghiệm nguyên 3x + 4y + 5z = 6.

Lời giải.
Ta có 3x + 4y ≡ 1 (mod 5), nên 3x + 4y = 1 + 5t. Từ đây ta có x = −1 + 3t, y = 1 − t, do đó
x = −1 + 3t

z = 1 − t. Vậy nghiệm của phương trình là y = 1 − t , t ∈ Z. 

z = 1 − t

Ví dụ 1.3. Cho a, b là các số nguyên dương nguyên tố cùng nhau, b > 1. Chứng minh
rằng với mọi số nguyên N , tồn tại duy nhất cặp số nguyên x, y thỏa mãn điều kiện

N = ax + by và 0 ≤ x < b.

80
1. MỘT SỐ PHƯƠNG TRÌNH DIOPHANTINE CỔ ĐIỂN

Lời giải.
• Chứng minh sự tồn tại
Vì (a,b) = 1 nên theo định lí Bezout, tồn tại hai số nguyên u, v sao cho

N = au + bv.

Mặt khác u = b.q + r với 0 ≤ r ≤ b − 1 nên ta có

N = a(b.q + r) + bv = a.r + b(v + a.q).

Chọn x = r, y = v + a.q ta được N = ax + by với0 ≤ x ≤ b − 1.


• Chứng minh tính duy nhất
Giả sử tồn tại hai bộ (x; y) và (x0 ; y 0 ) thỏa 0 ≤ x,x0 < b và

N = ax + by = ax0 + by 0

Hay

hu
a(x − x0 ) = b(y 0 − y) (1)
Do (a,b) = 1 nên từ (1) ta suy ra b|x − x0 . Lại có 0 ≤ x, x0 ≤ b − 1 nên suy ra x = x0 và y = y 0 .
Vậy bài toán được chứng minh. 

T
Ví dụ 1.4. Cho a, b là các số nguyên dương nguyên tố cùng nhau. Chứng minh rằng
N0 = ab − a − b là số nguyên lớn nhất không biểu diễn được dưới dạng ax + by với x, y là
t
các số nguyên không âm.
Tấ
Lời giải.
Để giải quyết bài toán, cần thực hiện hai bước sau
Bước 1: Chứng minh N0 không biểu diễn được dưới dạng ax + by với x,y là các số nguyên không
âm.
n

Giả sử tồn tại hai số nguyên không âm x, y sao cho


yễ

ax + by = N0 .

hay
gu

a(x + 1) + b(y + 1) = ab. (2.4)


.
Vì (a,b) = 1 nên từ (2.4), suy ra x + 1..b hay x + 1 ≥ b. Tương tự y + 1 ≥ a.
N

Khi đó
ax + by ≥ a(b − 1) + b(a − 1) = 2ab − a − b > ab − a − b.
Điều này dẫn đến mâu thuẫn. Do vậy N0 không biểu diễn được dưới dạng ax + by với x,y ∈ N.
Bước 2: Chứng minh với mọi số nguyên N > N0 thì N biểu diễn được dưới dạng ax + by với
x,y ∈ N.
Theo kết quả ví dụ 1.3 ta suy ra: tồn tại duy nhất cặp (x,y) để

N = ax + by với 0 ≤ x < b.

Ta chỉ cần chứng minh y ≥ 0. Thật vậy

N − ax ab − a − b − a(b − 1)
y= > = −1.
b b
Mà y là số nguyên nên ta có y ≥ 0.
Vậy bài toán được chứng minh. 
4!

81
1. MỘT SỐ PHƯƠNG TRÌNH DIOPHANTINE CỔ ĐIỂN

a) Bài toán trên được goi là định lí Sylvester.

b) Định lí Sylvester là một kết quả cho trường hợp riêng của bài toán tổng quát sau được gọi
là bài toán Frobenius về những đồng xu. Bài toán đó được phát biểu như sau:
Cho các số nguyên dương a1 , a2 , · · · , an có gcd(a1 , a2 , · · · , an ) = 1. Tìm số nguyên lớn nhất
không biểu diễn được dưới dạng k1 a1 + k2 a2 + · · · + kn an với k1 , k2 , · · · , kn là các số nguyên
không âm. Số nguyên lớn nhất này được gọi là số Frobenius và thường được ký hiệu là
g(a1 , a2 , · · · , an ).
Định lí Sylvester giải quyết trong trường hợp n = 2 và g(a,b) = ab − a − b.

Ví dụ 1.5 (Tổng quát bài VMO 2015). Cho các số nguyên dương a, b nguyên tố cùng
nhau. Khi đó N0 = a2 b + b2 a − a2 − b2 − ab là số nguyên dương lớn nhất không biểu diễn
được dưới dạng a2 x + aby + b2 z với x, y, z là các số nguyên không âm.

Lời giải.
ta chứng minh bài toán qua hai bước.

hu
Bước 1: Chứng minh N0 không biểu diễn được dưới dạng a2 x + aby + b2 z.
Giả sử tồn tại các số tự nhiên x, y, z để

T
N0 = a2 x + aby + b2 z

Hay
a2 (b − x − 1) + b2 (a − z − 1) = ab(y + 1).
t
Tấ
Từ đây, suy ra một trong hai số b − x − 1 và a − z − 1 có ít nhất một số dương. Không mất
.
tính tổng quát, ta giả sử b − x − 1 ≥ 0. Khi đó, do (a,b) = 1 nên ta suy ra b − x − 1..b, dẫn tới
b − x − 1 ≥ b hay x ≤ −1 (vô lí).
Do vậy N0 không biểu diễn được qua a2 x + aby + b2 z.
Bước 2: Chứng minh với mọi số nguyên N > N0 thì N biểu diễn được dưới dạng a2 x + aby + b2 z.
n

Do N > N0 nên N − ab2 + a2 + ab − a > ab2 − b2 − a nên theo định lí Sylvester ta suy ra tồn
yễ

tại các số tự nhiên u, z sao cho

N − ab2 + a2 + ab − a = ua + b2 z. (2.5)
gu

Vì u ≥ 0 nên u + ab − a − b + 1 > ab − a − b nên tiếp tục áp dụng định lí Sylvester suy ra tồn
tại các số tự nhiên x, y sao cho
N

u + ab − a − b + 1 = xa + yb. (2.6)

Thay (2.6) vào (2.5) ta được

N − ab2 + a2 + ab − a = a(ax + by − ab + a + b − 1) + b2 z

Hay
N = a2 x + aby + b2 z.
Bài toán được chứng minh. 

BÀI TẬP

Bài 1.1. Giải các phương trình nghiệm nguyên sau

a) 2x + 3y = 5. c) 2x + 3y − 5z = 8.

b) 3x − 7y = 13. d) 6x + 15y + 10z = 3.

82
1. MỘT SỐ PHƯƠNG TRÌNH DIOPHANTINE CỔ ĐIỂN

Bài 1.2. Tìm tất cả các số nguyên dương n sao cho n chai 3 dư 2, chia 5 dư 3 và chia 7 dư 5.
Bài 1.3. Cho ba đường thẳng x − 5y − 2 = 0, x − 8y − 1 = 0 và x − 11y − 3 = 0. Tìm tất cả các
số nguyên n để đường thẳng x − n = 0 cắt ba đường thẳng trên tại các điểm có tọa độ nguyên.
Bài 1.4. Cho a, b là hai số nguyên dương nguyên tố cùng nhau. Kí hiệu A là tập các số nguyên
dương n sao cho phương trình
ax + by = n
không có nghiệm nguyên dương.

a) Chứng minh rằng max A = ab.

b) Tính |A|.

Lời giải.

a) Trước hết ta chứng minh phương trình

hu
ax + by = ab (2.7)

không có nghiệm nguyên dương.


Giả sử (2.7) có nghiệm nguyên dương x, y. Khi đó ta có

T
by = a(b − x). (2.8)
.
t
Từ đây, suy ra x < b và y .. a, hay y = am với m ∈ N∗ . Thay vào phương trình (2.8) ta
Tấ
được
bm = b − x ⇔ x = b(1 − m) ≤ 0.
Điều này vô lí. Do đó phương trình (2.7) không có nghiệm nguyên dương.
Tieps theo ta chứng minh với mọi số nguyên dương n > ab thì phương trình
n

ax + by = n (2.9)
yễ

có nghiệm nguyên dương.


Thật vậy, xét b số ax với x ∈ {1,2, . . . ,b}. Do (a,b) = 1 nên {1,2, . . . ,b} là HĐĐ theo
gu

modunlo b, do đó tồn tại x ∈ {1,2, . . . ,b} để ax ≡ n (mod b), hay

n − ax = by ⇔ ax + by = n.
N

với y ∈ Z. Ta có
y = n − ax > ab − ab = 0
nên phương trình (2.9) có nghiệm nguyên dương.

b) Xét n ≤ ab. Ta chứng minh

n ∈ A ⇔ m = ab + a + b − n 6∈ A. (2.10)

Theo chứng minh trên tồn tại các số nguyên x0 , y0 thỏa mãn 1 ≤ x0 ≤ b và

n = ax0 + by0 .

Vì n ≤ ab nên y ≤ 0. Do đó

m = ab + a + b − n = ab + a + b − ax0 − by0 = a(b + 1 − x0 ) + b(1 − y0 ) = ax1 + by1 .

83
1. MỘT SỐ PHƯƠNG TRÌNH DIOPHANTINE CỔ ĐIỂN

với x1 = b + 1 − x0 > 0, y1 = 1 − y0 > 0. Do đó m 6∈ A.


Giả sử m 6∈ A, khi đó tồn tại các số nguyên dương x1 , y1 sao cho m = ax1 + by1 . Nếu n 6∈ A
thì tồn tại các số nguyên dương x0 , y0 sao cho n = ax0 + by0 . Khi đó

ab = m − a − b + n = ax1 + by1 − a − b + ax0 + by0 = a(x1 + x0 − 1) + b(y0 + y1 − 1).

Suy ra ab ∈ A, điều này vô lí. Do đó n ∈ A.


Ta thấy với n ∈ [1; a + b − 1] thì

m = ab + a + b − n ≥ ab + a + b − (a + b − 1) = ab + 1 > ab

nên m 6∈ A hay n ∈ A.
Xét n ∈ K = [a + b; ab] và ánh xạ f : K → K thỏa f (n) = ab + a + b − n, ta thấy f là một
|K| ab − a − b + 1
song ánh và n ∈ A ⇔ f (n) 6∈ A. Do đó |K ∩ A| = = .
2 2
ab − a − b + 1 ab + a + b − 1
Vậy |A| = a + b − 1 + = .
2 2

hu

Bài 1.5 (IMO 1983). Cho các số nguyên dương a, b, c đôi một nguyên tố cùng nhau. Chứng
minh rằng N0 = 2abc − ab − bc − ca là số nguyên dương nhỏ nhất không biểu diễn được dưới

T
dạng abx + bcy + caz với x, y, z là các số nguyên không âm.
Lời giải.
Bài toán được chứng minh qua hai bước t
Bước 1: Chứng minh N0 không biểu diễn được qua abx + bcy + caz. Chứng minh điều này tương
Tấ
tự như cách chứng minh định lí Sylvester .
Bước 2: Chứng minh với mọi số nguyên dương N > N0 thì N luôn biểu diễn được dưới dạng
abx + bcy + caz.
Vì (a; bc) = 1 nên tồn tại u, y sao cho au + bcy = N với 0 ≤ y ≤ a − 1 .
n

Tương tự, tồn tại v,z sao cho N = vb + zac với 0 ≤ z ≤ b − 1.


.
Từ đó, suy ra bcy + caz − N ..ab. Hay tồn tại số nguyên x sao cho
yễ

abx + bcy + caz = N.


gu

Ta chỉ cần chứng minh x ≥ 0. Thật vậy

N − bcy − caz 2abc − ab − bc − ca − bc(a − 1) − ca(b − 1)


x= > = −1.
ab ab
N

Suy ra x ≥ 0. Bài toán được chứng minh. 


Bài 1.6 (VN TST 2000). Cho ba số nguyên dương a, b, c đôi một nguyên tố cùng nhau.Số
nguyên dương n được gọi là số bướng bỉnh nếu n không biểu diễn được dưới dạng abx+bxy+caz
với x, y, z là các số nguyên dương. Hỏi có bao nhiêu số bướng bỉnh.
Lời giải.
Để giải bài toán này, ta chia làm hai bước
Bước 1: Chứng minh N0 = 2abc là số nguyên dương lớn nhất không biểu diễn được dưới dạng
abx + bcy + caz với x, y, z là các số nguyên dương.
Việc chứng minh N0 không biểu diễn được qua abx + bcy + caz với x, y, z là các số nguyên dương
được chứng minh tương tự như các bài toán trên.
Ta chứng minh với mọi số nguyên dương N > N0 thì N luôn biểu diễn được qua abx + bcy + caz
với x, y, z là các số nguyên dương.
Trước hết ta chứng minh nhận xét:
Với 1 ≤ x ≤ b, 1 ≤ y ≤ a thì ax + by lập thành hệ thặng dư đây đủ theo mô đun ab.
Thật vậy: Dễ thấy có tất cả a.b tổng ax + by với 1 ≤ x ≤ b, 1 ≤ y ≤ a.

84
1. MỘT SỐ PHƯƠNG TRÌNH DIOPHANTINE CỔ ĐIỂN

Giả sử tồn tại 1 ≤ x,x0 ≤ b, 1 ≤ y,y 0 ≤ a sao cho ax + by ≡ ax0 + by 0 ( mod ab).
.
Hay a(x − x0 ) ≡ b(y 0 − y)( mod ab). Từ đây, suy ra x − x0 ..b. Mà 1 ≤ x,x0 ≤ b nên suy ra x = x0 ,
dẫn đến y = y 0 . Do vậy nhận xét được chứng minh.
Theo nhận xét trên và kết hợp với a, b, c đôi một nguyên tố nên với 1 ≤ x ≤ b, 1 ≤ y ≤ a thì
c(ax + by) = acx + bcy lập thành hệ thặng dư đây đủ theo mô đun ab. Do đó với mọi số nguyên
dương N > N0 , luôn tồn tại 1 ≤ z0 ≤ a, 1 ≤ y0 ≤ b sao cho acz0 + bcy0 ≡ N ( mod ab) hay tồn
tại số nguyên x0 sao cho
acz0 + bcy0 + abx0 = N.
Lại có acz0 + bcy0 ≤ acb + bca = 2abc < N nên ta có x0 > 0.
Vậy với mọi N > N0 thì N biểu diễn được dưới dạng abx + bcy + caz với x, y, z là các số nguyên
dương.
Bước 2: Đặt B là tập các số nguyên dương không biểu diễn được dưới dạng abx + bcy + caz với
x, y, z là các số nguyên dương.
Và A = {1,2, · · · , ab + bc + ca − 1} ∪ {ab + bc + ca,ab + bc + ca + 1, · · · , 2abc}.
Dễ thấy {1,2, · · · , ab + bc + ca − 1} ⊂ B.

hu
Đặt C = {ab + bc + ca,ab + bc + ca + 1, · · · , 2abc}. Ta cần tìm |B ∩ C|.
Với mỗi n ∈ C ta xét hàm f (n) = 2abc + ab + bc + ca − n. Ta chứng minh

n ∈ B ⇔ f (n) ∈
/B (1).

T
Với n ∈ B, theo chứng minh trên suy ra tồn tại các số nguyên 1 ≤ y0 ≤ b, 1 ≤ z0 ≤ c và x0 sao
cho t
n = abx0 + bcy0 + caz0 .
Tấ
Vì n ∈ B nên x0 ≤ 0. Khi đó

f (n) = ab(1 − x0 ) + bc(1 + a − y0 ) + ca(1 + b − z0 ) ∈


/ B.

Giả sử tồn tại n ∈ / B. Suy ra tồn tại các số nguyên dương x, y, z,x0 , y 0 , z 0 sao cho
/ B để f (n) ∈
n

n = abx + bcy + caz và f (n) = abx0 + bcy 0 + caz 0


yễ

Hay
2abc = (x + x0 − 1)ab + (y + y 0 − 1)bc + (z + z 0 − 1)ca
gu

Suy ra 2abc ∈
/ B vô lí. Vậy (1) được chứng minh.
Từ chứng minh trên ta suy ra
N

1 2abc − ab − bc − ca + 1
|B ∩ C| = |C| = .
2 2
ab + bc + ca − 1
Vậy số các số bướng bỉnh là: abc + . 
2

85
2. MỘT SỐ PHƯƠNG PHÁP GIẢI PHƯƠNG TRÌNH NGHIỆM NGUYÊN

II. Phương trình bậc hai


1. Phương trình Pitago
Xét phương trình
x2 + y 2 = z 2 (2.11)
với x, y, z là các số nguyên dương.

• Một bộ nguyên dương (x ; y; z) thỏa mãn (2.11) được gọi là một bộ Pythagore. Chúng
biểu thị độ dài ba cạnh của một tam giác vuông.

• Nếu (x; y; z) là một bộ Pythagore thì (kx; ky; kz) cũng là một bộ Pythagore với
mọi
 x số nguyên dương k. Ngược lại, với mội
 x bộ Pythagore (x; y; z) và d = (x,y,z) thì bộ
y z y z
; ; cũng là bộ Pythagore và , , = 1.
d d d d d d
• Bộ Pythagore (x; y; z) mà (x, y, z) = 1 được gọi là bộ Pythagore nguyên thủy. Rõ

hu
ràng để giải (1) ta chỉ cần đi tìm các bộ Pythagore nguyên thủy. Hơn nữa nếu (x; y; z)
là bộ Pythagore nguyên thủy thì x, y, z đôi một nguyên tố cùng nhau.

Định lí sau sẽ cho chúng ta về cấu trúc nghiệm nguyên thủy của (2.11).

T
Định lí 4. Nếu (x; y; z) là bộ Pythagore nguyên thủy thì x, y khác tính chẵn lẻ. Nếu y chẵn
thì (x; y; z) có dạng 
2 2
x = a − b
 t
y = 2ab (2.12)
Tấ
 2 2
z =a +b

với a, b là hai số nguyên dương nguyên tố cùng nhau, khác nhau tính chẵn lẻ.
Ngược lại, nếu bộ (x; y; z) thỏa (2.12) với a, b là hai số nguyên dương nguyên tố cùng nhau,
khác nhau tính chẵn lẻ thì (x; y; z) là một bộ Pythagore nguyên thủy.
n
yễ

§2. Một số phương pháp giải phương trình nghiệm


nguyên
gu

I. Đưa về phương trình tích


N

Xét phương trình


F (x1 ,x2 , . . . ,xn ) = 0, (1)
với x1 , x2 , . . . , xn là các số nguyên (hoặc nguyên dương).
Giả sử ta có thể biến đổi (1) về dạng:

f1 (x1 ,x2 , . . . ,xn ) .f2 (x1 ,x2 , . . . ,xn ) . . . fk (x1 ,x2 , . . . ,xn ) = m. (2)

Trong đó f1 ,f2 , . . . ,fn là các đa thức n biến hệ số nguyên. Khi đó, ta phân tích m thành tích của
k số nguyên m1 , m2 , . . . ., mk và giải (2) chuyển về giải hệ phương trình


 f1 (x1 ,x2 , . . . ,xn ) = m1

f (x ,x , . . . ,x ) = m
2 1 2 n 2
.

 ..................

fk (x1 ,x2 , . . . ,xn ) = mk

86
2. MỘT SỐ PHƯƠNG PHÁP GIẢI PHƯƠNG TRÌNH NGHIỆM NGUYÊN

Ví dụ 2.1. Tìm nghiệm nguyên dương của phương trình


1 1 1
+ = .
x y 7

Lời giải.
Do tính chất đối xứng nên ta chỉ cần tìm nghiệm (x; y) mà x ≥ y.
Phương trình tương đương với

7 (x + y) = xy ⇔ (x − 7) (y − 7) = 49 = 72 .

Vì x − 7 ≥ y − 7 > −7 nên ta có các trường hợp sau:


( (
x − 7 = 49 x = 56
• ⇔ .
y−7=1 y=8

hu
(
x−7=7
• ⇔ x = y = 14.
y−7=7

T
Vậy nghiệm của phương trình là: (x; y) = (56; 8) , (8; 56) , (7; 7).
4! Nếu n = pα1 1 pα2 2 . . . pαk k thì số nghiệm nguyên dương của phương trình
t
1 1 1
Tấ
+ =
x y n

là (1 + 2α1 ) (1 + 2α2 ) . . . (1 + 2αk ).



n

Ví dụ 2.2. Tìm các số tự nhiên x, y thỏa mãn (xy − 7)2 = x2 + y 2 .


yễ

Lời giải.
Phương trình tương đương với
gu

(xy − 6)2 + 13 = (x + y)2


⇔ (xy − 6)2 − (x + y)2 = −13
N

⇔ (xy − 6 + (x + y)) (xy − 6 − (x + y)) = −13


( (
xy − 6 + x + y = 13 xy − 6 + x + y = 1
⇔ ∨
xy − 6 − (x + y) = −1 xy − 6 − (x + y) = −13
( (
x+y =7 x+y =7
⇔ ∨ .
xy = 12 xy = 0

Giải ra ta được các cặp nghiệm (x; y) là: (3; 4) , (4; 3) , (0; 7) , (7; 0) .


Ví dụ 2.3. Tìm các nghiệm nguyên dương của phương trình

x3 + y 3 + z 3 = 3xyz + 5.

87
2. MỘT SỐ PHƯƠNG PHÁP GIẢI PHƯƠNG TRÌNH NGHIỆM NGUYÊN

Lời giải.
Ta có
x3 + y 3 + z 3 − 3xyz = (x + y + z) x2 + y 2 + z 2 − xy − yz − zx .


Nên phương trình trở thành

(x + y + z) x2 + y 2 + z 2 − xy − yz − zx = 5.


Vì x + y + z > 1 nên ta có
( (
x+y+z =5 x+y+z =5
⇔ .
x2 + y 2 + z 2 − xy − yz − zx = 1 (x − y)2 + (y − z)2 + (z − x)2 = 2

Giả sử x ≥ y ≥ z, khi đó nếu x > y > z thì

(x − y)2 + (y − z)2 + (z − x)2 ≥ 3 > 2.

hu
Do đó phương trình vô nghiệm, suy ra trong ba số x, y, z có hai số bằng nhau. Suy ra x = y = z+1
hoặc x = y + 1 = z + 1.
Từ đó ta tìm được các bộ (x; y; z) là: (2; 2; 1) , (2; 1; 2) , (1; 2; 2) . 
Bài tập

T
Bài 2.1. Tìm nghiệm nguyên của phương trình x3 + y 3 − x2 y − xy 2 = 5.
Lời giải.
Phương trình t
⇔ (x + y) x2 − xy + y 2 − xy (x + y) = 5

Tấ
⇔ (x + y) (x − y)2 = 5
( ( (
x+y =5 x=2 x=3
⇔ 2 ⇔ ∨
(x − y) = 1 y=3 y=2
n

Vậy nghiệm của phương trình là (x; y) ∈ {(2; 3) , (3; 2)}.



yễ

Bài 2.2. Tìm các nghiệm nguyên của phương trình x3 + y 3 − 3xy = 3.
Lời giải.
gu

Phương trình tương đương với

x3 + y 3 + 1 − 3xy = 4 ⇔ (x + y + 1) x2 + y 2 + 1 − x − y − xy = 4.

N

Do
1
x2 + y 2 + 1 − x − y − xy = (x − y)2 + (x − 1)2 + (y − 1)2 ≥ 0,

2
nên ta có các trường hợp sau
( ( (
x+y+1=1 x = −y x = −y
• 2 2 ⇔ 2 ⇔ .
x + y + 1 − x − y − xy = 4 x =1 x = ±1

x + y = 1
( (
x+y+1=2 x+y =1
• ⇔ ⇔ (vô nghiệm).
x2 + y 2 + 1 − x − y − xy = 2 (x + y)2 − 3xy = 2 xy = − 1
3
( ( (
x+y+1=4 x+y =3 x+y =3
• 2 2 ⇔ 2 ⇔ .
x + y + 1 − x − y − xy = 1 (x + y) − (x + y) − 3xy = 0 xy = 2
( (
x=2 x=1
Suy ra hoặc .
y=1 y=2

88
2. MỘT SỐ PHƯƠNG PHÁP GIẢI PHƯƠNG TRÌNH NGHIỆM NGUYÊN

Vậy phương trình có các cặp nghiệm (−1; 1) , (1; −1) , (2; 1) , (1; 2).

Bài 2.3. Với số nguyên dương n, ta kí hiệu s (n) là số nghiệm nguyên dương của phương trình
1 1 1
+ = .
x y n
Tìm tất cả các số nguyên dương n sao cho s (n) = 5.
Lời giải.
Đặt n = pα1 1 .pα2 2 . . . pαk k , với p1 ,p2 , . . . ,pk là các số nguyên tố phân biệt và α1 ,α2 , . . . ,αk là các số
nguyên dương. Phương trình đã cho tương đương với
(x − n) (y − n) = n2 .
Số nghiệm của phương trình đã cho chính bằng số ước nguyên dương của n2 = p2α
1 .p2
1 2α2
. . . p2α
k
k

nên ta có
s (n) = (1 + 2α1 ) (1 + 2α2 ) . . . (1 + 2αk ) .

hu
Suy ra s (n) = 5 thì k = 1 và 1 + 2α1 = 5 ⇒ α1 = 2.
Vậy n = p2 với p nguyên tố là số cần tìm.


T
Bài 2.4. Tìm nghiệm nguyên của phương trình
x2 + 1 y 2 + 1 + 2 (x − y) (1 − xy) = 4 (1 + xy) .
 
t
Tấ
Lời giải.
Phương trình đã cho tương đương với
x2 y 2 + x2 + y 2 + 1 + 2 (x − y) (1 − xy) = 4 + 4xy
⇔ (xy − 1)2 + (x − y)2 + 2 (x − y) (1 − xy) = 4
n

⇔ (1 − xy + x − y)2 = 4 ⇔ 1 − xy + x − y = ±2.
yễ

• Ta có 1 − xy +
(x − y = 2 ⇔ (x (
− 1) (y − 1) = −2.
( (
x−1=1 x − 1 = −2 x=2 x = −1
Từ đó suy ra ∨ ⇔ ∨ .
gu

y − 1 = −2 y−1=1 y = −1 y=2
( (
x=2 x=3
• 1 − xy + x − y = −2 ⇔ (x − 1) (y − 1) = 2 ⇔ ∨ .
N

y=3 y=2

Vậy nghiệm của phương trình là: (x; y) = (2; −1) , (−1; 2) , (2; 3) , (3; 2) .

Bài 2.5. Giải phương trình nghiệm nguyên:
x2 (y − 1) + y 2 (x − 1) = 1.

Lời giải.
Đặt a = x − 1, b = y − 1 ta có phương trình
(a + 1)2 b + (b + 1)2 a = 1
⇔a2 b + b2 a + 4ab + a + b = 1
⇔ab (a + b) + a + b + 4ab = 1
⇔ (a + b) (ab + 1) + 4 (ab + 1) = 5
⇔ (ab + 1) (a + b + 4) = 5.

89
2. MỘT SỐ PHƯƠNG PHÁP GIẢI PHƯƠNG TRÌNH NGHIỆM NGUYÊN
( (
ab + 1 = 1 ab = 0
• ⇔ ⇔ (a; b) = (0; 1) , (1; 0) .
a+b+4=5 a+b=1
( (
ab + 1 = 5 ab = 4
• ⇔ (vô nghiệm).
a+b+4=1 a + b = −3
( (
ab + 1 = −5 ab = −6
• ⇔ ⇔ (a; b) = (1; −6) , (−6; 1) .
a + b + 4 = −1 a + b = −5
( (
ab + 1 = −1 ab = −2
• ⇔ (vô nghiệm).
a + b + 4 = −5 a + b = −9

Vậy nghiệm của phương trình là: (x; y) = (1; 2) , (2; 1) , (2; −4) , (−4; 2) .

Bài 2.6. Tìm tất cả các số nguyên dương x, y thỏa mãn 3x − 2y = 1.

hu
Lời giải.
( (x,y) = (1; 1) là một nghiệm của phương trình .
Ta thấy
x≥2
Xét .

T
y≥2
Ta có 3x = 2y + 1 ⇒ y lẻ, suy ra x chẵn hay x = 2x1 . Suy ra
32x1 − 1 = 2y ⇔ (3x1 − 1) (3x1 + 1) = 2y .
t
Tấ
(
3x1 − 1 = 2a
Dẫn đến với a + b = y; b > a ≥ 1.
3x1 + 1 = 2b
Từ đó, ta có
( (
n

a=1 a=1
2b − 2a = 2 ⇒ 2a 2b−a − 1 = 2 ⇒

⇔ ⇒ x1 = 1.
b−a=1 b=2
yễ

Vậy (x; y) = (1; 1) , (2; 3) là nghiệm cần tìm.



gu

Bài 2.7. Tìm nghiệm nguyên dương của phương trình


x3 + x2 + x + 1 = 2011y .
N

Lời giải.
Phương trình đã cho tương đương với:
x2 + 1 (x + 1) = 2011y .

(∗)

Ta có (x + 1,x2 + 1) = (x + 1,2) = 1 (vì từ (*) ta suy ra x + 1 lẻ).


Mà 2011 là số nguyên tố nên
(
x+1=1
2 y
 x + 1 = 2011

(∗) ⇔  (
 ⇔ x = y = 0.
 x + 1 = 2011y
x2 + 1 = 1

Vậy x = y = 0 là nghiệm của phương trình.




90
2. MỘT SỐ PHƯƠNG PHÁP GIẢI PHƯƠNG TRÌNH NGHIỆM NGUYÊN

Bài 2.8. Tìm nghiệm tự nhiên của phương trình

(2x + 1) (2x + 2) (2x + 3) (2x + 4) − 5y = 11879.

Lời giải.
Đặt
A = (2x + 1) (2x + 2) (2x + 3) (2x + 4) ,
ta có 2x · A là tích của 5 số tự nhiên liên tiếp nên 2x · A chia hết cho 5. Nhưng 2x không chia hết
cho 5, do đó A chia hết cho 5.
Nếu y ≥ 1, ta có
(2x + 1) (2x + 2) (2x + 3) (2x + 4) − 5y
chia hết cho 5 mà 11879 không chia hết cho 5 nên y ≥ 1 không thỏa mãn, suy ra y = 0.
Khi đó, ta có
(2x + 1) (2x + 2) (2x + 3) (2x + 4) − 5y = 11879
⇔ (2x + 1) (2x + 2) (2x + 3) (2x + 4) − 1 = 11879

hu
⇔ (2x + 1) (2x + 2) (2x + 3) (2x + 4) = 11880
⇔ (2x + 1) (2x + 2) (2x + 3) (2x + 4) = 9.10.11.12 ⇔ x = 3.

T
Vậy x = 3; y = 0 là hai giá trị cần tìm.

Bài 2.9. Tìm các bộ số nguyên dương (x; y; z) thỏa mãn phương trình
t
x3 + y 3 + z 3 − 3xyz = p,
Tấ
với p là số nguyên tố lớn hơn 3.
Lời giải.
Ta có
n

x3 + y 3 + z 3 − 3xyz = (x + y + z) x2 + y 2 + z 2 − xy − yz − zx .


Nên phương trình trở thành


yễ

(x + y + z) x2 + y 2 + z 2 − xy − yz − zx = p.

gu

Vì x + y + z > 1 nên ta có
(
x+y+z =p
.
N

x2 + y 2 + z 2 − xy − yz − zx = 1

Phương trình thứ hai của hệ được viết thành

(x − y)2 + (y − z)2 + (z − x)2 = 2.

Do tính đối xứng của x, y, z nên ta giả sử x ≥ y ≥ z, khi đó nếu x > y > z thì

(x − y)2 + (y − z)2 + (z − x)2 ≥ 3 > 2.

Do đó phương trình vô nghiệm, suy ra trong ba số x, y, z có hai số bằng nhau. Suy ra x = y = z+1
hoặc x = y + 1 = z + 1.
Vì số nguyên tố p > 3 nên p có dạng
 3k + 1 hoặc 3k +2.
p+2 p−1 p−1
Với dạng thứ nhất ta có nghiệm ; ; và các hoán vị.
 3 3 3
p+1 p+1 p−2
Với dạng thứ hai ta có nghiệm ; ; và các hoán vị.
3 3 3


91
2. MỘT SỐ PHƯƠNG PHÁP GIẢI PHƯƠNG TRÌNH NGHIỆM NGUYÊN

Bài 2.10 (Thái Nguyễn TST 2013). Tìm tất cả các nghiệm nguyên của phương trình

(x + 1)(x + 2)(x + 8)(x + 9) = y 2 .

Lời giải.
Ta có
(x + 1)(x + 2)(x + 8)(x + 9) = (x2 + 10x + 9)(x2 + 10x + 16) = y 2 .
.
Gọi x2 + 10x + 9 = t(t ∈ Z)ta có t(t + 7) = y 2 . Dễ thấy y .. t ⇒ y = kt(k ∈ Z) thay vào ta có

t(t + 7) = k 2 .t2 ⇔ t + 7 = k 2 .t ⇔ t(k 2 − 1) = 7.

Đến đây thử từng giá trị vì cả t và k 2 − 1 đều thuộc Z. Bài làm coi như xong.
4 ! Ta có (x2 + 10x + 9)(x2 + 10x + 16) = y 2 . Đặt x2 + 10x + 9 = t, ta có t2 + 7t = y 2 hay
(2t + 2y + 7)(2t − 2y + 7) = 49. Đến đây giải quyết tiếp dễ dàng.

hu

Bài 2.11. Giải phương trình nghiệm nguyên x, y :

x2 + 6xy + 8y 2 + 3x + 6y = 2.

T
Bài 2.12. Cho p và q là các số nguyên tố. Tìm số cặp nguyên dương (x; y) thỏa mãn phương
trình t
p q
+ = 1.
Tấ
x y
Bài 2.13. Giải phương trình nghiệm nguyên dương

x3 − y 3 = xy + 61.
n

Bài 2.14. Tìm số nguyên tố x và số nguyên y thỏa mãn


yễ

x − y 4 = 4.
gu

Bài 2.15. Tìm cặp số nguyên (x; y) thỏa mãn phương trình

x6 + 3x3 + 1 = y 4 .
N

Bài 2.16. Giải phương trình nghiệm tự nhiên

x2 + y x + y 2 = (x − y)3 .
 

Bài 2.17. Tìm tất cả các tam giác vuông có độ dài các cạnh là số nguyên và chu vi và diện tích
của tam giác bằng nhau.
Bài 2.18. Giải hệ phương trình nghiệm nguyên
(
x + y + z + u + v = xyuv + (x + y) (u + v)
.
xy + z + uv = xy (u + z) + uv (x + y)

92
2. MỘT SỐ PHƯƠNG PHÁP GIẢI PHƯƠNG TRÌNH NGHIỆM NGUYÊN

subsectionPhương pháp đánh giá Trong phương pháp này, ta hạn chế các biến bằng cách sử dụng
bất đẳng thức. Thông thường, quá trình đánh giá sẽ dẫn đến hữu hạn khả năng các biến.
. .
Ví dụ 2.1. Tìm các cặp số nguyên dương (m, n) thỏa mãn 2m − 1 .. n và 2n − 1 .. m.

Lờigiải.
2m − 1 ... n
Vì , suy ra
..
2n − 1 . m

.
(2m − 1) (2n − 1) = 4mn − 2 (m + n) + 1 .. mn.

Hay
.
2 (m + n) − 1 .. mn ⇒ 2 (m + n) − 1 ≥ mn

hu
⇔ m (n − 2) − 2 (n − 2) ≤ 3 ⇔ (m − 2) (n − 2) ≤ 3. (2.13)

Không mất tính tổng quát, ta giả sử m ≥ n. Ta xét các trường hợp sau:

T
.
• n = 1, khi đó 2n − 1 = 1 .. m ⇒ m = 1.
.
• n = 2, khi đó 2n − 1 = 3 .. m ⇒ m = 3. t
.
• n = 3, khi đó 2n − 1 = 5 .. m ⇒ m = 5.
Tấ
(
n−2≥2
• n ≥ 4 ⇒ m ≥ 4 nên ⇒ (n − 2) (m − 2) ≥ 4 nên (2.13) không thỏa mãn.
m−2≥2
n

Vậy các cặp (m,n) cần tìm là: (1; 1) , (2; 3) , (3; 2) , (3; 5) , (5; 3).

yễ

Ví dụ 2.2. Tìm nghiệm nguyên của phương trình: x2 + (x + 1)2 = y 4 + (y + 1)4 .


gu

Lời giải.
Khai triển và rút gọn hai vế ta được:
N

x(x + 1) = y 4 + 2y 3 + 3y 2 + 2y
⇔ x2 + x = y 2 (y + 1)2 + 2y(y + 1)
⇔ x2 + x + 1 = (y 2 + y + 1)2 (2.14)

• Nếu x > 0 thì từ x2 < 1 + x + x2 < (x + 1)2 . Suy ra 1 + x + x2 không là số chính phương
nên (2.14) không có nghiệm nguyên.

• Nếu x < −1 thì từ (x + 1)2 < 1 + x + x2 < x2 suy ra (2.14) không có nghiệm nguyên.
"
y=0
• Nếu x = 0 hoặc x = −1 thì từ (2.14) suy ra y 2 + y + 1 = ±1 ⇔ .
y = −1

Vậy phương trình có 4 nghiệm nguyên ( x; y) ∈ {(0; 0) ; (0; −1) ; (−1; 0) ; (−1; −1)} .


93
2. MỘT SỐ PHƯƠNG PHÁP GIẢI PHƯƠNG TRÌNH NGHIỆM NGUYÊN

Ví dụ 2.3. Tìm nghiệm nguyên dương của phương trình:

5(x + y + z + t) + 10 = 2xyzt. (2.15)

Lời giải.
Vì x, y, z có vai trò như nhau nên ta giả sử x ≥ y ≥ z ≥ t ≥ 1. Từ (2.15) suy ra:
"
5 5 5 10 30 t=1
2= + + + ≤ 3 ⇒ .
xyz xzt xyt xyzt t t=2

• Với t = 1ta có:



z=1
5 5 5 15 30 2
5(x + y + z) + 15 = 2xyz ⇒ 2 = + + + ≤ 2 ⇒ z ≤ 15 ⇒ z = 2 .

xy yz xz xyz z
z=3
– Với z = 1 ta có:

hu
( (
2x − 5 = 65 x = 35
 2y − 5 = 1  y=3
 
5(x + y) + 20 = 2xy ⇔ (2x − 5)(2y − 5) = 65 ⇔  ⇔ .

T
( (
 2x − 5 = 13  x=9
 

2y − 5 = 5 y=5
t
Ta có các nghiệm (x, y, z, t) = (35, 3, 1, 1 ) , (9, 5, 1, 1) và các hoán vị của chúng.
Tấ
– Với z = 2 và z = 3, phương trình không có nghiệm nguyên dương.
• Với t = 2, ta có:
5 5 5 20 35 35
5(x + y + z) + 20 = 4xyz ⇒ 4 = + + + ≤ 2 ⇒ z2 ≤ < 9.
xy yz xz xyz z 4
n

Suy ra z = 2 (vì z ≥ t = 2).


yễ

Khi đó:
5(x + y) + 30 = 8xy ⇔ (8x − 5)(8y − 5) = 265.
Do x ≥ y ≥ z ≥ t ≥ 2 nên 8x − 5 ≥ 8y − 5 ≥ 11, mà 265 = 53.5. Trường hợp này phương
gu

trình không có nghiệm nguyên dương.



N

Bài tập
Bài 2.1. Tìm tất cả các cặp số nguyên (x; y) thỏa mãn
x3 + y 3 = (x + y)2 .

Lời giải.
Tất cả các cặp số (k; −k) , k ∈ Z là một lời giải của phương trình.
Xét x + y 6= 0, khi đó phương trình trở thành
x2 − xy + y 2 = x + y.
Hay là
(x − y)2 + (x − 1)2 + (y − 1)2 = 2.
Từ bất đẳng thức
(x − 1)2 ≤ 1, (y − 1)2 ≤ 1
ta có được x, y ∈ [0; 2].
Chúng ta có các lời giải: (0; 1) , (1; 0) , (1; 2) , (2; 1) , (2; 2) . 

94
2. MỘT SỐ PHƯƠNG PHÁP GIẢI PHƯƠNG TRÌNH NGHIỆM NGUYÊN

Bài 2.2. Giải phương trình nghiệm nguyên dương


1 1 1 3
+ + = .
x y z 5

Lời giải.
3 3
Từ phương trình ta có thể giả sử 2 ≤ x ≤ y ≤ z. Ta có bất đẳng thức ≥ , hay x ∈ {3, 4, 5}.
x 5
1 1 1 100
• Nếu x = 2 thì + = với y ∈ {11, 12, · · · ,20} . Ta có z = 10 + và (y − 10)| 100.
y z 10 y − 10
Chúng ta có các nghiệm (2; 11; 110) , (2; 12; 60) , (2; 14; 35) , (2; 15; 30) , (2; 20; 20) .
1 1 4
• Nếu x = 3, thì + = với y ∈ {3; 4; 5; 6; 7}. Chúng ta có được các nghiệm là
y z 15
(3; 4; 60) , (3; 5; 15) , (3; 6; 10) .
1 1 7

hu
• Nếu x = 4, thì + = với y ∈ {4; 5}. Chúng ta có các nghiệm (4; 4; 10) .
y z 20
1 1 2
• Nếu x = 5, thì + = và y = z = 5 nên ta có nghiệm (5; 5; 5) .

T
y z 5

Bài 2.3. Tìm tất cả các bộ bốn số nguyên dương (x; y; z; w) thỏa mãn
t
Tấ
x2 + y 2 + z 2 + 2xy + 2x (z − 1) + 2y (z + 1) = w2 .

Lời giải.
Ta có
n

(x + y + z ± 1)2 = x2 + y 2 + z 2 + 2xy + 2x (z ± 1) + 2y (z ± 1) ± 2z + 1.
yễ

Suy ra
(x + y + z − 1)2 < w2 < (x + y + z + 1)2 .
Nên ta có
gu

x2 + y 2 + z 2 + 2xy + 2x (z − 1) + 2y (z + 1) = (x + y + z)2 .
Dẫn tới x = y.
Vậy nghiệm của phương trình là (m; m; n; 2m + n) với m,n ∈ Z+ . 
N

Bài 2.4. Tìm tất cả các nghiệm nguyên của phương trình

x3 + (x + 1)3 + (x + 2)3 + · · · + (x + 7)3 = y 3 .

Lời giải.
Đặt
P (x) = x3 + (x + 1)3 + (x + 2)3 + · · · + (x + 7)3 .
Với x ≥ 0 ta có

(2x + 7)3 = 8x3 + 84x2 + 294x + 343 < P (x) < 8x3 + 120x2 + 600x + 1000 = (2x + 10)3 ,

do đó 2x + 7 < y < 2x + 10.


Nên y chỉ có thể là 2x + 8 hoặc 2x + 9. Nhưng phương trình

P (x) − (2x + 8)3 = −12x2 + 36x + 272 = 0

95
2. MỘT SỐ PHƯƠNG PHÁP GIẢI PHƯƠNG TRÌNH NGHIỆM NGUYÊN


P (x) − (2x + 9)3 = −24x2 − 66x + 55 = 0.
Không có nghiệm nguyên, do đó phương trình đã cho không có nghiệm nguyên khi x ≥ 0.
Xét x < 0, ta thấy P (−x − 7) = −P (x), do đó với x ≤ −7 thì phương trình ban đầu vô nghiệm.
Suy ra x ∈ {−1; −2; −3; −4; −5; −6} .
Bằng cách thay các giá trị của x vào kiểm tra ta thu được các nghiệm của phương trình là
(−2; 6) , (−3; 4) , (−4; −4) , (−5; −6) .

Bài 2.5. Tìm các số nguyên dương x,y,z thỏa mãn
   
1 1 1
1+ 1+ 1+ = 2.
x y z

Lời giải.
Không mất tính tổng quát, ta giả sử x ≥ y ≥ z. Ta có

hu
 3
1
1+ ≥ 2 hay z ≤ 3.
z

T
• z = 1 ta có phương trình   
1 1
1+ 1+ = 1,
ty z
phương trình không có nghiệm nguyên dương.
Tấ
• z = 2 ta có phương trình   
1 1 4
1+ 1+ = .
y z 3
 2
1 4
n

Ta có 1 + ≥ nên y < 7, thay vào ta thu được các nghiệm (7; 6; 2) , (9; 5; 2) , (15; 4; 2) .
y 3
yễ

• z = 3 ta có phương trình   
1 1 3
1+ 1+ = .
y z 2
gu

Suy ra y < 5, từ đó ta tìm được các nghiệm (8; 3; 3) và (5; 4; 3) .


Vậy nghiệm của phương trình đã cho là: (7; 6; 2) , (9; 5; 2) , (15; 4; 2) , (8; 3; 3) , (5; 4; 3) và các
N

hoán vị.

Bài 2.6. Giải phương trình nghiệm nguyên dương

3 (xy + yz + zx) = 4xyz.

Bài 2.7. Tìm tất cả các bộ nguyên dương (x; y; z) thỏa mãn phương trình

xy + yz + zx − xyz = 2.

Bài 2.8. Tìm tất cả các bộ nguyên dương (x; y; z) thỏa mãn phương trình

(x + y)2 + 3x + y + 1 = z 2 .

96
2. MỘT SỐ PHƯƠNG PHÁP GIẢI PHƯƠNG TRÌNH NGHIỆM NGUYÊN

Bài 2.9. Xác định tất cả các cặp số nguyên (x; y) thỏa mãn phương trình

(x + 1)4 − (x − 1)4 = y 3 .

Bài 2.10. Chứng minh rằng phương trình

x6 + ax4 + bx2 + c = y 3 ,

Với a ∈ {3,4,5} , b ∈ {4,5, . . . ,12} , c ∈ {1,2, . . . ,8} không có nghiệm nguyên.


Bài 2.11. Giải phương trình nghiệm nguyên dương

x2 y + y 2 z + z 2 x = 3xyz.

Bài 2.12. Tìm các số nguyên x,y thỏa mãn phương trình

hu
2
x2 − y 2 = 1 + 16y.

T
Bài 2.13. Tìm tất cả các bộ số nguyên (a,b,c,x,y,z) thỏa mãn
( t
a + b + c = xyz
Tấ
x + y + z = abc

và a ≥ b ≥ c ≥ 1, x ≥ y ≥ z ≥ 1.
Bài 2.14. Cho các số nguyên dương x,y,z,u,v thỏa mãn phương trình
n

xyzuv = x + y + z + u + v.
yễ

Tìm giá trị lớn nhất của max {x,y,z,u,v} .


Bài 2.15. Tìm các số nguyên dương phân biệt x,y,z,t thỏa phương trình
gu

x2 + y 2 + z 2 + t2 = 3 (x + y + z + t) .
N

Bài 2.16. Cho a,b là các số nguyên dương thỏa mãn ab + 1 là ước của a2 + b2 . Chứng minh rằng
a2 + b 2
là số chính phương.
ab + 1

II. Phương pháp lùi vô hạn


Phương pháp lùi vô hạn thường được áp dụng để chứng minh mệnh đề P (n) sai với n đủ lớn
như sau:
Cho k là số tự nhiên. Ta có:

• P (k) không đúng.

• Nếu P (m) đúng với số nguyên bất kì m > k, thì sẽ có một số nguyên j, m > j ≥ k sao
cho P (j) đúng.

97
2. MỘT SỐ PHƯƠNG PHÁP GIẢI PHƯƠNG TRÌNH NGHIỆM NGUYÊN

Khi đó P (n) sai với mọi n ≥ k.

Ví dụ 2.1. Tìm các số tự nhiên x,y,z thỏa mãn phương trình

x3 + 2y 3 = 4z 3 .

Lời giải.
Ta thấy x = y = z = 0 là một nghiệm của phương trình.
Giả sử phương trình có nghiệm (x; y; z) với z 6= 0. Khi đó ta gọi (x0 ; y0 ; z0 ) là một nghiệm mà
z0 6= 0 và nhỏ nhất.
. .
Ta có x3 + 2y 3 = 4z 3 nên x3 ..2 ⇒ x0 ..2 ⇒ x0 = 2x1 . Khi đó
0 0 0 0

4x31 + y03 = 2z03 .

Tương tự, ta cũng có y0 = 2y1 và

hu
2x31 + 4y13 = z03 ⇒ z0 = 2z1

T
x31 + 2y13 = 4z13 .
Từ đó, suy ra (x1 ; y1 ; z1 ) cũng là một nghiệm của phương trình đã cho, mà z1 < z0 nên ta suy ra
t
điều vô lí.
Vậy phương trình chỉ có nghiệm duy nhất x = y = z = 0. 
Tấ
Ví dụ 2.2. Tìm các nghiệm tự nhiên của phương trình 2x − 1 = xy.

Lời giải.
n

Ta thấy (0; k) , k ∈ N và (1; 1) là nghiệm của phương trình. Ta chứng minh phương trình vô
nghiệm khi x ≥ 2 bằng phương pháp lùi vô hạn.
yễ

.
Giả sử phương trình có nghiệm x ≥ 2. Gọi p là ước nguyên tố bất của x, khi đó do 2x − 1..x nên
.
2x − 1 .. p.
.
gu

Mặt khác, theo định lí Phecma nhỏ thì 2p−1 − 1..p.


.
Gọi h là số nguyên dương nhỏ nhất để 2h − 1..p, khi đó
N

( (
h| x h |x
⇒ .
h| p − 1 h≤p−1<p

Gọi p1 là ước nguyên tố của h, khi đó p1 | x và p1 < p.


Tương tự vậy, ta xây dựng được dãy vô hạn các số nguyên tố p > p1 > p2 > . . . đều là ước của
x. Điều này vô lí vì x có hữu hạn ước nguyên tố.


Ví dụ 2.3 (TST KHTN Vòng 2 năm 2013). Tìm tất cả các bộ số (a,b,c,d)thỏa mãn

a2 + 7b2 = 3c2 + 2cd + 5d2 .

Lời giải.
Ta có
3a2 + 21b2 = (3c + d)2 + 14d2 ⇔ 3a2 − x2 = 7(2d2 − 3b2 ), x = 3c + d.

98
2. MỘT SỐ PHƯƠNG PHÁP GIẢI PHƯƠNG TRÌNH NGHIỆM NGUYÊN

.
Do một số chính phương chia 7 chỉ có thể nhận dư thuộc 0; 1; 2; 4 mà 3a2 − x2 .. 7 nên 7 |a; 7| x.
.
Suy ra 2d2 − 3b2 ..7, tương tự có 7 |d, 7| b.
Giả sử phương trình có nghiệm (a, b, x, d) khác (0, 0, 0, 0).
Đặt a1 , b1 , x1 , d1 là nghiệm có |a1 | + · · · + |d1 | nhỏ nhất.
Tương tự ta có 7|a1 ; . . . ; 7|d1 hay
a1 d1
a2 = , . . . ,d2 =
7 7
cũng là nghiệm của phương trình, mà |a2 | + · · · + |d2 | < |a1 | + · · · + |d1 |, vô lí.
Vậy pt chỉ có nghiệm a = b = c = d = 0.

Bài tập
Bài 2.1. Tìm nghiệm nguyên của phương trình:

x2 + y 2 + z 2 + t2 = 2xyzt.

hu
Lời giải.
Giả sử (x0 ,y0 ,z0 ,t0 ) là nghiệm nguyên của phương trình khi đó:

T
x20 + y02 + z02 + t20 = 2x0 y0 z0 t0 .

là số chẵn nên trong các số x0 ,y0 ,z0 ,t0 phải có số chẵn số lẻ (0; 2 hoặc 4 ).
t
• Nếu x0 ,y0 ,z0 ,t0 đều lẻ thì
Tấ
.
(x20 + y02 + z02 + t20 )..4,
.
trong khi đó 2x0 y0 z0 t0 6 ..4.

• Nếu trong các số x0 ,y0 ,z0 ,t0 có hai số lẻ thì (x20 + y02 + z02 + t20 ) ≡ 2(mod4), trong khi đó
n

.
2x0 y0 z0 t0 ..4.
yễ

Vậy x0 ,y0 ,z0 ,t0 phải là các số chẵn. Đặt

x0 = 2x1 , y0 = 2y1 , z0 = 2z1 , t0 = 2t1 .


gu

Phương trình trở thành:


x21 + y12 + z12 + t21 = 8x1 y1 z1 t1 .
N

Lý luận tương tự ta có:


x22 + y22 + z22 + t22 = 8x2 y2 z2 t2 .
x1 y1 z1 t1
Với x2 = ,y2 = ,z2 = ,t2 = , tiếp tục ta có:
2 2 2 2
x0 y0 z0 t0
xn = n
, yn = n , zn = n , tn = n ,
2 2 2 2
là số nguyên vơi mọi n, điều này chỉ xảy ra khi x0 = y0 = z0 = t0 = 0.
Vậy (0,0,0,0) là nghiệm duy nhất của phương trình đã cho. 
Bài 2.2. Chứng minh rằng phương trình x3 + 3y 3 = 9z 3 không có nghiệm nguyên dương.
Lời giải.
Dễ thấy nếu một trong ba số x,y,z bằng 0 thì hai số còn lại cũng bằng 0 nên (x,y,z) = (0,0,0) là
một nghiệm nguyên của (1).
Xét xyz 6= 0, ta sẽ chứng minh (1) vô nghiệm nguyên.

99
2. MỘT SỐ PHƯƠNG PHÁP GIẢI PHƯƠNG TRÌNH NGHIỆM NGUYÊN

Giả sử (1) có nghiệm nguyên. Theo nguyên tắc cực hạn tồn tại nghiệm (x0 ,y0 ,z0 ) ∈ Z của (1)
thỏa mãn (|x0 | + |y0 | + |z0 |) nhỏ nhất. Ta có

x30 + 3y03 = 9z03 . (2)


. . . .
Từ (2) vì 3y03 .. 3 và 9z03 .. 3 ⇒ x30 .. 3 ⇒ x0 .. 3. Đặt x0 = 3x1 . Khi đó

(2) ⇔ 27x31 + 3y03 = 9z03 ⇔ 9x31 + y03 = 3z03 . (3)


. .
Từ (3) suy ra y03 .. 3 ⇒ y0 .. 3. Đặt y0 = 3y1 . Khi đó

(3) ⇔ 9x31 + 27y13 = 3z03 ⇔ 3x31 + 9y13 = z03 . (4)


. .
Từ (4) suy ra z03 .. 3 ⇒ z0 .. 3. Đặt z0 = 3z1 . Tiếp tục

(4) ⇔ 3x31 + 9y13 = 27z13 ⇔ x31 + 3y13 = 9z13 .

hu
x y z 
0 0 0
Do đó bộ (x1 ,y1 ,z1 ) hay , , là một nghiệm của (1). Nhưng rõ ràng
3 3 3
x y z

T
0 0 0
|x1 | + |y1 | + |z1 | = + + < |x0 | + |y0 | + |z0 |
3 3 3
điều này mâu thuẫn với tính nhỏ nhất của tổng (|x0 | + |y0 | + |z0 |).
t
Vậy phương trình có nghiệm duy nhất (x,y,z) = (0,0,0).
Tấ
Nhận xét. Bằng phương pháp tương tự ta có thể chứng tỏ phương trình x3 + py 3 = p2 z 3 có
nghiệm duy nhất (x,y,z) = (0,0,0) với p là số nguyên tố bất kì hay tổng quát hơn là phương trình
xn1 +pxn2 +p2 xn3 +· · ·+pn−2 xnn−1 = pn−1 xnn có nghiệm nguyên duy nhất (x1 ,x2 , . . . ,xn ) = (0,0, . . . ,0).

n

Bài 2.3. Chứng minh rằng phương trình x2 + y 2 + z 2 = 2xyz không có nghiệm nguyên dương.
Lời giải.
yễ

Dễ thấy nếu một trong ba số x,y,z bằng 0 thì hai số còn lại cũng bằng 0 nên (x,y,z) = (0,0,0) là
một nghiệm nguyên của (1).
Xét xyz 6= 0, ta sẽ chứng minh (1) vô nghiệm nguyên.
gu

Thật vậy, giả sử trái lại, tức là phương trình (1) có nghiệm nguyên (x0 ,y0 ,z0 ) với x0 ,y0 ,z0 6= 0.
Theo nguyên tắc cực hạn, tồn tại nghiệm (x0 ,y0 ,z0 ) ∈ Z của (1) thỏa mãn |x0 | nhỏ nhất.
.
Nếu trong ba số không có số nào chẵn thì V T ≡ 1 (mod 2), mà V P .. 2 (vô lý).
N

.
Nếu trong ba số có ít nhất một số chẵn, giả sử z0 .. 2 ⇒ x20 + y02 ≡ 0 (mod 4), suy ra x0 , y0 đều
chẵn. Do đó, cả ba số x0 , y0 , z0 đều chẵn.
Đặt x0 = 2x1 , y0 = 2y1 , z0 = 2z1 (x1 ,y1 ,z1 ∈ Z), khi đó

(1) ⇔ 4x21 + 4y12 + 4z12 = 16x1 y1 z1 ⇔ x21 + y12 + z12 = 4x1 y1 z1 .

Nếu cả ba số x1 ,y1 ,z1 đều lẻ thì vế trái lẻ, trong khi vế phải chẵn, vô lí. Vậy phải có ít nhất một
.
số là số chẵn. Giả sử z1 .. 2 ⇒ x2 + y 2 ≡ 0 (mod 4), suy ra x1 và y1 đều chẵn. Do đó cả ba số
1 1
x1 ,y1 ,z1 đều chẵn, tức là đều chia hết cho 2. Từ đó suy ra x0 ,y0 ,z0 chia hết cho 22 . Lặp lại quá
trình trên suy ra x0 ,y0 ,z0 chia hết cho 2k , với k là số nguyên dương bất kì. Mà x0 cố định, trong
khi lim 2k = +∞ nên chỉ có thể x0 = 0, mâu thuẫn với trường hợp đang xét xyz 6= 0.
k→+∞
Vậy phương trình có nghiệm duy nhất (x,y,z) = (0,0,0). 
Bài 2.4. Tìm giá trị lớn nhất của m2 + n2 với m,n ∈ {1,2, . . . ,1981} thỏa mãn
2
n2 − mn − m2 = 1.

100
2. MỘT SỐ PHƯƠNG PHÁP GIẢI PHƯƠNG TRÌNH NGHIỆM NGUYÊN

Lời giải.
Ta thấy (m, n) = (1; 1) thỏa mãn phương trình (n2 − mn − m2 )2 = 1. Ngược lại với cặp (m, n)
thỏa mãn phương trình và 0 < m < n thì m < n < 2m và ta có biến đổi
2
(n2 − mn − m2 )2 = (n − m)2 + mn − 2m2
2
= (n − m)2 + m(n − m) − m2
2
= m2 − m(n − m) − (n − m)2 ,

nên ta có (n − m, m) thỏa mãn phương trình và 0 < n − m < m.


Do biến đổi (m, n) → (n − m, m) chỉ thực hiện được hữu hạn lần nên đến một lúc nào đó ta
sẽ thu được bộ (1, 1). Do đó, tất cả các cặp nghiệm của phương trình đều có thể bắt đầu từ bộ
(1, 1) và áp dụng biến đổi (m, n) → (n, m + n), ta thu được

(1, 1) → (2, 1) → (3, 2) → (5, 3) → · · ·

hu
Ta thấy tất cả các cặp nghiệm của phương trình là hai số hạng liên tiếp của dãy Fibonaci được
xác định như sau F1 = F2 = 1 và Fn+2 = Fn+1 + Fn với mọi n ≥ 1. Số hạng lớn nhất trong dãy
2 2
Fibonaci không vượt quá 1981 là F16 = 1597, nên đáp số của bài toán là F15 + F16 = 3514578.

T

Bài 2.5. Chứng minh rằng phương trình
t
x4 + y 4 = z 2 . (1)
Tấ
không có nghiệm nguyên dương.
Lời giải.
Giả sử phương trình (1) có nghiệm nguyên dương. Không mất tính tổng quát, ta xét các nghiệm
bộ 3 nguyên thủy (tức là (x2 ; y 2 ; z) = 1 do (1) là phương trình Pythagore).
n

. . .
Nếu x2 ; y 2 .. p (p nguyên tố) thì z 2 .. p ⇒ z .. p ⇒ (x2 ; y 2 ; z) > 1 (vô lý).
Tương tự suy ra x2 , y 2 , z đôi một nguyên tố cùng nhau.
yễ

Ta sẽ chứng minh x, y khác tính chẵn lẻ.


.
Thật vậy, nếu x, y cùng tính chẵn lẻ thì V P .. 4 (vô lý). Vậy x, y khác tính chẵn lẻ.
gu

Giả sử x chẵn, y lẻ. Theo nguyên tắc cực hạn tồn tại (x0 ; y0 ; z0 ) là nghiệm của (1) mà z0 nhỏ
nhất. Vì (1) có dạng phương trình Pythagore nên tồn tại m,n ∈ Z+ , m > n, (m,n) = 1 sao cho

x20 = 2mn; y02 = m2 − n2 ; z0 = m2 + n2 .


N

Có y02 = m2 − n2 ⇔ m2 = y02 + n2 ⇒ m lẻ. Mà y02 = 2mn = m(2n) nên (m,2n) = 1. Suy ra m và


.
2n đều là số chính phương, suy ra n .. 2. Đặt m = m2 . 1
Xét phương trình m2 = y02 + n2 dạng Pythagore nên tồn tại a,b ∈ N, a > b, (a,b) = 1 sao cho

m = a2 + b 2 (*)
y 0 = a2 − b 2
.
n = 2ab (do n .. 2)

Có n = 2ab ⇔ 2n = 4ab là số chính phương mà (a; b) = 1 ⇒ a; b là 2 số chính phương. Đặt


a = a21 , b = b21 . Khi đó (∗) có dạng m21 = a41 + b41 , suy ra (a1 ; b1 ; m1 ) là một nghiệm của (1) nhưng
m1 < z0 (mâu thuẫn). Do đó phương trình (1) vô nghiệm nguyên dương.
Vậy có điều phải chứng minh. 
Bài 2.6. Tìm tất cả các giá trị k sao cho phương trình (x + y + z)2 = kxyz có nghiệm nguyên
dương.

101
2. MỘT SỐ PHƯƠNG PHÁP GIẢI PHƯƠNG TRÌNH NGHIỆM NGUYÊN

Bài 2.7 (PTNK 2003). Tìm tất cả các số nguyên dương k sao cho phương trình
x2 − k 2 − 4 y 2 = −24


có nghiệm nguyên dương.


Bài 2.8. Giải phương trình nghiệm nguyên dương
x2 + y 2 + x + y + 1 = xyz.

Lời giải.
Ta chứng minh trước hết z = 5.
Gọi (x1 , y1 , z1 ) là một nghiệm của phương trình với z1 6= 5. Khi đó x1 6= y1 , nếu không thì
x1 [x2 (z1 − 2) − 2] = 1, điều này không thể xảy ra với z1 6= 5.
Ta có
0 = x21 + y12 + x1 + y1 + 1 − x1 y1 z1
= (y1 z1 − x1 − 1)2 + y12 + (y1 z1 − x1 − 1) + y1 + 1 − (y1 z1 − x1 − 1)y1 z1 ,

hu
nên (x1 ,y2 ,z2 ) = (y1 z1 − x1 − 1,y1 ,z1 ) là một nghiệm của phương trình, vì x1 (y1 z1 − x1 − 1) =
y12 + y1 + 1 > 0 nên x2 = y1 z1 − x1 − 1 > 0.
Lưu ý rằng nếu x1 > y1 , thì x1 ≥ y1 + 1 nên

T
x21 > y12 + y1 + 1 = x1 (y1 z1 − x1 − 1) = x1 x2 .
Nên x1 > x2 . Tiếp tục với cách làm tương tự như trên ta thu được dãy các nghiệm nguyên dương
(xk ,yk ,zk ) của phương trình với x1 > x2 > x3 > · · · , điều này không thể xảy ra.
t
Do đó ta có z = 5.
Tấ
Khi đó ta có phương trình
x2 + y 2 + x + y + 1 = 5xy.
Ta thấy x, y cùng lẻ. Đặt
3x − 1 3y − 1
u= ,v= (1)
n

2 2
thì phương trình đã cho trở thành
yễ

u2 − 5uv + v 2 = −3. (2)


Ta thấy (u0 ,v0 ) = (1,1) là một nghiệm của (2). Giả sử (u1 ,v1 ) là một nghiệm của (2) với u1 > v1 .
gu

Khi đó
v12 + (5v1 − u1 )2 + 3 = 5v1 (5v1 − u1 ),
nên (u2 ,v2 ) = (v1 ,5v1 − u1 ) là một nghiệm của (2). Từ
N

(u1 − v1 )(u1 − 4v1 ) = u21 − 5u1 v1 + 4v12 = 3v12 − 3 ≥ 0,


nên ta có u1 ≥ 4v1 , do đó v2 = 5v1 − u1 ≤ v1 . Do đó từ (u1 ,v1 ) ta xây dựng được dãy nghiệm
(u2 ,v2 ), (u3 ,v3 ), . . . với v1 ≥ v2 ≥ v3 ≥ · · · . Do đó tồn tại số nguyên dương k để vk+1 = 5vk − uk
và uk+1 = vk . Do đó
uk = vk−1 , k ≥ 1 và vk+1 = 5vk − vk−1 , v0 = 1, v1 = 4.
Từ đó ta tìm được
√ √ √ √
 !n−1 !n−1 
1 3 + 21 5 + 21 3 − 21 5 − 21
un = √  − 
21 2 2 2 2
" √ √ !n √ √ !n # (3)
1 3 + 21 5 + 21 3 − 21 5 − 21
vn = √ − , n ≥ 0.
21 2 2 2 2
 
2un + 1 2vn + 1
Vậy nghiệm của phương trình (1) là , , 5 với un , vn xác định bởi (3). 
3 3

102
2. MỘT SỐ PHƯƠNG PHÁP GIẢI PHƯƠNG TRÌNH NGHIỆM NGUYÊN

Bài 2.9. Cho dãy số (xn )+∞ +∞


n=1 và (yn )n=1 được xác định như sau

x0 = 1, x1 = 4, xn+2 = 3xn+1 − xn , ∀n = 0; 1; 2; . . .
y0 = 1, y1 = 2, yn+2 = 3yn+1 − yn , ∀n = 0; 1; 2; . . .

a) Chứng minh rằng x2n − 5yn2 + 4 = 0 với n = 0; 1; 2; . . .

b) Giả sử a, b là các số nguyên dương mà a2 − 5b2 + 4 = 0. Chứng minh tồn tại số tự nhiên k
sao cho xk = a và yk = b.

(VMO 1999)

Lời giải.

a) Ta dễ dàng chứng minh được công thức sau theo quy nạp
3 5 1 3
xn+1 = xn + yn ; yn+1 = xn + yn .

hu
2 2 2 2
Từ đó, ta lại chứng minh công thức x2n − 5yn2 + 4 = 0 theo quy nạp. (1)
Với n = 0 thì 1 − 5 + 4 = 0, suy ra (1) đúng.

T
Giả sử (1) đúng với n = k. Ta chứng minh (1) đúng với n = k + 1.
 2  2
3 5 1 3
Có x2k+1 − 5yk+1
2
+4= xk + y k − 5 xk + yk + 4 = x2k − 5yk2 + 4 = 0 đúng.
2 2 t 2 2
Do đó (1) đúng (đpcm).
Tấ
b) Xét phương trình x2 − 5y 2 + 4 = 0. (2)
Gọi (a0 ; b0 ) là một nghiệm nguyên dương bất kì của (2). Thay vào (2) ta có

a20 − 5b20 + 4 = 0.
n

Dựa vào phần a), ta đặt


yễ

 
a0 = 3 a1 + 5 b 1
 a1 = 3a0 − 5b0

2 2 ⇔ 2
1 3 3b0 − a0
gu

b 0 = a1 + b 1
 b1 =
 .
2 2 2
Bây giờ ta tìm điều kiện để a1 ,b1 là các số nguyên dương.
N

Vì a20 − 5b20 + 4 = 0 ⇔ a20 ≡ b20 (mod 4) ⇔ a0 và b0 khác tính chẵn lẻ nên a1 , b1 ∈ Z.


Vì a20 = 5b20 − 4 < 9b20 ⇔ a0 < 3b0 ⇒ b1 > 0.
 2
2 2 3 9
Để a1 > 0 ⇔ 3a0 > 5b0 ⇔ a0 = 5b0 − 4 ≤ 5 a0 − 4 = a20 − 4 ⇔ a20 ≥ 5 ⇔ a0 ≥ 3.
5 5
 ta xét a0 ≥ 3 thì sẽ có điều kiện sau: (a0 ; b0 ) là một nghiệm nguyên dương của
Do đó, nếu
3a0 − 5b0 3b0 − a0
(2) thì ; cũng là một nghiệm nguyên dương của (2). Từ đó ta xây
2 2
dựng một dãy giảm trong đó tất cả các bộ số đều là nghiệm nguyên dương của (2)
 
3 5 3 1
(∗) (a0 ; b0 ) −→ a0 − b0 ; b0 − a0 −→ · · ·
2 2 2 2

Theo nguyên tắc cực hạn tồn tại (m; n) là một bộ của dãy thỏa mãn (m + n) nhỏ nhất.
3m − 5n 3n − m
Suy ra (m1 ; n1 ) cũng là một bộ của dãy với m1 = , n1 = . Nhưng m1 +n1 =
2 2
m − n < m + n (vô lí). Vì vậy a0 hay m < 3 ⇒ m = 1; 2.
Nếu m = 2 ⇒ 5n2 = 8 (vô lí).

103
2. MỘT SỐ PHƯƠNG PHÁP GIẢI PHƯƠNG TRÌNH NGHIỆM NGUYÊN

Nếu m = 1 ⇒ 5n2 = 5 ⇒ n = 1.
Do đó, dãy (∗) kết thúc bằng bộ (1; 1) mà theo phần a) ta đã xây dựng được một ánh xạ
ngược
(∗∗) (1; 1) −→ (4; 2) −→ · · · ⇒ dãy (∗) và (∗∗) trùng nhau.
Vậy ta có điều phải chứng minh.
3 5 1 3
Nhận xét. Để tìm ra công thức xn+1 = xn + yn ; yn+1 = xn + yn ta có thể làm như sau:
2 2 2 2
Đặt xn+1 = axn + byn . Thay n = 0 và n = 1 ta có hệ phương trình

a = 3
( 
4=a+b 2
⇔ 5
11 = 4a + 2b b = .

2
3 5
Suy ra xn+1 = xn + yn . Từ đây ta chứng minh bằng quy nạp.
2 2

hu
1 3
Tương tự ta tìm ra yn+1 = xn + yn .
2 2


T
III. Phương pháp chọn môđunlô
Trong một số bài toán phương trình nghiệm nguyên, việc xét môđunlô có thể giúp ta hạn chế
t
được miền giá trị của các biến hoặc dẫn tới điều vô lí để từ đó kết luận được phương trình vô
Tấ
nghiệm.

Ví dụ 2.1 (Baltic Way 2016). Tìm cặp số nguyên tố (p; q) thỏa mãn

p3 − q 5 = (p + q)2 .
n
yễ

Lời giải.
Xét p, q 6= 3. Khi đó p3 ≡ ±1 (mod 3) và q 5 ≡ ±1 (mod 3).
gu

Nếu p ≡ q (mod 3) thì p3 − q 5 ≡ 0 (mod 3) và p + q ≡ ±1 (mod 3). Điều này dẫn tới vô lí.
Do đó, trong hai số p, q có ít nhất một số bằng 3.
N

• p = 3 ta có
8 − q 5 = (3 + q)2 ⇔ q 5 + q 2 + 6q + 1 = 0.
Phương trình này vô nghiệm.

• q = 3 ta có
p3 − 243 = (p + 3)2 ⇔ p3 − p2 − 6p − 252 = 0 ⇔ p = 7.

Vậy (p; q) = (7; 3) là cặp nghiệm cần tìm. 

Ví dụ 2.2. Tìm tất cả các số nguyên tố p sao cho tồn tại các số nguyên x,y thỏa mãn
(
p + 1 = 2x2
.
p2 + 1 = 2y 2

Lời giải.

104
2. MỘT SỐ PHƯƠNG PHÁP GIẢI PHƯƠNG TRÌNH NGHIỆM NGUYÊN

Từ đề bài ta có p > 2 và

x2 ≡ y 2 (mod p) ⇒ x ≡ ±y (mod p).

Mà x < y < p nên ta có

x ≡ −y (mod p) ⇒ x + y ≡ 0 (mod p) ⇒ x + y = p.

Suy ra

p2 + 1 = 2 (p − x)2 = 2p2 − 4xp + 2x2 = 2p2 − 4xp + p + 1


⇔ p2 − 4xp + p = 0 ⇔ p = 4x − 1.

Suy ra "
x = 0 → p = −1
4x = 2x2 ⇒ .
x=2→p=7

hu
Vậy p = 7 là giá trị cần tìm. 

Ví dụ 2.3 (Greek National mathematical olympiad 2016). Tìm tất cả các bộ số tự

T
nhiên (x,y,z) thỏa mãn
x2 + y 2 = 3 · 2016z + 77.
t
Tấ
Lời giải.
( ( ( (
2 2
x = 16 x = 16 x = 4 x=8
• Nếu z = 0 ta có x2 + y 2 = 80 ⇔ 2 ∨ 2 ⇔ ∨ .
y = 64 y = 64 y=8 y=4
n

• Xét z ≥ 1. Do 7|2016, 77 nên ta có


yễ


7 x2 + y 2 ⇒ 7 |x, y ,

hay x = 7a,y = 7b; a,b ∈ N∗ .


gu

Khi đó
7 a2 + b2 = 3 · 2016z−1 + 11.



N


7 7 a2 + b2 và 711


nên z − 1 = 0 ⇔ z = 1. Khi đó

a2 + b2 = 2 ⇔ a = b = 1.

Vậy (x; y; z) ∈ {(4; 8; 0) , (8; 4; 0) , (7; 7; 1)}.




Ví dụ 2.4 (Bankan MO 2016). Tìm tất cả các số nguyên a,b,c sao cho tồn tại số tự
nhiên n để
(a − b) (b − c) (c − a)
+ 2 = 2016n .
2

Lời giải.

105
2. MỘT SỐ PHƯƠNG PHÁP GIẢI PHƯƠNG TRÌNH NGHIỆM NGUYÊN

Đặt x = a − b,y = b − c,z = c − a ta có x + y + z = 0 nên x3 + y 3 + z 3 = 3xyz.


Giả sử
(a − b) (b − c) (c − a)
+ 2 = 2016n ,
2
với n ∈ N.
Ta có
(a − b) (b − c) (c − a) = 2.2016n − 4.
Suy ra
x3 + y 3 + z 3 = 6.2016n − 12. (∗)
Nếu n ≥ 1 thì từ (*) ta có
x3 + y 3 + z 3 ≡ 2 (mod 7). (1)
Mà x ∈ N thì x3 ≡ 0, ± 1 (mod 7) nên từ (1) ta suy ra trong ba số x, y, z có một số chia hết
cho 7 và hai số còn lại đồng dư với 1 theo modunlo 7. Nhưng khi đó
x+y+z ≡2 (mod 7),

hu
điều này vô lí vì x + y + z = 0 ≡ 0 (mod 7). Từ đó, suy ra n = 0. Khi đó
x3 + y 3 + z 3 = −6 ⇔ (x; y; z) = (−2; 1; 1) , (1; −2; 1) , (1; 1; −2) .

T
Xét 
a − b = −2
 (
a=b−2
b−c=1 ⇒ .
 tc=b−1
c−a=1

Tấ
Vậy các bộ cần tìm là: (a; b; c) = (k − 2; k; k − 1), k ∈ Z và các hoán vị.


Ví dụ 2.5 (IMO Shortlist 2002). Tìm số nguyên dương t nhỏ nhất sao cho tồn tại các
số nguyên x1 ,x2 , . . . ,xt thỏa
n

x31 + x32 + . . . + x3t = 20022002 .


yễ
gu

Lời giải.
Nếu 3| x thì x3 ≡ 0 (mod 9).
Nếu (x,3) = 1 thì x3 đồng dư 1 hoặc −1 (mod 9).
N

Như vậy, x31 + x32 + . . . + x3t đồng dư từ −t đến t (mod 9). Mà


20022002 ≡ 42002 ≡ 46·333 · 44 ≡ 44 ≡ 4 (mod 9).
Như vậy, nếu t ≤ 3 thì x31 + x32 + . . . + x3t không đồng dư 4 modulo 9 nên đẳng thức sau
x31 + x32 + . . . + x3t = 20022002
không xảy ra.
Với t = 4, ta thử tìm 1 nghiệm của phương trình
x31 + x32 + x33 + x34 = 20022002 .
3
Để ý 20022002 = 2002667 .2002, như vậy ta đặt xi = mi · 2002667 , với
m31 + m32 + m33 + m34 = 2002.
Ta chọn m1 = m2 = 10; m3 = m4 = 1 thỏa yêu cầu này.
Vậy tmin = 4.


106
2. MỘT SỐ PHƯƠNG PHÁP GIẢI PHƯƠNG TRÌNH NGHIỆM NGUYÊN

Ví dụ 2.6 (THTT số 423). Cho p ≥ 5 là số nguyên tố, n là số nguyên dương sao cho
các số p − 1, pn, n + 1 đôi một không có ước lớn hơn2. Chứng minh rằng phương trình sau
không có nghiệm nguyên dương x,y

2 + x + x2 + · · · + xp−1 = y n+1 . (1)

Lời giải.
Giả sử tồn tại các số nguyên dương x, y thỏa (1).

• x = 1, ta có p + 1 = y n+1 , suy ra

p = y n+1 − 1 = (y − 1)(y n + y n−1 + · · · + y + 1).

Do đó

hu
(
y−1=1
⇒ p = 1 + 2 + 22 + · · · + 2n = 2n+1 − 1.
y n + y n−1 + · · · + y + 1 = p

T
Suy ra 2n+1 ≡ 1 (mod p).
Gọi k là số nguyên dương nhỏ nhất thỏa 2k ≡ 1 (mod p), ta có k ≥ 3.
.
Vì 2n+1 ≡ 2p−1 ≡ 1 (mod p) ⇒ (n + 1,p − 1) .. k vô lí, vì (n + 1, p − 1) ≤ 2.
t
Tấ
• x ≥ 2, ta có

p−1 p−2 xp − 1
x +x + ··· + x + 1 = = y n+1 − 1 = (y − 1)(y n + y n−1 + · · · + y + 1).
x−1
xp − 1
n

Xét q là một ước nguyên tố bất kì của , ta có (x,q) = 1 nên xq−1 ≡ 1 (mod q).
x−1
. . .
yễ

Mà xp − 1 .. q và q là số nguyên tố nên x − 1 .. q hoặc q − 1 .. p.


Nếu x ≡ 1 (mod p) thì
gu

1 + x + x2 + · · · + xp−1 ≡ p (mod q),


.
suy ra p .. q hay p = q hoặc q ≡ 1 (mod p).
xp − 1
N

Suy ra tất cả các ước nguyên tố q của thì q = p hoặc q = 1 (mod p).
x−1
Dẫn tới hai số y − 1 và y n + y n−1 + · · · + y + 1 hoặc chia hết cho p hoặc có số dư là 1 khi
chia cho p.

– y − 1 ≡ 0 (mod p) ⇔ y ≡ 1 (mod p), suy ra

y n + y n−1 + · · · + y + 1 ≡ n + 1 (mod p).

Vì (n + 1,p) ≤ 2 nên pn + 1 và (n,p) ≤ 2 nên n + 11 (mod p). Do đó trường hợp này


không xảy ra
– y − 1 ≡ 1 (mod p) ⇔ y ≡ 2 (mod p).
Khi đó
y n + y n−1 + · · · + y + 1 ≡ 2n+1 − 1 (mod p).
Ta có 2n+1 − 10 (mod p) vì (n + 1,p − 1) ≤ 2, 2n+1 − 2 = 2 (2n − 1), mà (n,p − 1) ≤ 2
nên 2n − 10 (mod p). Do đó 2n+1 − 11 (mod p).

107
2. MỘT SỐ PHƯƠNG PHÁP GIẢI PHƯƠNG TRÌNH NGHIỆM NGUYÊN

Tóm lại trong các trường hợp ta đều có điều giả sử ban đầu là sai. Bài toán được chứng minh

Bài tập
Bài 2.1. Chứng minh rằng phương trình x5 − y 2 = 4 không có nghiệm nguyên.
Lời giải.
Ta xét theo modulo 11.
Ta có (x5 )2 = x10 ≡ 0 hoặc 1 (mod 11) với mọi x, nên x5 ≡ −1, 0, 1 (mod 11). Do đó x5 − 4 ≡
6, 7, 8 (mod 11). Mà một số chính phương khi chia cho 11 có các số dư 0, 1, 3, 4, 5, 9, do đó
phương trình đã cho không có nghệm nguyên. 
Bài 2.2. Chứng minh rằng phương trình sau không có nghiệm nguyên

(x + 1)2 + (x + 2)2 + · · · + (x + 2001)2 = y 2 .

Lời giải.
Đặt x = z − 1001, ta thu được phương trình

hu
(z − 1000)2 + (z − 999)2 + · · · + z 2 + · · · + (z + 1000)2 = y 2 ,

hay

T
2001z 2 + 2 12 + 22 + · · · + 10002 = y 2 .


Sử dụng đẳng thức


n (n + 1) (2n + 1)
12 + 22 + · · · + n2 = ,
t 6
Tấ
Ta thu được phương trình
2001z 2 + 1000 · 1001 · 667 = y 2 .
Ta có
2001z 2 + 1000 · 1001 · 667 ≡ 2 (mod 3)
n

và y 2 ≡ 0, 1 (mod 3) nên phương trình đã cho vô nghiệm.


yễ


Bài 2.3. Chứng minh rằng phương trình

x3 + y 4 = 7
gu

không có nghiệm nguyên.


Bài 2.4. Xác định tất cả nghiệm không âm (x1 ,x2 , . . . ,x14 ) của phương trình
N

x41 + x42 + · · · + x414 = 15999.

Bài 2.5. Tìm tất cả các cặp số nguyên (x; y) thỏa mãn

x3 − 4xy + y 3 + 1 = 0.

Bài 2.6. Chứng minh rằng phương trình

4xy − x − y = z 2

không có nghiệm nguyên dương.


Bài 2.7 (Titu Andreescu). Tìm các cặp số nguyên dương (x; y) thỏa mãn

x2 − y! = 2001.

108
2. MỘT SỐ PHƯƠNG PHÁP GIẢI PHƯƠNG TRÌNH NGHIỆM NGUYÊN

Bài 2.8. Tìm các số nguyên dương (x; y) thỏa mãn phương trình

3x − 2y = 7.

Bài 2.9. Tìm các số tự nhiên x,y,z thỏa mãn phương trình

5x · 7y + 4 = 3z .

Bài 2.10 (USAJMO 2013). Tìm tất cả các số nguyên a, b sao cho a5 b + 3 và ab5 + 3 đều là
lập phương của một số nguyên.
Bài 2.11 (Bosnhia TST 2014). Tìm tất cả các số nguyên không âm x, y sao cho 7x − 2.5y =
−1.
Bài 2.12 (Turkey JBMO TST 2016). Tìm tất cả các cặp số nguyên tố (p; q) thỏa mãn

p3 + 7q = q 9 + 5p2 + 18p.

hu
Bài 2.13 (AZE JBMO TST 2016). Tìm tất cả các cặp số tự nhiên (x,y,z) thỏa mãn

2013x + 2014y = 2015z .

T
Bài 2.14 (USAMO 2005). Chứng minh rằng hệ phương trình
(
t
x6 + x3 + x3 y + y = 147157
Tấ
x3 + x3 y + y 2 + y + z 9 = 157147

không có ngiệm nguyên x, y, z.


Bài 2.15 (Ấn Độ). Tìm nghiệm nguyên dương của phương trình 7x = 3y + 4.
n

Bài 2.16 (TST Thổ Nhĩ Kì 2013). Kí hiệu ϕ(n) là phi hàm Euler của n. Giải phương trình
yễ

nghiệm nguyên dương ( n ≥ 2) sau

2n + (n − ϕ(n) − 1)! = nm + 1.
gu

Bài 2.17 (Tukey TST 2017). Tìm các số nguyên dương m, n và số nguyên tố p thỏa mãn

m 3 + n n3 + m = p3 .
 
N

Bài 2.18 (IMO lần 23). Chứng minh rằng nếu n là số nguyên dương để phương trình

x3 − 3xy 2 + y 3 = n

có một nghiệm nguyên (x; y) thì phương trình có ít nhất ba nghiệm nguyên. Chứng minh rằng
phương trình không có nghiệm nguyên khi n = 2891.
Lời giải.
Ta có
x3 − 3xy 2 + y 3 = 2x3 − 3x2 y − x3 − 3x2 y + 3xy 2 − y 3


= 2x3 − 3x2 y + (y − x)3


= (y − x)3 − 3 (y − x) (−x)2 + (−x)3 .

x3 − 3xy 2 + y 3 = (x − y)3 + 3x2 y + 2y 3
= (x − y)3 − 3 (x − y) (−y)2 + (−y)3 .

109
2. MỘT SỐ PHƯƠNG PHÁP GIẢI PHƯƠNG TRÌNH NGHIỆM NGUYÊN

Do đó, nếu (x; y) là nghiệm nguyên của phương trình đã cho thì x2 + y 2 6= 0 và (y − x; −x) ,
(x − y; −y)
( cũng là nghiệm( của phương trình.
y−x=x x−y =x
Hơn nữa và đều dẫn tới x = y = 0. Do đó, nếu phương trình đã cho có
−x=y −y =x
một nghiệm thì sẽ có ít nhất 3 nghiệm.
Giả sử phương trình có nghiệm (x; y) với n = 2891.
Do 2891 ≡ 2 (mod 3) nên x3 + y 3 ≡ 2 (mod 3).
Suy ra x ≡ y ≡ 1 (mod 3) hoặc x ≡ −1 (mod 3),y ≡ 0 (mod 3) hoặc x ≡ 0 (mod 3), y ≡ −1
(mod 3).
Nếu x ≡ y ≡ 1 (mod 3) thì x − y ≡ 0 (mod 3), do đó nếu phương trình đã cho có nghiệm thì
.
sẽ có một giá trị chia hết cho 3. Giả sử x..3, khi đó x3 − 3xy 2 ≡ 0 (mod 9) nên y 3 ≡ 2891 ≡ 2
(mod 9).
Nhưng một số lập phương khi chia cho 9 có các số dư là 0, ± 1, ± 4, ± 7. Do đó, dẫn tới điều vô lí.
Vậy phương trình không có nghiệm nguyên khi n = 2891. 

hu
IV. Phương pháp quy nạp

Ví dụ 2.1. Chứng minh rằng với mọi số nguyên n ≥ 3, luôn tồn tại hai số nguyên dương

T
lẻ x, y sao cho 7x2 + y 2 = 2n .

Lời giải. t
Chúng ta chứng minh tồn tại các số nguyên dương lẻ xn , yn sao cho
Tấ
7x2n + yn2 = 2n , n ≥ 3.

Với n = 3, ta chọn x3 = y3 = 1.
Giả sử tồn tại các số nguyên dương lẻ xn ,yn thỏa
n

7x2n + yn2 = 2n .
yễ

Ta chứng minh tồn tại số nguyên dương lẻ xn+1 ,yn+1 thỏa

7x2n+1 + yn+1
2
= 2n+1 .
gu

Ta có  2  2
xn ± y n 7xn ∓ yn
= 2 7x2n + yn2 = 2n+1 .

7 +
N

2 2
xn − y n xn + yn
Hơn nữa, do xn , yn lẻ nên trong hai số và có ít nhất một số lẻ. Chẳng hạn, ta
2 2
xn − y n
giả sử lẻ, khi đó
2
7xn + yn y n − xn
= 4xn +
2 2
là số nguyên dương lẻ.
|xn − yn | 7xn + yn
Ta chọn xn+1 = và yn+1 = . Ta có đpcm.
2 2


Ví dụ 2.2 (VMO 2010). Chứng minh rằng với mỗi số nguyên dương n , phương trình

x2 + 15y 2 = 4n

có ít nhất n nghiệm tự nhiên (x,y) .

110
2. MỘT SỐ PHƯƠNG PHÁP GIẢI PHƯƠNG TRÌNH NGHIỆM NGUYÊN

Lời giải.
Với n = 1, phương trình x2 + 15y 2 = 4n có một nghiệm tự nhiên là (x, y) = (2, 0).
Với n = 2, phương trình x2 + 15y 2 = 4n có 2 nghiệm tự nhiên là (x, y) = (4, 0); (1, 1).
Giả sử với n ≥ 2, phương trình x2 +15y 2 = 4n có n nghiệm tự nhiên là (x1 , y1 ), (x2 , y2 ), · · · , (xn , yn )
khi đó (x, y) = (2xk , 2yk ) (1 ≤ k ≤ n) là các nghiệm tự nhiên của phương trình

x2 + 15y 2 = 4n + 1.

Ta chứng minh rằng với mỗi số nguyên n ≥ 2 tồn tại cặp số nguyên dương lẻ (xn , yn ) sao cho
sao cho x2n + 15yn2 = 4n .
Thật vậy với n = 2 , chọn x2 = 1, y2 = 1.
Giả sử với n ≥ 2 tồn tại cặp số nguyên dương lẻ (xn , yn ) sao cho sao cho x2n + 15yn2 = 4n . Ta
chứng minh rằng mỗi cặp
   
|15yn − xn | xn + y n 15yn + xn |yn − xn |
X= ,Y = , X= ,Y =
2 2 2 2

hu
thoả măn X 2 + 15Y 2 = 4n+1 .
Thật vậy  2  2
15yn ∓ xn y n ± xn
= 4 x2n + 15yn2 = 4 · 4n = 4n+1 .


T
+ 15
2 2
và xn , yn lẻ nên
t
xn = 2k + 1, yn = 2l + 1 (k, l ∈ Z) ⇒
xn + y n
=k+l+1
Tấ
2

|yn − xn | |(2l + 1) − (2k + 1)|
= = |l − k| .
2 2
n

xn + yn |yn − xn |
Điều đó chứng tỏ rằng một trong các số , là lẻ .
2 2
yễ

2
Vì vậy với n + 1 tồn tại các số tự nhiên lẻ xn+1 và yn+1 thoả măn x2n+1 + 15yn+1 = 4n+1 .
Từ đó ta có điều phải chứng minh. 
gu

Ví dụ 2.3. Tìm các số nguyên dương phân biệt x1 ,x2 , . . . ,xm thỏa phương trình

x31 + x32 + · · · + x3m = (x1 + x2 + · · · + xm )2 .


N

Lời giải.
Trước hết ta chứng minh bổ đề sau
Bổ đề 1. Cho các số nguyên dương phân biệt a1 ,a2 , . . . ,an . Khi đó

a31 + a32 + . . . + a3n ≥ (a1 + a2 + . . . + an )2 . (1)

đúng với mọi n ≥ 1.


Chứng minh. Không mất tính tổng quát, ta giả sử a1 < a2 < · · · < an .
Với n = 1 ta có a31 ≥ a21 nên (1) đúng.
Giả sử (1) đúng với n = k và a1 < a2 < · · · < ak < ak+1 là các số nguyên dương. Khi đó
ak+1 ≥ ak + 1, do đó

(ak+1 − 1) ak+1 ak (ak + 1)


≥ = 1 + 2 + · · · + ak .
2 2

111
2. MỘT SỐ PHƯƠNG PHÁP GIẢI PHƯƠNG TRÌNH NGHIỆM NGUYÊN

Vì 1 + 2 + · · · + ak là tổng của ak số nguyên dương đầu tiên, nên

1 + 2 + · · · + ak ≥ a1 + a2 + · · · + ak .

Suy ra
(ak+1 − 1) ak+1
≥ a1 + a2 + · · · + ak .
2
Hay
a3k+1 ≥ 2ak+1 (a1 + a2 + · · · + ak ) + a2k+1 .
Do đó
a31 + a32 + · · · + a3k + a3k+1 ≥ (a1 + a2 + · · · + ak )2 + 2ak+1 (a1 + a2 + · · · + ak ) + a2k+1
= (a1 + a2 + · · · + ak+1 )2 .

Suy ra bổ đề được chứng minh.


Trở lại bài toán. Không mất tính tổng quát, ta giả sử x1 < x2 < · · · < xm . Khi đó x1 ≥ 1,x2 ≥

hu
2, · · · ,xm ≥ m và

xm−1 ≤ xm − 1, xm−2 ≤ xm − 2, · · · , x1 ≤ xm − (m − 1) .

T
Suy ra
m (m − 1)
x1 + x2 + · · · + xm−1 ≤ (m − 1) xm − . (2)
t 2
Áp dụng bổ đề ta có
Tấ
x31 + x32 + · · · + x3m−1 ≥ (x1 + x2 + · · · + xm−1 )2 . (3)

Mặt khác, từ phương trình ta thu được

x31 + x32 + · · · + x3m−1 + x3m = x2m + 2xm (x1 + x2 + · · · + xm−1 ) + x2m−1 .


n

(4)
yễ

Từ (3) và (4) ta có
x3m ≤ 2xm (x1 + x2 + · · · + xm−1 ) + x2m ,
hay
gu

x2m ≤ 2 (x1 + x2 + · · · + xm−1 ) + xm .


Kết hợp với (2) ta thu được
N

x2m ≤ 2 (m − 1) xm − m (m − 1) + xm ,

hay
(xm − m) (xm − (m − 1)) ≤ 0.
Vì xm > m − 1 nên ta có xm = m.
Vậy x1 = 1, x2 = 2, . . . , xm = m và các hoán vị là các số cần tìm.

Bài tập
Bài 2.1. Chứng minh rằng với mọi số nguyên dương n, phương trình

x2 + y 2 + z 2 = 59n

luôn có nghiệm nguyên dương x, y, z.


Lời giải.
Xét các bộ (x1 ; y1 ; z1 ) = (1; 3; 7) và (x2 ; y2 ; z2 ) = (14; 39; 42), ta có
x21 + y12 + z12 = 59 và x22 + y22 + z22 = 592 .

112
2. MỘT SỐ PHƯƠNG PHÁP GIẢI PHƯƠNG TRÌNH NGHIỆM NGUYÊN

Do đó, phương trình có nghiệm nguyên dương khi n = 1, n = 2.


Giả sử phương trình
x2 + y 2 + z 2 = 59n
có nghiệm nguyên dương xn , yn , zn .
Đặt xn+2 = 59xn , yn+2 = 59yn , zn+2 = 59zn , khi đó
2
x2n+2 + yn+2 2
= 592 x2n + yn2 + zn2 = 59n+2 .

+ zn+2

Hay phương trình


x2 + y 2 + z 2 = 59n+2
có nghiệm nguyên dương xn+2 , yn+2 , zn+2 .
Nên theo phương pháp quy nạp với bước nhảy s = 2 ta có đpcm.

Bài 2.2. Chứng minh rằng với mọi số nguyên dương n ≥ 3, phương trình
1 1 1
+ + ··· + =1 (1)

hu
x1 x2 xn
luôn có nghiệm nguyên dương phân biệt x1 , x2 , . . . , xn .
Lời giải.

T
Ta có
1 1 1
+ + = 1,
2 3 6 t
nên phương trình có nghiệm nguyên dương phân biệt khi n = 3.
Tấ
Giả sử tồn tại các số nguyên dương phân biệt x1 , x2 , . . . ,xn thỏa mãn
1 1 1
+ + ··· + = 1.
x1 x 2 xn
Khi đó
n

1 1 1 1
+ + ··· + = ,
2x1 2x2 2xn 2
yễ

nên
1 1 1 1
+ + + ··· + = 1.
2 2x1 2x2 2xn
gu

Suy ra (2; 2x1 ; 2x2 ; . . . ; 2xn ) là nghiệm nguyên dương phân biệt của phương trình
1 1 1 1
+ + ··· + + = 1.
N

x1 x 2 xn xn+1
Vậy bài toán được chứng minh.
Nhận xét.
a) Ta có
n−1 n−1
Xk+1−1 X 1 X n−1 n−1
X k 1 1
= = − =1− ,
k=1
(k + 1)! k=1 (k + 1)! k=1
k! k=1 (k + 1)! n!

Nên
1 1 1 1 1
+ + + ··· + + = 1.
2! 3! 4! n! n!
1 2 3 n−1
Do đó  
2! 3! n!
(x1 ; x2 ; . . . ; xn ) = ; ;...; ; n!
1 2 n−1
thỏa phương trình đã cho.

113
2. MỘT SỐ PHƯƠNG PHÁP GIẢI PHƯƠNG TRÌNH NGHIỆM NGUYÊN

b) Ta có một bộ nghiệm nguyên dương phân biệt của phương trình đã cho là

2,22 ,23 , . . . ,2n−2 ,2n−2 + 1,2n−2 2n−2 + 1 .




Thật vậy
1 1 1 1 1
+ 2 + · · · + n−2 + n−2 + n−2 n−2
2 2 2 2 +1 2 (2 + 1)
n−2
1 2 1
= 1 − n−2 + n−2 n−2 + n−2 n−2
2 2 (2 + 1) 2 (2 + 1)
1 1
= 1 − n−2 + n−2 = 1.
2 2
c) Xét dãy số (an ) : a1 = 2 và
an+1 = a2n − an + 1, n ≥ 1.
Ta có an ∈ Z+ và an+1 > an ∀n ≥ 1.
Hơn nữa an+1 − 1 = an (an − 1), suy ra

hu
1 1 1 1
= = − .
an+1 − 1 an (an − 1) an − 1 an

T
hay
1 1 1
= − .
an an − 1 an+1 − 1
t
Do đó
Tấ
n−1
X 1 1 1
= − ,
a k a 1−1 a n−1
k=1
suy ra
1 1 1 1
+ + ··· + + = 1.
n

a1 a2 an−1 an − 1
yễ

Từ đây ta có (a1 ,a2 , . . . ,an−1 ,an − 1) là một bộ nghiệm thỏa yêu cầu bài toán.

d) Người ta chưa biết được có tồn tại vô hạn số nguyên dương n sao cho phương trình (1) có
nghiệm nguyên dương (x1 ,x2 , · · · ,xn ) mà x1 ,x2 , . . . ,xn là các số lẻ phân biệt.
gu


Bài 2.3. Chứng minh rằng với mọi số nguyên n ≥ 412, luôn tồn tại các số nguyên dương
N

x1 , x2 . . . , xn thỏa mãn
1 1 1
3
+ 3 · · · + 3 = 1. (1)
x1 x2 xn

Lời giải.
Ta có
1 1 1 1
= 3 + 3 ··· +
a 3
(2a) (2a) (2a)3
trong đó ở vế phải là tổng của 8 số hạng.
Do đó, nếu phương trình
1 1 1
3
+ 3 ··· + 3 = 1
x1 x2 xn
có nghiệm nguyên dương thì phương trình
1 1 1
3
+ 3 ··· + 3 = 1
x1 x2 xn+7

114
2. MỘT SỐ PHƯƠNG PHÁP GIẢI PHƯƠNG TRÌNH NGHIỆM NGUYÊN

cũng có nghiệm nguyên dương.


Do đó, ta chứng minh bài toán bằng phương pháp quy nạp toán học với bước nhảy s = 7, điều
này thực hiện được nếu ta chứng minh được phương trình (1) có nghiệm khi

n = 412, 413, 414, 415, 416, 417, 418.

Ý tưởng chính để thực hiện điều này là ta chứng minh điều này là ta chứng minh các trường hợp
n nhỏ rồi xét theo modulo 7. Ta có
27
• = 1 và 27 ≡ 412 (mod 7).
33
4 9 36
• 3
+ 3 + 3 = 1 và 4 + 6 + 36 ≡ 413 (mod 7).
2 3 6
4 32
• + = 1 và 4 + 32 ≡ 414 (mod 7) .
23 43
18 243

hu
• + 3 = 1 và 18 + 243 ≡ 415 (mod 7) .
33 9
18 16 144
• + 3 + 3 = 1 và 18 + 16 + 144 ≡ 416 (mod 7).
33 4 12

T
4 16 36 144
• 3
+ 3 + 3 + 3 = 1 và 4 + 16 + 36 + 144 ≡ 417 (mod 7).
2 4 6 12 t
4 9 81 324
• + 3 + 3 + 3 = 1 và 4 + 9 + 81 + 324 ≡ 418 (mod 7).
Tấ
23 3 9 18
Vậy bài toán được chứng minh.

Bài 2.4. Chứng minh rằng với mọi số nguyên n ≥ 2, luôn tồn tại các số nguyên lẻ x,y sao cho
n

2
x − 17y 2 = 4n .
yễ

Bài 2.5. Chứng minh rằng với mọi số nguyên dương n, phương trình
gu

x2 + xy + y 2 = 7n

luôn có nghiệm nguyên x, y.


N

Bài 2.6. Chứng minh rằng với mọi số nguyên n ≥ 1, luôn tồn tại các số nguyên x, y, z thỏa
mãn
n
x2 + y 2 + z 2 = 32 .
k (k + 1)
Bài 2.7. Số nguyên tk = được gọi là số tam giác thứ k với k ≥ 1. Chứng minh rằng
2
với mọi số nguyên n ≥ 3 thì phương trình
1 1 1
+ + ··· + = 1,
x1 x 2 xn

luôn có nghiệm trong tập các số tam giác.


Bài 2.8. Chứng minh rằng với mọi n≥6,ˆ phương trình

1 1 1
2
+ 2 + ··· + 2 = 1
x1 x2 xn

luôn có nghiệm nguyên dương.

115
2. MỘT SỐ PHƯƠNG PHÁP GIẢI PHƯƠNG TRÌNH NGHIỆM NGUYÊN

Bài 2.9. Chứng minh rằng với mọi số nguyên s ≥ 2, tồn tại các số nguyên dương x0 ,x1 , . . . ,xs
sao cho
1 1 1 1
2
+ 2 + ··· + 2 = 2,
x1 x2 xs x0
Với x0 < x1 < x2 < · · · < xs .
Bài 2.10. Chứng minh rằng với mọi số tự nhiên k, phương trình

x2 + y 2 = z 2 + k

luôn có nghiệm nguyên dương x, y, z với x < y < z.


Bài 2.11. Chứng minh rằng phương trình

(x + 1)2 + x2 = y 2

có vô số nghiệm nguyên dương x, y.

hu
Bài 2.12. Tìm các số nguyên dương phân biệt x1 ,x2 , . . . ,x2002 thỏa phương trình

x21 + x22 · · · + x22002 = 1335 (x1 + x2 + · · · + x2002 ) .

T
t
Tấ
n
yễ
gu
N

116

You might also like